Пределы с е: Число e, замечательные пределы — определение, первый и второй замечательный предел, примеры

Второй замечательный предел, следствия, примеры

Второй замечательный (особый) предел часто вызывает трудности у студентов, хотя сам предел довольно прост и понятен на практике. Он позволяет раскрывать неопределенности вида единица в степени бесконечность . Замечательный предел имеет следующий вид

где «е»-экспонента.

Следствия второго замечательного предела

1)


2)
3)
4)
5)
6)

На практике следствия второго предела реже встречаются на практике чем он сам, однако без них некоторые задачи в простой способ не решить.

Примеры на замечательный предел

Рассмотрим некоторые примеры из сборника А.В. Тевяшев, А.Г. Литвин, Г.М. Кривошеева и др.»Высшая математика в примерах и задачах. Ч.5 Тесты» (Харьков, 2007, ст. 99).

Пример 6.1. Найти предел функции
а)

Решение.
Преобразуем функцию к виду при котором возможно применить формулу замечательного предела

В результате можем применить правило замечательного предела

 

б)

Решение.
Подобно предыдущему примеру превращаем функцию в скобках чтобы применить замечательный предел

Нужно отметить, что в этом примере и во многих подобных константы в степенях, как правило вклада не несут. Функцию можно расписать следующим образом

Предел умышленно расписан в виде произведения двух множителей чтобы Вы убедились что константы в степенях вклада не несут. Их цель запутать Вас, если плохо знаете теоретический материал или сомневаетесь в правильности решения. Во всех последующих примерах мы не будем расписывать примеры на произведение двух границ, однако помните, что они не меняют конечного результата (вклад — множитель единица).

————————————

в)

Решение.
Выполняем преобразование заданной функции

Запись в таком виде сделана специально, потому что степень нужно свести к подобному виду

В такой простой способ получили искомый предел функции. В дальнейшем необходимые замены или подсказки будут выделены цветом из общего решения.

 

г)

Решение.
Выполним замену переменных в пределе

и определенные преобразования для нахождения предела

Бывают случаи, когда прямо применить правило второго замечательного предела довольно сложно, в таких ситуациях используйте простые замены которые Вам понятны и позволяют в быстрый способ найти предел.

 

Пример 6. 2 Вычислить предел функции

а)

Решение.
Сводим функцию к правилу замечательного предела

Подставляем и вычисляем, выполняя нужные манипуляции с показателями

 

в)

Решение.
За известным уже алгоритмом преобразуем функцию

Применяя определение второго важного предела находим

 

Пример 6. 3 Определить предел функции
б)

Решение.
Сведем функцию для применения замечательного предела

Подставляем в границу и упрощаем

 

г)

Решение.
«Как найти предел ? — скажете Вы, ведь переменная равна минус бесконечности.
В этом примере видим что аргумент стремится к минус бесконечности, кроме того функция в скобках следует не до единицы, а до 2 при больших аргументах.

Учитывая что степень отрицательный получим следующее значение предела

Во всех примерах второго замечательного предела следует сначала проверять условие что выражение в скобках стремится к единице. Если нет, то предел функции в зависимости от степени будет равен или нулю или бесконечности. Те из Вас кто часто решает примеры такие проверки осуществляет автоматически. Остальные сводят границу в экспоненте в определенном степени, но все равно вылезает множителем или ноль или бесконечность. В конечном варианте правы все, однако в первом случае тратится гораздо меньше времени, которое так необходима на контрольных работах, тестах, ВНО. Поэтому выбирайте для себя простой путь и делайте в обучении правильные выводы.

 

Пример 6. 5 Найти предел функции
а)

Решение.
Заданный пример на вид отличается от предыдущих, однако решение получаем по такой же схеме. Выполняем преобразования функции в скобках под правило замечательного предела

Осталось в степени выделить обратный множитель

и подставить в границу

По такой схеме вычисляйте все подобные пределы, она проста и не требует дополнительных пояснений.

————————————

в)

Решение.
К рассматриваемому примеру великих преобразований делать не нужно. Он имеет достаточно простую запись и решение осуществляем в одну строку

Практикуйте с подобными пределами, используйте удобные для себя схемы сведения задач под необходимое правило. Не бойтесь делать ошибки — без них обучение не обходится!

Второй замечательный предел

Вторым замечательным пределом называется предел

,

где

— иррациональное число.

Непосредственная подстановка бесконечности в выражение приводит к бесконечности вида .

Значит, если при непосредственном вычислении предела у вас получилась неопределённость такого вида, то решать задачу следует путём приведения ко второму замечательному пределу. Во всех этих задачах для получения второго замечательного предела требуется производить замену сложной функции более простой.

Второй замечательный предел может быть записан в другом виде, если положить тогда .

Из условия

получим

   (Alt)

Пример 1. Найти предел .

Решение. Подстановка вместо x бесконечности приводит к неопределённости:

.

Значит, нужно привести выражение ко второму замечательному пределу. Облегчим себе жизнь перед заменой сложной функции более простой, представив степень :

.

Заменяем функцию 6x переменной n, которая также стремится к бесконечности:

.

Это второй замечательный предел, индивидуальна только степень числа е:

.

А проверить решение задачи на пределы можно на калькуляторе пределов онлайн.

Пример 2. Найти предел .

Решение. Непосредственная подстановка приводит к неопределённости «бесконечность делить на бесконечность в степени бесконечность»:

.

Бесконечность в показателе степени — признак того, что выражение можно привести к отношению двух вторых замечательных пределов. В самом деле, если числитель и знаменатель поделить почленно на x, то слева и в числителе и в знаменателе будет уже по единице:

.

Почти второй замечательный предел. А чтобы это было не почти, а вторым замечательным пределом, нужно, чтобы во вторых слагаемых и в числителе, и в знаменателе были единицы. Для этого произведём замены функций:

.

.

Подставляем и получаем:

.

Это уже отношение вторых замечательных пределов, а степени выражений в числителе и знаменателе — индивидуальны:

.

Пример 3. Найти предел

Решение. Применяем разновидность (Alt) второго замечательного предела:

Проверить решение задачи на пределы можно на калькуляторе пределов онлайн.


Второй замечательный предел служит средством решения многих задач физики, биологии, социальных наук. Показательная функция с основанием e возникает при выводе количественного закона, которому подчиняются многие естественные процессы: рост народонаселения, рост количества древесины на лесных массивах, радиоактивный распад и т.д.

Для вывода этого закона используется формула сложных процентов

,

где — сумма, наращенная через t лет, — начальная сумма, p — процентная такса, t — время роста в годах.

При этом предполагается, что проценты присоединяются к начальной сумме в конце каждого года. Если же ввести условие присоединения процентов по отдельным частям года, равным 1/n доле его, а процентная такса p по-прежнему пусть относится к целому году, то по истечении каждой такой части года наращенные суммы соответственно составят

По прошествии одного года начальная сумма обратится в , по прошествии двух лет — в , по прошествии t лет — в .

Если же предположить, что прирост процентов происходит непрерывно, т. е. число промежутков, на которые делится год, неограниченно возрастает (), а каждый из них стремится к нулю, то величина наращенной суммы выразится следующей формулой:

,

очень напоминающей второй замечательный предел.

Чтобы выразить этот предел через e, положим , тогда . Из условия следует, что , поэтому

Используя формулу альтернативного представления второго замечательного предела (Alt), приведённую в начале статьи, получим показательный закон роста:

.

Заменив p на -p, получим показательный закон убывания:

.

Например, если население страны возрастает на 2% в год, то по формуле показательного закона роста можно с неплохим приближением рассчитать численность населения страны через t лет: , где — численность населения в начале отсчёта.

НазадЛистатьВперёд>>>

Нет времени вникать в решение? Можно заказать работу!

К началу страницы

Пройти тест по теме Предел

Начало темы «Предел»

Что такое предел функции и как его найти

Первый замечательный предел

Продолжение темы «Предел»

Бесконечно малые

Исчисление I. Пределы в бесконечности, часть II

Показать мобильное уведомление Показать все примечания Скрыть все примечания

Мобильное уведомление

Похоже, вы находитесь на устройстве с «узкой» шириной экрана ( т. е. вы, вероятно, на мобильном телефоне). Из-за характера математики на этом сайте лучше всего просматривать в ландшафтном режиме. Если ваше устройство не находится в ландшафтном режиме, многие уравнения будут отображаться сбоку вашего устройства (должна быть возможность прокрутки, чтобы увидеть их), а некоторые пункты меню будут обрезаны из-за узкой ширины экрана.

Раздел 2.8: Пределы на бесконечности, Часть II

В предыдущем разделе мы рассмотрели пределы на бесконечность многочленов и/или рациональные выражения, включающие многочлены. В этом разделе мы хотим взглянуть на некоторые другие типы функций, которые часто проявляются в пределах бесконечности. Функции, которые мы будем здесь рассматривать, — это экспоненты, натуральные логарифмы и арктангенсы. 9{-x}} = \infty\]

Суть этого примера заключалась в том, чтобы показать, что если показатель экспоненты стремится к бесконечности в пределе, то экспоненциальная функция также стремится к бесконечности в пределе. Точно так же, если экспонента стремится к минус бесконечности в пределе, тогда экспонента в пределе стремится к нулю.

Заметим также, что в последнем разделе значение предела не зависело от того, ушли ли мы в плюс или минус бесконечность. В приведенном выше примере мы уже видели, что изменение знака бесконечности может изменить ответ, поэтому не замыкайтесь на каких-либо предположениях, которые вы могли сделать из работы в предыдущем разделе! 9{ — 15x}}} \справа)\)

Показать все решения Скрыть все решения

Итак, единственная разница между этими двумя пределами заключается в том, что в первом мы берем предел, когда мы идем к плюс бесконечности, а во втором мы идем к минус бесконечности. К этому моменту мы смогли «повторно использовать» работу из первого ограничения, по крайней мере, в части второго ограничения. С экспонентами, которые часто не будут иметь место, мы собираемся рассматривать каждую из них как отдельные проблемы. 9{ — 15x}}} \right) = \infty — \infty + \infty + 0 — 0\]

Последние два термина не проблема (однако они будут в следующей части, понимаете?). Однако первые три представляют собой проблему, поскольку они представляют нам еще одну неопределенную форму.

При работе с многочленами мы выносим на множитель член с наибольшим показателем в нем. Давайте сделаем то же самое здесь. Однако теперь нам приходится иметь дело как с положительными, так и с отрицательными показателями, а также с тем, что мы подразумеваем под «самым большим» показателем. Имея дело с ними здесь, мы смотрим на термины, вызывающие проблемы, и спрашиваем: «Каков наибольший показатель степени в этих терминах?». Итак, поскольку проблемы вызывают только первые три члена ( 9{ — 25x}}} \справа)} \справа]\]

Обратите внимание, что при этом факторинге все оставшиеся экспоненты теперь имеют отрицательные показатели, и мы знаем, что для этого предела (, т.е. , уходящего в положительную бесконечность) все они будут равны нулю в пределе, и поэтому больше не будут вызывать проблем.

Теперь мы можем взять предел двух факторов. Первое — это явно бесконечность, а второе — конечное число (в данном случае — единица), поэтому раздел «Факты из бесконечных пределов» дает нам следующий предел: 9{ — 15x}}} \right)\) Показать решение

Давайте начнем так же, как и в первой части. Возьмем предел каждой из частей. На этот раз обратите внимание, что, поскольку наш предел стремится к отрицательной бесконечности, первые три экспоненты фактически устремляются к нулю (потому что их показатели степени стремятся к минус бесконечности в пределе). Последние две экспоненты в пределе уйдут в бесконечность (потому что их показатели в пределе уйдут в плюс бесконечность).

Получение пределов дает, 9{ — 15x}}} \right) = 0 — 0 + 0 + \infty — \infty \]

Таким образом, последние два члена являются здесь проблемой, поскольку они снова оставляют нас с неопределенной формой. Как и в первом примере, мы собираемся исключить «наибольшую» экспоненту в последних двух терминах. { — 15x}}\). 9{ — 15x}}} \справа) = — \infty \]

Итак, при работе с суммами и/или разностями показательных функций мы ищем экспоненту с «наибольшей» степенью и помним здесь, что «наибольшая» означает степень, наиболее далекую от нуля. Также помните, что если мы смотрим на предел плюс бесконечность, только экспоненты с положительными показателями будут вызывать проблемы, поэтому это единственные условия, на которые мы обращаем внимание при определении наибольшего показателя. Точно так же, если мы смотрим на предел минус бесконечность, то только экспоненты с отрицательными показателями будут вызывать проблемы, и поэтому только они рассматриваются при определении наибольшего показателя.

Наконец, как вы могли догадаться из предыдущего примера, при работе с суммой и/или разностью экспонент все, что нам нужно сделать, это посмотреть на наибольшую экспоненту, чтобы определить поведение всего выражения. Опять же, помня, что если предел равен плюс бесконечности, мы рассматриваем экспоненты только с положительными показателями, а если мы смотрим на предел минус бесконечность, мы рассматриваем только экспоненты с отрицательными показателями.

Давайте теперь рассмотрим некоторые рациональные функции, включающие экспоненты. 9{ — x}}}}\) Показать решение

Основная концепция, связанная с решением этой задачи, такая же, как и с рациональными выражениями в предыдущем разделе. Мы смотрим на знаменатель и определяем экспоненциальную функцию с «наибольшей» степенью, которую мы затем выносим как из числителя, так и из знаменателя. Мы будем использовать те же рассуждения, что и в предыдущем примере, чтобы определить «наибольшую» экспоненту. В случае, когда мы рассматриваем предел плюс бесконечность, мы рассматриваем экспоненты только с положительными показателями. 9{ — x}}}}\) Показать решение

В этом случае мы собираемся минус бесконечность в пределе, поэтому мы будем смотреть на экспоненты в знаменателе с отрицательными показателями при определении «наибольшего» показателя. Однако в этой задаче есть только один, поэтому мы будем использовать его.

Опять же, не забывайте смотреть только на знаменатель. НЕ используйте экспоненту из числителя, даже если она «больше», чем экспонента в знаменателе. Мы всегда смотрим только на знаменатель, когда определяем, какой член вынести за скобки, независимо от того, что происходит в числителе. 9+ }} \ln x = — \infty \hspace{0,5 дюйма}\hspace{0,25 дюйма}\mathop {\lim }\limits_{x \to \infty} \ln x = \infty \]

Обратите внимание, что нам пришлось сделать правый предел для первого, так как мы не можем подставить отрицательные \(x\) в логарифм. Это означает, что нормального предела не будет, поскольку мы должны смотреть на \(x\) с обеих сторон рассматриваемой точки, а \(x\) слева от нуля отрицательны.

Из предыдущего примера видно, что если аргумент журнала (то, что мы берем из журнала) стремится к нулю справа ( , т.е. всегда положительное), тогда журнал достигает отрицательной бесконечности в пределе, а если аргумент достигает бесконечности, то журнал также достигает бесконечности в пределе.

Обратите также внимание, что мы не можем смотреть на предел логарифма, когда \(x\) приближается к минус бесконечности, поскольку мы не можем подставлять отрицательные числа в логарифм. { — 1}}x\) Показать решение 9{ — 1}} \ влево ( {\ гидроразрыва {1} {x}} \ справа) = — \ гидроразрыва {\ пи {2} \]

Чтобы увидеть точное и математическое определение этого типа предела, см. раздел «Определение предела» в конце этой главы.

Объяснение урока: Число Эйлера (𝑒) как предел

В этом объяснении мы узнаем, как использовать определение 𝑒 (число Эйлера) для оценки некоторых специальные лимиты.

Число Эйлера (𝑒=2,71828…) очень полезно и возникает во многих различных областях науки. математика, включая расчет сложных процентов, задачи оптимизации, исчисление и определение функция, представляющая стандартное нормальное распределение вероятностей.

Число могло быть изначально найдено при поиске экспоненциальной функции, которая дифференцируется сама в себя. Однако мы также можем найти число Эйлера, используя пределы, и это то, что мы рассмотрим в этом объяснении.

Мы можем определить число Эйлера, используя следующий предел: 𝑒=1+1𝑥. lim→∞

Используя таблицу значений, мы можем увидеть, как этот предел приближается к числу Эйлера по мере увеличения 𝑥.

90 313 903 20 1+110000=2,71814…
𝑥 1+1 𝑥
1 1+11=2
10 1+110=2,59374…
100 1+1100=2,70481…
1‎ ‎000 1+11000=2,71692…
10‎ ‎000
100‎ ‎ 000 1+1100000=2,71826…

Мы можем использовать этот предел, чтобы помочь оценить пределы и решить проблемы, связанные с пределами этой формы. Прежде чем мы посмотрим на любой примеров, давайте рассмотрим другой предел, который также приводит к числу Эйлера.

Попробуем подставить 𝑥=1𝑦 в наш предыдущий предел. Поскольку мы рассматриваем предел как 𝑥→∞ и 𝑥=1𝑦, мы можем сказать, что при 𝑥→∞, 𝑦→0. Подставляя их в 𝑒=1+1𝑥lim→∞, мы получим 𝑒=(1+𝑦). lim→

Теперь мы знаем два предела, которые дают число Эйлера. Подведем итоги только что показанных результатов.

Определение: число Эйлера как предел оценить пределы, которые мы не могли раньше.

Пример 1: вычисление предела с использованием константы Эйлера

Определить lim→∞1+1𝑥.

Ответ

Мы могли бы сначала попытаться вычислить этот предел напрямую. В нашем пределе 𝑥 приближается к бесконечности, что означает, что знаменатель 1𝑥 неограниченно растет, а числитель остается постоянным, поэтому 1𝑥 приближается к 0. Это означает, что выражение в наших скобках приближается к 1. Однако наш показатель степени (4𝑥) равен приближается к бесконечности, когда 𝑥 приближается к бесконечности.

Итак, мы получаем lim→∞∞1+1𝑥=1, что является неопределенной формой. Это означает, что нам нужно будет попробовать какой-то другой метод для оценки нашего предела.

Заметим, что заданный нами предел очень похож на тот, который выражает значение числа Эйлера: 𝑒=1+1𝑥. lim→∞

нам дали показатель степени 4𝑥, а не 𝑥. Мы можем использовать законы экспонент, чтобы перевыразить это следующим образом:

Прежде чем мы сможем подставить число Эйлера в предельное выражение, нам нужно вывести показатель степени 4 за предел. При условии, что новый предел существует, мы можем использовать правило степени для пределов, чтобы добиться этого:

Предел внутри нашей экспоненты существует, потому что это просто наш предельный результат для числа Эйлера 𝑒. Итак, мы используем наш предельный результат и заменяем предел в скобках на 𝑒, что дает нам limlim→∞→∞1+1𝑥=1+1𝑥=𝑒.

Наш следующий пример показывает, как мы можем использовать наш другой предельный результат, чтобы помочь нам оценить предел.

Пример 2. Нахождение пределов путем их преобразования в формы пределов в натуральной степени

Определить lim→(𝑥+1).

Ответ

Поскольку нас просят оценить предел, мы могли бы начать с попытки сделать это напрямую. Когда 𝑥 приближается к 0, выражение в скобках приближается к 1, а величина нашего показателя степени неограниченно растет. Это неопределенная форма, а именно 1∞, поэтому нам нужно будет попробовать другой метод.

Мы видим, что наш предел аналогичен одному из наших предельных результатов, включающих число Эйлера, которое равно lim→(1+𝑥)=𝑒.

Итак, мы можем попробовать использовать этот результат, чтобы оценить наш предел.

Для этого мы хотим, чтобы наш показатель степени 1110𝑥 был таким же, как у предельного результата, который равен 1𝑥. Чтобы сделать это, мы начнем с использования наших законов показателей степени, чтобы переписать наш предел: где мы переставляем члены в скобках и используем тот факт, что 1110𝑥=1𝑥⋅1110.

На данный момент мы хотим использовать наш предельный результат, включающий число Эйлера; однако сначала нам нужно вывести нашу экспоненту за пределы нашего предела, а для этого нам нужно использовать правило степени для пределов.

Это говорит нам, что мы можем вывести экспоненту за пределы предела при условии, что наш новый предел существует.

В нашем случае имеем limlim→→(1+𝑥)=(1+𝑥), и мы знаем, что это верно потому что предел в наших скобках точно такой же, как предельный результат, включающий число Эйлера. Подставляя этот предел равным 𝑒, получаем (1+𝑥)=𝑒.lim→

Не всегда возможно напрямую использовать наши предельные результаты для числа Эйлера 𝑒. Возможно, нам придется использовать другие инструменты, такие как полиномиальное деление, факторинг или замену. Однако основная предпосылка та же самая: мы берем предел, который мы не можем оценить, и записываем его в форме для 𝑒, которую мы затем можем использовать для оценки наших предельных результатов.

Пример 3. Оценка предела путем преобразования его в форму предела в натуральной степени

Определить lim→∞1−7𝑥.

Ответ

Нас просят оценить предел, который мы могли бы попытаться оценить напрямую. Таким образом, когда 𝑥 приближается к ∞, выражение в скобках приближается к 1, а показатель степени неограниченно растет. Следовательно, имеем lim→∞∞1−7𝑥=1.

Это неопределенная форма, поэтому нам нужно будет попробовать другой метод для оценки этого предела.

Этот предел подобен одному из предельных результатов, связанных с числом Эйлера, поэтому мы можем попробовать использовать этот результат, чтобы помочь нам оценить наш предел. У нас есть много вариантов, как это сделать.

Мы попытаемся записать этот предел в форме, где мы можем использовать 𝑒=(1+𝑛).lim→

Однако также можно использовать 𝑒=1+1𝑛. lim→∞

Обычно один из предельных результатов оказывается проще, чем другой, и может быть очень сложно определить, какой предельный результат использовать, просто взглянув на вопрос, поэтому, если мы застряли, используя один результат, мы всегда можем попробовать использовать другой предел, который находится в форме с показателем степени 1𝑛.

Чтобы записать наш предел в этой форме, мы будем использовать замену. Мы хотели бы 1+𝑛 в наших скобках, поэтому мы используем замену 𝑛=−7𝑥.

Мы можем изменить эту замену, чтобы найти 𝑥 через 𝑛 так, чтобы 𝑥=−7𝑛.

Умножая это на 5, мы получаем 5𝑥=−35𝑛.

Используя эту замену, мы можем переписать наш предел как и мы хотим знать, что происходит с точки зрения 𝑛, поэтому нам нужно посмотреть на нашу замену. Поскольку 𝑥 приближается к бесконечности, −7𝑥 приближается к 0, а поскольку 𝑛=−7𝑥, мы также должны иметь то, что 𝑛 приближается к 0,9.0003

Это дает нам limlim→∞→(1+𝑛)=(1+𝑛).

Теперь воспользуемся одним из законов экспонент: limlim→ →(1+𝑛)=(1+𝑛).

Наконец, применим степенное правило для пределов: limlim→→ (1+𝑛)=(1+𝑛),, что, как мы знаем, мы можем сделать в этом случае, потому что полученный предел является нашим предельным результатом, включающим число Эйлера.

Все, что нам нужно сделать сейчас, это заменить этот предел на 𝑒 и переставить так, чтобы в итоге мы получили (1+𝑛)=𝑒=1𝑒.lim→

В нашем следующем примере мы рассмотрим предел рациональной функции, возведенной в линейный показатель.

Пример 4. Решение пределов путем преобразования их в формы пределов в натуральном показателе

Определить lim→∞𝑥+4𝑥−4.

Ответ

Мы могли бы попробовать вычислить этот предел напрямую. В наших скобках у нас есть рациональная функция, и мы знаем, что по мере того, как 𝑥 приближается к ∞, мы можем увидеть, что происходит, посмотрев на частное старших членов в нашей рациональной функции. Поскольку это равно 1, предел нашей рациональной функции также равен 1. Однако наш показатель степени неограниченно растет, поэтому мы имеем lim→∞∞𝑥+4𝑥−4=1, что является неопределенная форма. Поэтому нам нужно будет попробовать другой метод для оценки этого предела.

Вместо этого попробуем оценить это, используя предельный результат, включающий число Эйлера: →∞→∞𝑥+4𝑥−4=𝑥−4+8𝑥−4=1+8𝑥−4.

Если мы сравним два предела, мы увидим, что нам нужно использовать замену. Мы хотим, чтобы в скобках было 1+1𝑛, поэтому мы используем замену 1𝑛=8𝑥−4.

Когда 𝑥 приближается к бесконечности, мы видим, что 8𝑥−4 приближается к 0, поэтому 𝑛 также должно стремиться к бесконечности.

Прежде чем использовать эту замену, нам также нужно переставить 𝑥 через 𝑛, что мы можем сделать следующим образом.

Возьмем обратное значение обеих частей нашей подстановки, что даст нам 𝑛=𝑥−48.

Затем умножаем на 8 и прибавляем по 4 к обеим сторонам: 𝑥=8𝑛+4.

Теперь мы можем использовать эту замену, чтобы переписать наш предел: 1+1𝑛.

Мы хотим использовать наш предельный результат, но сначала нам нужно, чтобы показатель степени был равен 𝑛. Для этого нам сначала потребуется использовать законы экспонент в сочетании с правилом произведения для пределов, так что 1𝑛⋅1+1𝑛.

Чтобы использовать правило произведения для пределов, нам нужно, чтобы существовал предел обоих факторов. Мы покажем в нашей работе, что оба этих предела существуют.

Мы можем вычислить один из этих пределов напрямую: lim→∞1+1𝑛=1.

Затем, чтобы вычислить наш другой предел, мы используем законы экспонент и правило степени для пределов, истинно при условии, что этот предел существует, а мы знаем, что он существует, потому что это наш предыдущий предельный результат. Это означает, что мы можем заменить этот предел константой Эйлера 𝑒: 1+1𝑛=𝑒.lim→∞

Следовательно, мы показали lim→∞𝑥+4𝑥−4=𝑒.

Мы также можем использовать эти результаты для решения пределов, включающих более сложные функции.

Пример 5. Оценка пределов путем преобразования их в формы пределов в натуральной степени

Определить limtan→−4𝑥+1cot.

Ответ

Мы могли бы попробовать вычислить этот предел напрямую. Внутри скобок у нас непрерывная функция, поэтому мы можем просто подставить 𝑥=0. Однако наш показатель степени неограниченно растет, поэтому мы имеем limtanlimtan→→∞∞−4𝑥+1=−40+1=1,cot, что является неопределенной формой, поэтому мы нужно будет попробовать другой метод, чтобы оценить этот предел.

Вместо этого попробуем оценить это, используя предельный результат, включающий число Эйлера, то есть 𝑒=(1+𝑛).lim→

Чтобы сравнить это с пределом, нас просят find нам нужно будет преобразовать выражение в скобках в форму 1+𝑛. Для этого мы начнем с подстановки 𝑛=−4𝑥.tan

. Мы знаем, что когда 𝑥 приближается к 0, −4𝑥tan будет приближаться к 0 путем прямой подстановки, поэтому 𝑛 также должно стремиться к 0. Кроме того, взяв обратную величину обоих стороны нашей замены и перестановки, мы имеем −4𝑛=1𝑥=𝑥.tancot

Используя все это, мы можем переписать наш предел в виде законы показателей и правило степени для пределов для получения требуемого показателя степени 1𝑛: 𝑛).

Конечно, это при условии, что предел внутри наших скобок существует, что, как мы знаем, существует, поскольку lim→(1+𝑛)=𝑒.

Наконец, мы можем использовать наш предельный результат, чтобы вычислить предел в скобках как константу Эйлера:0003

Следовательно, мы смогли показать limtan→−4𝑥+1=1𝑒. cot

До сих пор мы исследовали пределы, которые приводят к числу Эйлера. Рассмотрим теперь некоторые ограничения, приводящие к обратная функция 𝑓(𝑥)=𝑒. Когда мы рассматриваем любую экспоненциальную функцию, 𝑝(𝑥)=𝑎, мы знаем, что его обратная функция является логарифмической функцией с основанием 𝑎, 𝑞(𝑥)=𝑥log. Поэтому, когда мы рассматриваем обратную функцию 𝑓(𝑥)=𝑒, мы знаем, что это будет логарифмическая функция с основанием 𝑒, 𝑔(𝑥)=𝑥log, натуральный логарифм. Натуральный логарифм функция может быть записана как 𝑔(𝑥)=(𝑥)ln. Вот график, показывающий экспоненциальную функцию и функция натурального логарифма. Мы можем видеть, как они являются отражением друг друга в строке 𝑦=𝑥.

Прежде чем мы определим натуральный логарифм как предел, напомним некоторые полезные свойства:

  1. 𝑦=𝑥ln эквивалентно 𝑒=𝑥,
  2. 𝑒=𝑥ln,
  3. ln 𝑒=1,
  4. ln1 =0,
  5. loglnln𝑥=𝑥𝑎.
    Для каждого 𝑥,𝑦∈ℝ и 𝑛∈ℝ,
  6. lnlnln𝑥𝑦=𝑥+𝑦,
  7. lnlnln𝑥𝑦=𝑥−𝑦,
  8. lnln𝑥=𝑛𝑥,
  9. lnlog𝑥×𝑒=1.

Мы начнем с рассмотрения уравнения

𝑦=𝑎−1, (1)

где 𝑥 и 𝑦 — переменные, а 𝑎 — вещественная константа. Для этого уравнение, мы можем видеть, что когда 𝑥→0, 𝑎→1 и, следовательно, 𝑦→0 тоже. Мы можем переместить 1 в другую часть уравнения и взять натуральный логарифм обеих частей этого уравнения, чтобы получить lnln𝑎=(𝑦+1).

Используя свойства логарифмов, мы можем преобразовать это в

𝑥𝑎=(𝑦+1)𝑥=(𝑦+1)𝑎.lnlnlnln (2)

Теперь рассмотрим следующий предел: lim→𝑎−1𝑥.

Используя (1) и (2), мы можем переписать это как limlimlnln→→𝑎−1𝑥=𝑦𝑎(𝑦+1).

Обратите внимание, что, как упоминалось ранее, когда 𝑥→0, 𝑦→0 также. Мы можем изменить правую часть этого уравнения, используя свойства логарифмов и пределов, следующим образом: limlimlnlnlimlnlnlimlnlnlnlimlnlnlnlim→→→→→→𝑎−1𝑥=𝑦𝑎(𝑦+1)=𝑎(𝑦+1)= 𝑎(𝑦+1)=𝑎(𝑦 +1)=𝑎(𝑦+1). 

Теперь можно заметить, что предел в знаменателе дроби — это предел, равный числу Эйлера. Следовательно, мы можем сказать, что limlnlnln→𝑎−1𝑥=𝑎𝑒=𝑎.

Мы можем использовать это предельное определение натурального логарифма для решения задач. Еще пара определений пределов которые также могут помочь нам решить проблемы limloglogandlimln→→(𝑥+1)𝑥=𝑒(𝑥+1)𝑥=1.

Ниже мы можем обобщить пределы, полученные в виде логарифмов.

Определение: логарифмы как ограничения

LimlnlimlogliMln →  →  → 𝑎 — 1𝑥 = 𝑎, (𝑥+1) 𝑥 = 𝑒, (𝑥+1) 𝑥 = 1.

Давайте теперь рассмотрим несколько примеров того, как мы можем использовать эти ограничения для решения задач.

Пример 6. Вычисление предела с помощью натурального логарифма

Определить lim→7−12𝑥.

Ответ

Нам дали ограничение на оценку, поэтому мы можем сначала попытаться сделать это напрямую. Если мы подставим 0 в наш предел, мы получим 00, который не определен. Следовательно, нам нужно будет использовать какой-то другой метод для определения этот предел.

Когда мы смотрим на этот предел, мы можем заметить, что он очень похож на предел, вычисляемый с помощью натурального логарифма. Позволять сравним его с этим пределом: limln→𝑎−1𝑥=𝑎.

Мы заметили, что это очень похоже на предел, который нас попросили оценить. Константа 𝑎 равна до 7. У нас также есть небольшие различия в показателе степени и знаменателе дроби. Мощность 3𝑥 вместо 𝑥, а в знаменателе 2𝑥 вместо 𝑥. Чтобы оценить предел, нам нужно попытаться манипулировать им, чтобы он был более похож на предел, который нам дали.

Начнем с изменения показателя степени. Мы можем сделать это, используя замену. Мы хотим сделать так, чтобы экспонента была просто переменная, а не переменная, умноженная на константу. Мы можем использовать замену 3𝑥=𝑢, что эквивалентно к 𝑥=𝑢3.

Перед тем, как мы сделаем эту замену, мы должны рассмотреть, что произойдет с пределом. В пределе, указанном в вопросе, мы рассматривают, когда 𝑥→0. Мы используем замену 𝑥=𝑢3, поэтому мы можем видеть что когда 𝑥 стремится к 0, то же самое будет и 𝑢. Следовательно, когда 𝑥→0, 𝑢→0 также. Теперь мы можем выполнить нашу замену следующим образом: limlim→→7−12𝑥=7−1.

Теперь мы можем видеть, что показатель степени равен просто 𝑢. Мы убрали коэффициент с помощью подстановки. Теперь это выглядит очень похоже на форму, которая нам нужна для ее оценки. Осталось разобраться с коэффициентом 𝑢 в знаменателе дроби. Мы можем начать с перемещения константы перед дробью: limlim→→7−1=327−1𝑢.

Теперь, используя предельные законы, мы можем переместить постоянный коэффициент перед пределом: limlim→→327−1𝑢=327−1𝑢.

Теперь мы можем видеть, что наш предел имеет форму limln→𝑎−1𝑥=𝑎, поэтому мы можем оценить чтобы получить наше решение: limlimln→→7−12𝑥=327−1𝑢=327.

Теперь мы рассмотрим последний пример в этом объяснении.

Пример 7. Вычисление предела с помощью натурального логарифма

Определить limln→(𝑥−1)𝑥−2.

Ответ

Мы могли бы попытаться решить этот вопрос с помощью прямой замены. Однако, если мы попытаемся подставить 𝑥=2 в пределе мы получим 00, что не определено. Нам нужно будет использовать другой метод в чтобы оценить этот предел. Заметим, что он похож на предел, который мы умеем вычислять, а именно limln→(𝑥+1)𝑥=1.

Мы заметили, что между этими двумя ограничениями есть несколько различий. Это значение, при котором лимит оценивается и с добавлением некоторых постоянных членов как в числителе, так и в знаменателе. Мы можем выполнить замену в нашем исходном пределе, чтобы попытаться заставить его выглядеть как предел, который мы знаем, как оценивать. мы заменим 𝑥−2=𝑢, что эквивалентно 𝑥=𝑢+2.

Прежде чем мы выполним эту замену, нам нужно рассмотреть, что произойдет с предельным значением 𝑥→2. Поскольку у нас есть 𝑢=𝑥−2, мы можем сказать, что при 𝑥→2, 𝑢→0. Мы теперь готов заменить: limlnlimln→→(𝑥−1)𝑥−2=(𝑢+1)𝑢.

После этой подстановки мы видим, что предел в точности имеет вид limln→(𝑥+1)𝑥=1. Следовательно, мы можем сказать, что limln→(𝑥−1)𝑥−2=1.

Давайте закончим повторением некоторых основных моментов этого объяснения.

Ключевые моменты

  • Мы нашли и доказали два полезных предельных результата, связанных с числом Эйлера: 𝑒=(1+𝑥)𝑒=1+1𝑛.limandlim→→∞
  • Мы нашли и доказали три полезных предельных результата, связанных с натуральным логарифмом: limlnlimlogloglimln→→→𝑎−1𝑥=𝑎,(𝑥+1)𝑥=𝑒,(𝑥+1)𝑥=1.
  • Мы можем использовать эти результаты для оценки пределов, которые дают неопределенные формы путем прямой замены или оценки.
  • Чтобы использовать эти результаты, иногда нам может понадобиться манипулировать нашим пределом, используя такие методы, как полиномиальное деление, замещение или факторинг.

Вычислите arccos 1 arccos: Вычислите: 1. arccos1 — arccos (-1/2) + arccos √3/2 2. arccos (sin п/6)3. tg (arccos…

Алгебра 10-11 класс. Обратные тригонометрические функции — math200.ru

Skip to content

Алгебра 10-11 класс. Обратные тригонометрические функцииadmin2022-10-14T14:31:36+03:00

Скачать файл в формате pdf.


Алгебра 10-11 класс. Обратные тригонометрические функции
Задача 1. Вычислите    \(\frac{{\arcsin \frac{1}{2}}}{{\arccos \frac{1}{2}}}\)

Ответ

ОТВЕТ: 0,5.

Задача 2. Вычислите    \(\frac{{\arccos \frac{{\sqrt 2 }}{2}}}{{\arcsin \frac{{\sqrt 3 }}{2}}}\)

Ответ

ОТВЕТ: 0,75.

Задача 3. Вычислите    \(\frac{{2\arccos \,0}}{{\arcsin \,1}}\)

Ответ

ОТВЕТ: 2.

Задача 4. Вычислите    \(\frac{{4\,\arcsin \,0}}{{3\arccos \frac{{\sqrt 2 }}{2}}}\)

Ответ

ОТВЕТ: 0.

Задача 5. Вычислите    \(\frac{{\arcsin \left( { — \frac{1}{2}} \right)}}{{\arccos \left( { — \frac{1}{2}} \right)}}\)

Ответ

ОТВЕТ: -0,25.

Задача 6. Вычислите    \(\frac{{\arccos \left( { — \frac{{\sqrt 2 }}{2}} \right)}}{{\arcsin \left( { — \frac{{\sqrt 3 }}{2}} \right)}}\)

Ответ

ОТВЕТ: -2,25.

Задача 7. Вычислите    \(\frac{{\arccos \,\left( { — 1} \right)}}{{4\arcsin \,\left( { — 1} \right)}}\)

Ответ

ОТВЕТ: -0,5.

Задача 8. Вычислите    \(\frac{{\arccos \,1}}{{3\arcsin 1}}\)

Ответ

ОТВЕТ: 0.

Задача 9. Вычислите    \(\frac{3}{\pi }\left( {\arcsin \left( { — \frac{{\sqrt 2 }}{2}} \right) + 2\arccos \left( { — \frac{{\sqrt 3 }}{2}} \right)} \right)\)

Ответ

ОТВЕТ: 4,25.

Задача 10. Вычислите    \(\frac{1}{\pi }\left( {3\arcsin \frac{1}{2} — 4\arccos \left( { — \frac{{\sqrt 2 }}{2}} \right)} \right)\)

Ответ

ОТВЕТ: -2,5.

Задача 11. Вычислите    \(\frac{5}{\pi }\left( {\arccos \,0 — 4\arcsin \left( { — \frac{{\sqrt 2 }}{2}} \right)} \right)\)

Ответ

ОТВЕТ: 7,5.

Задача 12. Вычислите    \(\frac{4}{\pi }\left( {4\arccos \left( { — 1} \right) — \arcsin \frac{{\sqrt 2 }}{2}} \right)\)

Ответ

ОТВЕТ: 15.

Задача 13. Вычислите    \(\frac{{{\text{arctg}}\,1}}{{{\text{arcctg}}\,\sqrt 3 }}\)

Ответ

ОТВЕТ: 1,5.

Задача 14. Вычислите    \(\frac{{{\text{arcctg}}\,\frac{{\sqrt 3 }}{3}}}{{{\text{arctg}}\,\sqrt 3 }}\)

Ответ

ОТВЕТ: 1.

Задача 15. Вычислите    \(\frac{{{\text{2}}\,{\text{arcctg}}\,0}}{{{\text{arctg}}\,\frac{{\sqrt 3 }}{3}}}\)

Ответ

ОТВЕТ: 6.

Задача 16. Вычислите    \(\frac{{6\,{\text{arctg}}\,0}}{{{\text{arcctg}}\,1}}\)

Ответ

ОТВЕТ: 0.

Задача 17. Вычислите    \(\frac{{{\text{9}}\,{\text{arctg}}\,\left( { — \sqrt 3 } \right)}}{{{\text{arcctg}}\,\left( { — 1} \right)}}\)

Ответ

ОТВЕТ: -4.

Задача 18. Вычислите    \(\frac{{6\,{\text{arcctg}}\,\left( { — \sqrt 3 } \right)}}{{{\text{arctg}}\,\left( { — 1} \right)}}\)

Ответ

ОТВЕТ: -20.

Задача 19. Вычислите    \(\frac{{\arccos \left( { — \frac{{\sqrt 2 }}{2}} \right)}}{{{\text{arctg}}\,\left( { — 1} \right)}}\)

Ответ

ОТВЕТ: -3.

Задача 20. Вычислите    \(\frac{{{\text{arcctg}}\,\left( { — \sqrt 3 } \right)}}{{{\text{arcsin}}\left( { — \frac{1}{2}} \right)}}\)

Ответ

ОТВЕТ: -5.

Задача 21. Вычислите    \(\frac{6}{\pi }\left( {{\text{arctg}}\,\sqrt 3  + 3\,{\text{arcctg}}\sqrt 3 } \right)\)

Ответ

ОТВЕТ: 5.

Задача 22. Вычислите    \(\frac{2}{\pi }\left( {{\text{4}}\,{\text{arcctg}}\,\left( { — 1} \right) — 6\,{\text{arctg}}\left( { — \sqrt 3 } \right)} \right)\)

Ответ

ОТВЕТ: 10.

Задача 23. Вычислите    \(\frac{3}{\pi }\left( {{\text{arctg}}\,1 — \arcsin \frac{{\sqrt 2 }}{2}} \right)\)

Ответ

ОТВЕТ: 0.

Задача 24. Вычислите    \(\frac{3}{\pi }\left( {{\text{arcctg}}\,\left( { — \frac{{\sqrt 3 }}{3}} \right) — \arccos \left( { — \frac{{\sqrt 2 }}{2}} \right)} \right)\)

Ответ

ОТВЕТ: -0,25.

Задача 25. Вычислите    \(\frac{1}{\pi }\left( {{\text{arcsin}}\frac{1}{3} + {\text{arccos}}\frac{1}{3}} \right)\)

Ответ

ОТВЕТ: 0,5.

Задача 26. Вычислите    \( — \frac{3}{\pi }\left( {{\text{arctg}}\,{\text{2}}\, + {\text{arcctg}}\,{\text{2}}} \right)\)

Ответ

ОТВЕТ: -1,5.

Задача 27. Вычислите    \(\frac{2}{\pi }\left( {{\text{arcsin}}\left( { — \frac{1}{4}} \right) + {\text{arccos}}\left( { — \frac{1}{4}} \right)} \right)\)

Ответ

ОТВЕТ: 1.

Задача 28. Вычислите    \(\frac{8}{\pi }\left( {{\text{arctg}}\,\left( { — 3} \right)\, + {\text{arcctg}}\,\left( { — 3} \right)} \right)\)

Ответ

ОТВЕТ: 4.

Задача 29. Вычислите    \(\sin \left( {\frac{1}{6}{\text{arccos}}\frac{1}{3} + \frac{1}{6}{\text{arccos}}\left( { — \frac{1}{3}} \right)} \right)\)

Ответ

ОТВЕТ: 0,5.

Задача 30. Вычислите    \(\cos \left( {\frac{1}{3}{\text{arcctg}}\frac{1}{2} + \frac{1}{3}{\text{arcctg}}\left( { — \frac{1}{2}} \right)} \right)\)

Ответ

ОТВЕТ: 0,5.

Задача 31. Вычислите    \(8\sin \left( {\arcsin \frac{1}{4}} \right) + 12\cos \left( {\arccos \frac{1}{3}} \right)\)

Ответ

ОТВЕТ: 6.

Задача 32. Вычислите    \(\frac{1}{6}{\text{tg}}\left( {{\text{arctg}}\,3} \right) — \frac{1}{2}{\text{ctg}}\left( {{\text{arcctg}}\,5} \right)\)

Ответ

ОТВЕТ: -2.

Задача 33. Вычислите    \(2\sin \left( {\arcsin \left( { — \frac{1}{5}} \right)} \right) — 14\cos \left( {\arccos \left( { — \frac{1}{7}} \right)} \right)\)

Ответ

ОТВЕТ: 1,6.

Задача 34. Вычислите    \({\text{3}}\,{\text{tg}}\left( {{\text{arctg}}\,\left( { — \frac{1}{3}} \right)} \right)\,\, + \,{\text{4}}\,{\text{ctg}}\left( {{\text{arcctg}}\,\left( { — \frac{1}{4}} \right)} \right)\)

Ответ

ОТВЕТ: -2.

Задача 35. Вычислите    \(\sin \left( {\frac{\pi }{2} + \arccos \frac{1}{4}} \right)\)

Ответ

ОТВЕТ: 0,25.

Задача 36. Вычислите    \(3\sin \left( {\pi  + \arcsin \frac{1}{5}} \right)\)

Ответ

ОТВЕТ: -0,6.

Задача 37. Вычислите    \(3\cos \left( {\frac{{3\pi }}{2} — \arcsin \left( { — \frac{1}{6}} \right)} \right)\)

Ответ

ОТВЕТ: 0,5.

Задача 38. Вычислите    \(2\cos \left( {3\pi  — \arccos \left( { — \frac{1}{8}} \right)} \right)\)

Ответ

ОТВЕТ: 0,25.

Задача 39. Вычислите    \(\sin \left( {\arccos \frac{3}{5}} \right)\)

Ответ

ОТВЕТ: 0,8.

Задача 40. Вычислите    \(13\sin \left( {\arccos \left( { — \frac{{12}}{{13}}} \right)} \right)\)

Ответ

ОТВЕТ: 5.

Задача 41. Вычислите    \(\cos \left( {\arcsin \,0,8} \right)\)

Ответ

ОТВЕТ: 0,6.

Задача 42. Вычислите    \(\cos \left( {\arcsin \left( { — \frac{{24}}{{25}}} \right)} \right)\)

Ответ

ОТВЕТ: 0,28.

Задача 43. Вычислите    \(\sin \left( {{\text{arctg}}\frac{3}{4}} \right)\)

Ответ

ОТВЕТ: 0,6.

Задача 44. Вычислите    \(\sin \left( {{\text{arcctg}}\left( { — \sqrt {15} } \right)} \right)\)

Ответ

ОТВЕТ: 0,25.

Задача 45. Вычислите    \(\cos \left( {{\text{arctg}}\left( { — \frac{3}{4}} \right)} \right)\)

Ответ

ОТВЕТ: 0,8.

Задача 46. Вычислите    \(\frac{1}{{\sqrt 5 }}\cos \left( {{\text{arcctg}}\,2} \right)\)

Ответ

ОТВЕТ: 0,4.

Задача 47. Вычислите    \({\text{tg}}\left( {{\text{arcctg}}\frac{1}{4}} \right)\)

Ответ

ОТВЕТ: 4.

Задача 48. Вычислите    \({\text{ctg}}\left( {{\text{arctg}}\left( { — \frac{1}{3}} \right)} \right)\)

Ответ

ОТВЕТ: -3.

Задача 49. Вычислите    \({\text{tg}}\left( {\arcsin 0,6} \right)\)

Ответ

ОТВЕТ: 0,75.

Задача 50. Вычислите    \(12\,{\text{tg}}\left( {\arccos \frac{{12}}{{13}}} \right)\)

Ответ

ОТВЕТ: 5.

Задача 51. Вычислите    \(\sqrt 2 \,{\text{ctg}}\left( {\arcsin \frac{1}{3}} \right)\)

Ответ

ОТВЕТ: 4.

Задача 52. Вычислите    \(\sqrt {15} \,{\text{ctg}}\left( {\arccos \left( { — \frac{1}{4}} \right)} \right)\)

Ответ

ОТВЕТ: -1.

Задача 53. Вычислите    \(\arccos \left( {\sin \left( { — 2} \right)} \right)\)

Ответ

ОТВЕТ: \(\frac{{3\pi }}{2} — 2\).

Задача 54. Вычислите    \(\arcsin \left( {\sin \frac{{17\pi }}{5}} \right)\)

Ответ

ОТВЕТ: \( — \frac{{2\pi }}{5}\).

Задача 55. Вычислите    \(\arccos \left( {\sin \left( { — \frac{{3\pi }}{7}} \right)} \right)\)

Ответ

ОТВЕТ: \(\frac{{13\pi }}{{14}}\).

Задача 56. Вычислите    \(\arcsin \left( {\sin \frac{{22\pi }}{7}} \right)\)

Ответ

ОТВЕТ: \( — \frac{\pi }{7}\).

Задача 57. Вычислите    \(\arccos \left( {\cos \frac{{29\pi }}{5}} \right)\)

Ответ

ОТВЕТ: \(\frac{\pi }{5}\).

Задача 58. Вычислите    \(\arccos \left( {\sin \left( { — \frac{{22\pi }}{5}} \right)} \right)\)

Ответ

ОТВЕТ: \(\frac{{9\pi }}{{10}}\).

Задача 59. Вычислите    \({\text{arctg}}\left( {{\text{ctg}}\,3} \right)\)

Ответ

ОТВЕТ: \(\frac{\pi }{2} — 3\).

Задача 60. Вычислите    \({\text{arcctg}}\left( {{\text{tg}}\,10} \right)\)

Ответ

ОТВЕТ: \(\frac{{7\pi }}{2} — 10\).

Реклама

Поддержать нас

Определение арксинуса, арккосинуса числа а

Похожие презентации:

Синус суммы и разности двух углов

1. Работам устно

Существует ли угол, для
которого
sin 3
sin
2
2
cos
2
5
cos
4
Вычислите
sin( )
6
cos( )
4
sin
cos
3
2

2. Определение арксинуса, арккосинуса числа а

Арксинус числа а , |а | ≤ 1 ([–1; 1]), есть такое число
α из промежутка [– π / 2; π / 2 ], синус которого
Sin
равен числу а
1 π/2
sin (arcsin a) = a
а
arcsin (– a) = – arcsin a
arcsin a
α
–α
x
–a
-π/2 -1
arcsin (– a)
Sin
Вычислите:
1
arcsin
2 6
2
12
2
2
arcsin
4
2
3
arcsin
2
3
arcsin ( 1)
π/2
2
1
arcsin
2
6
-π/2
1
23
12
Ищу число из отрезка
[-π/2; π/2], синус
которого равен …
y
2
3
arcsin a , sin a
a
1;1
,
;
4
2 2
6
0
x
2
3
4
6
arcsin 0
1
arcsin
2
2
arcsin
2
3
arcsin
2
arcsin 1
arcsin 1,5
Не существует
arcsin
3
Не существует
2
arcsin(
)
2
3
arcsin(
)
2
arcsin( 1)
Арккосинус числа а , |а | ≤ 1 ([–1; 1]), есть такое
число α из промежутка [ 0; π ], косинус которого
Sin
равен а
cos (arccos a) = a
arccos a
arcсоs (– a)
π -1
–a
arccos (– a) = π – arccos a
α
а
0 Cos
1
Вычислите:
3
arcсos
2 6
arcсos( 1)
arcсos 0
π
1
21
22
0
3
2
0 Cos
2
2 3
arcсos
4
2
1 2
arcсos
2 3
Ищу число из отрезка
[0; π], косинус
которого равен…. .

8. Имеет ли смысл выражение?

аrcsin (-1/2)
да
аrcsin 1,5
нет
arcsin 3 20
нет
arccos 5
нет
arccos 3 1
да
arccos
5
да
π
1
= 6
arcsin
2
3
π
arcsin
=
2
3
π
1
arcsin ( — 2 ) = — ОТВЕТЫ
6
π
arcsin 1 =
2
π
2
arcsin (
)= — 4
2
arccos
1 = π
2
3
π
3
arccos
2 = 6
1 2π
arccos (− 1 ) = π ̶ ОТВЕТЫ
arccos =
2
2
3
2 ) 3π
(
arccos
=
2
4
arccos 0 =
π
2

11. Заполни таблицу

Арктангенс числа а есть число (угол) α из интервала
(-π/2;π/2), тангенс которого равен а
tg (arctg a) = a
1

у
π/2
а
arctg a
α
–α
0
arctg (- a)

-1 — π/2
arctg (– a) = – arctg a
х

Арккотангенс числа а есть число (угол)
α из интервала (0; π),
котангенс которого равен а
ctg (arcctg a) = a

а
1 у
arcctg (- a)
π

arcctg a
α
0
○х
0
-1
arcctg (– a) = π – arcctg a
arсtg
arcsin
3
2
arccos 1
2
1
3
arсctg
1=
arсtg
3=
+ arccos
3
2
+ arcsin
=
=
П
6
П
ОТВЕТЫ
4
П
3
П
П
П
+ 6 =
2
3
ОТВЕТЫП
1
= П + П =
2
6
2
3
14

15.

Домашнее задание1.Сделать конспект по параграфу 21(п.1- 4) +
презентация
2.Заполнить в тетради таблицу слайд 12
3.Номера 21.1, 21.2, 21.4, 21.13,
21.14, 21.31, 21.32 (выполнить в учебнике).
4.В тетради № 21.16,21.17,21.18,21.33,21.34

English     Русский Правила

Ряд для арккосуса (арккосинуса) около 1

Предположим, вам нужно оценить арккосинус аргумента около 1. Для этого есть ряд:

Этот ряд можно найти, например, здесь.

Это пригодится, например, при работе с аналогом теоремы Пифагора о сфере.

Вы можете просто использовать сериал и идти своим путем. Но происходит гораздо больше, чем кажется на первый взгляд.

Почему это невозможно

Мы рассматриваем ряд для арккосинуса с центром в 1, и все же арккосинус является многозначным в окрестности 1: для аргумента чуть меньше 1 есть два возможных угла, которые имеют такой косинус. Это важно, потому что для того, чтобы иметь степенной ряд в точке, функция должна хорошо вести себя на диске вокруг этой точки на комплексной плоскости. Мало того, что наша функция плохо себя ведет, она даже плохо определена , пока мы не рассмотрим разрезы ветвей. Кроме того, функция arccos( z ) не дифференцируем в 1; производная имеет сингулярность в точке 1.

Короче говоря, мы пытаемся разложить функцию в ряд в точке, где функция ведет себя плохо. Это звучит невозможно или, по крайней мере, опрометчиво.

Почему это возможно

Приведенная выше серия не является серией для arccos как таковой. Если мы разделим обе части ряда на √(2-2 z ), мы увидим, что на самом деле у нас есть ряд для

Хотя arccos ведет себя плохо при 1, так же как и √(2-2 z ), и их отношение хорошо себя ведет при 1. Фактически, это аналитическая функция, и поэтому она имеет степенной ряд.

Арккосинус имеет расширение ряда , но не имеет ряда степеней . Ряд вверху не является степенным рядом, потому что он состоит не только из степеней z . Справа также есть функция извлечения квадратного корня, и эта функция имеет решающее значение для того, чтобы все работало.

Мотивация

Почему кому-то придет в голову найти ряд таким образом, то есть зачем делить на √(2-2 z )?

Для небольших значений z ,

и т. д.

Можно было бы надеяться, и это правильно получается, что путем деления arccos( z ) на хорошее приближение можно получить функцию, достаточно хорошую для расширения в силовом ряду.

Почему это возможно

Давайте построим график

, чтобы увидеть, выглядит ли это разумной функцией. Обычно в сложных переменных нет золотой середины: функции либо аналитические, либо плохо себя ведут. И в числителе, и в знаменателе есть разрезы, но мы надеемся, что разрезы совпадают, и отношение можно плавно распространить на разрезы.

График ниже показывает, что это так.

Этот график был создан с помощью

 f[z_] := ArcCos[z] / Sqrt[2 - 2 z]
    ComplexPlot3D[f[z], {z, 0 - I, 2 + I}]
 

Белая полоса поперек графика не случайный артефакт построения, а иллюстрирует нечто важное.

Невозможно расширить arccos( z ) до функции, аналитической для всех z . Вы должны исключить некоторые значения z от домена, т.е. нужно сделать отсечения ветвей, а Mathematica делает эти разрезы вдоль действительной оси для z ≤ -1 и z ≥ 1.

Функция квадратного корня также требует отсечения ветвей, и Mathematica выбирает, что ветвь разреза проходит вдоль отрицательной действительной оси, что означает, что мы должны исключить z ≥ 1. Таким образом, разрезы ветвей нашего числителя и знаменателя совпадают. (Инверсный косинус имеет дополнительное сечение ветви, но оно не близко к 1, поэтому для наших целей это не имеет значения.)

Таким образом, ряд в верхней части сообщения расширяет arccos в точке, где функция плохо себя ведет, путем деления ее на другую функцию, которая плохо определена таким же образом, создавая функцию, которая ведет себя хорошо.

Связанные посты

  • Обрезки ветвей
  • Конструктив Пикард
  • Начальная загрузка минимальной математической библиотеки
  • Ламберт В. снова наносит удар

Обратные тригонометрические производные, Как доказать обратные тригонометрические производные?

Обратные тригонометрические производные Производные обратных тригонометрических функций называются обратными тригонометрическими производными, которые важны в исчислении и используются для решения многих задач в физике, технике и других областях. Понимание этих производных является неотъемлемой частью освоения исчисления, и они часто встречаются как в теоретических, так и в прикладных задачах. Если вы ищете производные обратного триггера, прочитайте содержание ниже.

Источник изображения: Fresherslive 92)

, где x — входное значение функции, а диапазон функции арккотангенса ограничен (0, π).

Важно отметить, что при нахождении производной обратных тригонометрических функций нужно быть осторожным с областью определения и диапазоном функций. В частности, нам нужно убедиться, что входное значение обратной тригонометрической функции находится в пределах ее области определения, а выходное значение — в пределах ее диапазона.

Таким образом, производные обратных тригонометрических функций можно найти, используя основные правила дифференцирования и обратные тригонометрические тождества. Важно помнить об области определения и диапазоне функций при нахождении их производных, а логарифмические тождества можно использовать для выражения обратных тригонометрических функций через логарифмические функции. 92)

Таким образом, производные обратных тригонометрических функций могут быть доказаны с помощью фундаментальной теоремы исчисления и тригонометрических тождеств. Как запомнить обратные тригонометрические производные?

Обратные тригонометрические функции — это обратные функции тригонометрических функций, таких как синус, косинус, тангенс и т. д. Производные этих обратных тригонометрических функций бывает трудно запомнить, но есть несколько приемов, которые помогут вам их запомнить.

    92). Понимание этих шаблонов может помочь вам легче запомнить производные.
  1. Практика: Практика — это ключ к запоминанию производных. Решайте практические задачи и тесты, чтобы помочь укрепить формулы и шаблоны в уме. Чем больше вы практикуетесь, тем легче вам будет запоминать производные.
  2. Мнемотехника: Мнемоника — это вспомогательное средство для запоминания, которое может помочь вам запомнить производные. Например, вы можете использовать аббревиатуру «SOHCAHTOA», чтобы запомнить тригонометрические отношения: синус = противолежащее/гипотенуза, косинус = прилежащее/гипотенуза, тангенс = противоположное/прилежащее. Точно так же вы можете использовать мнемонику для запоминания производных обратных тригонометрических функций, таких как «IDA» (обратная, производная, дуга).
  3. Визуализируйте: Визуализация производных может помочь вам запомнить их. Нарисуйте графики обратных тригонометрических функций и их производных. Увидев взаимосвязь между функциями и их производными, вы сможете легче запомнить формулы.

Что такое 6 триггерных производных?

Шесть тригонометрических функций: синус, косинус, тангенс, котангенс, секанс и косеканс. Производные этих функций важны в исчислении и имеют множество приложений в физике, технике и других областях.

Производные шести тригонометрических функций следующие:

  1. Производная синуса:

(d/dx) sin(x) = cos(x)

Это означает, что скорость изменения функции синуса в любой точке равна косинусу этой точки. Производная синуса используется в таких приложениях, как колебания и волны.

  1. Производная косинуса:

(d/dx) cos(x) = -sin(x)

Это означает, что скорость изменения функции косинуса в любой точке равна отрицательному синусу этой точки. Производная косинуса используется в таких приложениях, как колебания и волны, а также при вычислении интегралов. 92(x)

Это означает, что скорость изменения функции котангенса в любой точке равна отрицательному квадрату косеканса этой точки. Производная котангенса используется в таких приложениях, как тригонометрические тождества и интегрирование.

  1. Производная секанса:

(d/dx) sec(x) = sec(x)tan(x)

Это означает, что скорость изменения функции секущей в любой точке равна произведению секанса и тангенса в этой точке. (-1)(x) или arcsec(x). 92 – 1))

Это можно использовать для нахождения производной любой функции, включающей функцию арксеканса, используя цепное правило.

Калькулятор обратных тригонометрических производных

Существует несколько онлайн-калькуляторов, которые могут вычислять обратные тригонометрические производные. Вот несколько вариантов:

  1. Symbolab: На этом веб-сайте есть калькулятор производной обратных тригонометрических функций, включая arcsin, arccos и arctan. Он также предоставляет пошаговые решения и график функции.
  2. Mathway: Mathway также имеет калькулятор для обратных тригонометрических производных. Это позволяет пользователям выбирать функцию, которую они хотят различать, например, arcsec или arccot, и предоставляет ответ с подробными шагами.
  3. Desmos: этот графический онлайн-калькулятор имеет встроенную функцию дифференцирования, которая может вычислять производную обратных тригонометрических функций. Пользователи могут ввести функцию, которую они хотят дифференцировать, и просмотреть график и производную рядом друг с другом. 2) 92)

    Это лишь несколько примеров того, как найти обратные тригонометрические производные с помощью формул и цепного правила.

    Обратные тригонометрические производные – часто задаваемые вопросы

    1. Что такое обратная тригонометрическая функция?

    Обратная тригонометрическая функция — это функция, которая дает угол, синус, косинус или тангенс которого равен заданному значению.

    2. Какие обратные тригонометрические функции являются наиболее распространенными?

    Наиболее распространенными обратными тригонометрическими функциями являются arcsin(x), arccos(x) и arctan(x). 92).

    6. Что такое цепное правило и как оно используется для оценки обратных триггерных производных?

    Цепное правило — это метод исчисления, используемый для оценки производной сложной функции. Он используется для оценки производных обратных триггеров, сначала находя производную внешней функции, а затем находя производную внутренней функции.

    7. Что такое правило произведения и как оно используется для оценки обратных триггерных производных?

    Правило произведения — это метод исчисления, используемый для оценки производной произведения двух функций. Он используется для оценки обратных триггерных производных путем применения правила произведения к производной функции.

    8. Что такое правило отношения и как оно используется для оценки обратных триггерных производных?

    Факторное правило — это метод исчисления, используемый для вычисления производной частного двух функций. Он используется для оценки обратных триггерных производных путем применения правила отношения к производной функции.

    9. Что такое интегрирование подстановкой и как оно используется для оценки обратных триггерных функций?

    Интегрирование подстановкой — это метод исчисления, используемый для вычисления интеграла функции путем подстановки.

Построить график онлайн 3д: Построить трехмерный график онлайн

Планируйте свою гардеробную с помощью онлайн конструктора

Проектирование систем хранения Larvij

с помощью 3D планировщика

Сам себе дизайнер. 3D план гардеробной — это просто.
Визуализация будущей системы хранения с перечнем элементов и стоимостью комплекта. Возможность сохранить проект и оформить заказ.

Работает онлайн непосредственно в браузерах как на стационарных компьютерах и ноутбуках, так и на мобильных. Не требует установки дополнительных приложений.

Перейти в Конструктор

Планировщик гардеробных Ларвидж

это удобный инструмент, с помощью которого вы сможете легко и быстро спроектировать подходящую именно для вас гардеробную систему

Добро пожаловать в мир ваших фантазий по организации хранения

Воспользовавшись нашим 3D конструктором гардеробных, вы сможете самостоятельно продумать дизайн гардеробной, создать комфортное для жизни пространство в своей квартире или доме. Воспользовавшись нашим онлайн сервисом по проектированию гардеробных вы с легкостью можете выбрать необходимые элементы вашей стеллажной системы или воспользоваться уже специально подобранными готовыми комплектами.

Видео инструкция по работе с 3D планировщиком Larvij

Как работать с планировщиком?

Размер помещения

И так, в начале работы предлагается выбрать форму помещения: одиночная стена или угловая конфигурация комнаты. Здесь же можно открыть и ранее сохраненный проект. Вам будет предложено указать размер стены. Это можно сделать несколькими способами — передвигая ползунок мышкой или ввести точный размер вручную.

Выбор элементов

После этого в главной части планировщика Ларвидж проектируем гардеробную, выбирая необходимые элементы, находящиеся в нижней части окна. Выбрав полку, к примеру, перемещаете ее мышкой на свободное место на стене и все необходимые элементы для ее установки и монтажа будут автоматически подобраны программой. Все элементы легко перемещаются или заменяются на другие, рабочую область возможно уменьшать или увеличивать в масштабе с помощью колесика мыши. Две смежные секции можно объединить, соединив их смежными стойками.

Готовые решения

Кроме того, возможно воспользоваться вариантами готовых комплектов для гардеробных и визуально оценить их функциональность. Для удобства и наглядного представления каждый вариант готовых комплектов имеет фотографию стеллажной системы в интерьере. Закончив проектирование, вы увидите вашу спецификацию гардеробной со списком элементов и ценами, которую сможете сохранить или распечатать для заказа гардеробной. Вот так легко проектируются и создаются гардеробные системы Ларвидж.

Школа компьютерной графики и дизайна «Art and Shock»

Школа компьютерной графики и дизайна «Art and Shock»

Санкт-Петербург

Пн — Вс 11. 00-19.00

+7 (812) 926-06-16Заказать звонок

Обучим дизайну и 3D моделированию

с поддержкой в трудоустройстве по
авторской программе школы Евгении Туминой

Подробнее

В программе будут
кейсы от дизайн-студий.

Мы активно отвечаем на все вопросы в нашей группе Вконтакте.

Классы оборудованы компьютерами с необходимым ПО.

Преподаватели всегда тщательно проверяют задания.

Вы получите
все обучающие материалы

В конце курса у вас будет достойный проект в портфолио.

Ваши навыки мы подтверждаем сертификатом об окончании курса.

Лучшим ученикам выпадает возможность пройти стажировку в дизайн-студиях.

#Дизайн интерьера#3Ds MAX

Все

Онлайн

Оффлайн

#Оффлайн

Курс направлен на изучение программ 3Ds MAX, Corona Render и Photoshop. За 2 месяца вы с нуля научитесь создавать фотореалистичные интерьеры.

Подробнее о курсе

#Онлайн

Online Курс направлен на подготовку востребованных специалистов, знания которых помогают реализовать самые смелые творческие идеи интерьеров в 3D, занятия проходят в любой удобный для вас день.

Подробнее о курсе

#Онлайн

Online курс направлен на создание фотореалистичных визуализаций архитектурных сооружений и ландшафта на профессиональном уровне. Занятия проходят в любой удобный день, из любой точки мира.

Подробнее о курсе

Смотреть
все курсы

Сформируем цели

Подберем курс

Продемонстрируем процесc

На занятиях берутся примеры из опыта преподавателей-практиков. Все обучение построено на основе реальных кейсов и ситуаций. А результатом курса является не просто сертификат, а портфолио, с которым вы можете устраиваться на работу или уходить во фриланс.

Помимо обучения они ведут реальные проекты, поэтому знают, что происходит в индустрии и является трендом. Многие из преподавателей имеют опыт работы с международными проектами. Также эксперты делятся секретами поиска заказов, работы с подрядчиками.

В школе компьютерной графики мы делаем упор на создание сообщества, в котором постоянно проходят мастер-классы, совместные выезды. Ведь так важно, чтобы после обучения вы находились в кругу единомышленников. Для этого у нас есть специальные чаты и биржа выпускников, на которой каждый может разместить свое портфолио и найти заказчика.

Основатель школы и разработчик программы обучения. Визуализатор, дизайнер и основатель студии Art and Shock. Куратор учебных программ по визуализации и дизайну.

Дизайнер, визуализатор и преподаватель 3Ds MAX/Corona renderer/Adobe Photoshop.

Дизайнер, визуализатор и преподаватель 3Ds MAX/Corona renderer/Adobe Photoshop.

Web-дизайнер, эксперт UX/UI, преподаватель курса web-дизайн.

Подробнее о преподавателях

Смотреть видеоотзыв

Отличная школа! Дружеская атмосфера, готовность отвечать на все вопросы, индивидуальный подход и отличный результат в виде работы в портфолио) Всем советую, не ошибетесь!) Я пошла на курсы после университета, чтобы сделать хорошее портфолио. Так и получилось) После курса попробовала податься в дизайн студию и меня взяли! Сейчас занимаюсь как дизайном, так и визуализацией.

3Dравствуйте!
Как написала Анастасия Калашникова, все началось с бесплатного марафона. Марафон увлек и привел на курс. В целом, курс очень понравился. Структурировано, понятно, доступно, интересно. Конечно, можно бесконечно добавлять новый материал, но тут есть опасность перегруза большим количеством информации, и потерей интереса к занятиям. Всё равно, всегда останутся дополнительные вопросы, ответы на которые можно найти в интернете или задать в чате группы.
Что касается пожеланий, то я писал как-то в чате о том, что Slate Material Editor нагляднее для восприятия (для меня точно), и если в видеоуроках использовать его, то это должно помочь восприятию и без того сложной темы материалов. И может добавить уроки по Unreal Engine 4 – сейчас, как мне кажется, этот движок набирает популярность.
Особенно хочется отметить оперативность поддержки и объем дополнительной информации в ответах на вопросы учеников в чате — это очень помогает. И это круто! Лично мне это очень понравилось в вашей школе! Как вы все успеваете, учитывая, что мы не единственны курс?! И разбор работ — тоже очень крутая тема — мотивирует!
Спасибо всей команде школы и лично Евгении за работу! И за интерес к 3D!
Вы молодцы! Удачи! Успехов! Очень рад знакомству с ARTandSHOCK!

Читать полностью

Спасибо школе «Art and Shock» за пройденный курс по освоению 3DsMax!
Перед тем, как выбрать школу, было просмотрено множество вариантов, но они не внушали доверия и казалось, что дают очень поверхностные знания.
И вот благодаря марафону, который длился всего 3 дня, я узнала про эту прекрасную школу!
⠀Очень рада, что выбрала именно школу «Art and Shock», потому что если у вас есть мечта, желание учиться и познавать что-то новое, здесь созданы все условия, получить эти знания и начать осуществлять свою мечту! Спасибо за время проведённое с пользой и знания, которые я приобрела после обучения!
⠀Результат обучения в школе меня очень впечатлил! С программой до этого я была не знакома и вот, что у меня получилось

Читать полностью

Увидев рекламу бесплатного онлайн краса, решил проверить свои силы в 3d визуализации; Вспомнить старый добрый 3Ds MAX. С 3Ds MAX знаком еще с 2004 года, но серьезно заняться изучением всё как-то стимула не хватало. Ознакомившись с бесплатным курсом, мне понравилась подача материала и финальные работы конкурсантов, они выглядят прекрасно, особенно для начинающего 3D специалиста. Оперативные и четкие ответы помогли без труда записаться на курс и пройти его с хорошим результатом на выходе, в итоге первая и красивая картинка в своей работе в 3D.
Вдохновившись результатом посмотрел несколько вебинаров где подробно рассказали о профессии и полноценном курсе. Курс рассчитан на 2,5 месяца, за небольшую плату получаешь большую и полезную для работы коллекцию 3D моделей, продуманный и проработанный курс с видео инструкциями и оформленный по урокам.
Каждый урок на сайте для ученика описан с переведенным материалом нужных настроек программы, и того какого результата нужно достичь до завершения урока с приведенным видео разбором по выполнению данного урока. Подача материала подходит новичкам которые не работали в данной программе, а так же и тем кто с программой знаком, есть много полезного. Проверка заданий проходит достаточно быстро, есть хорошая обратная связь как на сайте ученика так и в группе в VK где помогают кураторы, учителя и ученики.
По завершению обучения можно смоделировать 3d модели на уровне достаточном чтобы смоделировать предмет интерьера без поиска его в интернете. Смоделировать модель по фотографии, построить интерьер визуализацию по фотографии.
Много полезного, всего не описать 🙂 Остается нарабатывать скорость моделирования, и опыт.
Спасибо кураторам и учителям, за помощь и вдохновение! Без Вас так возможно и не занялся 3D моделированием на уровне достаточном для принятия заказов и работы.

Читать полностью

Все статьи блога Тelegram каналы для 3d дизайнера Читать подробнее

Все новости

Читать подробнее

Даю согласие на обработку персональных данных

3D-графика — Понятные руководства — Wiki

Ранее мы описали основы программирования R и предоставили краткие руководства по импорту данных в R , а также по визуализации данных с использованием базовых графиков R.

В этой главе описывается создание статических и интерактивных трехмерных ( 3D ) графиков . Мы также предоставляем пакет R под названием graph4d для простого создания и пошаговой настройки трехмерных графиков в программном обеспечении R.


  • Статические 3D точечные диаграммы
    • Простые 3D-графики рассеяния: пакет scatterplot3d
    • Расширенные 3D-графики: пакет plot3D
  • Интерактивные 3D-графики
    • Интерактивные трехмерные точечные диаграммы
    • Руководство по системе 3D-визуализации RGL

  • Установка и загрузка scaterplot3d
  • Подготовить данные
  • Функция scatterplot3d()
  • Базовые 3D-диаграммы рассеяния
  • Изменить основной заголовок и метки осей
  • Изменить форму и цвет точек
  • Изменить формы точек по группам
  • Изменить цвета точек по группам
  • Изменить общий вид графика
    • Убрать рамку вокруг участка
    • Добавить сетки на scatterplot3d
  • Добавить бары
  • Модификация вывода scatterplot3d
    • Добавить легенды
    • Добавить метки точек
    • Добавить плоскость регрессии и дополнительные точки

Подробнее: —> Простые точечные 3D-графики: пакет scatterplot3d.

  • Установка и загрузка пакета plot3D
  • Подготовить данные
  • Диаграммы рассеяния
    • Базовая диаграмма рассеяния
    • Изменить тип рамки вокруг графика
    • Изменение цвета по группам
    • Изменить положение легенды
    • Направление просмотра 3D
    • Заголовки и метки осей
    • Пометки и этикетки
    • Добавить точки и текст к существующему графику
  • Линейные графики
    • Добавить доверительный интервал
    • Причудливый 3D-график рассеяния с маленькими точками на базовой плоскости
    • Плоскость регрессии
  • text3D: нанесение трехмерных текстов
  • text3D и scatter3D
  • Трехмерная гистограмма
  • scatter2D: 2D точечная диаграмма
  • текст2D
  • Интерактивный сюжет

Подробнее: —> Расширенные 3D-графики: пакет plot3D.

  • Установка и загрузка необходимых пакетов
  • Подготовить данные
  • Функция scatter3d
  • Базовые 3D-диаграммы рассеяния
  • Нанесите точки на группы
    • График по умолчанию
    • Удалить поверхности
    • Добавить эллипсоиды концентрации
    • Изменить цвета точек по группам
  • Оси
    • Изменить метки осей
    • Удалить шкалы осей
    • Изменить цвета оси
  • Добавить текстовые метки для точек
  • Экспорт изображений

3D точечный график rgl

Подробнее: —> Интерактивные 3D точечные графики.

  • Установить пакет RGL
  • Загрузить пакет RGL
  • Подготовить данные
  • Запустить и закрыть устройство RGL
  • Трехмерная точечная диаграмма
    • Базовый график
    • Изменить фон и цвета точек
    • Изменить форму точек
  • rgl_init(): пользовательская функция для инициализации устройства RGL
  • Добавление украшения ограничивающей рамки
  • Добавить осевые линии и метки
  • Установите пропорции осей x, y и z
  • Изменение цвета точек по группам
  • Изменить форму точек
  • Добавить эллипс концентрации
  • Плоскость регрессии
  • Создать фильм сцены RGL
  • Экспорт изображений в формате png или pdf
  • Экспорт графика в интерактивный файл HTML
  • Выберите прямоугольник в сцене RGL
  • Определение точек на графике
  • Интерфейс R3D
  • функции РГЛ
    • Управление устройством
    • Функции формы
    • Управление сценами
    • Настройка среды
    • Настройка внешнего вида
    • Экспорт скриншота
    • Назначить фокус окну RGL

RGL фильм 3d

Подробнее —> Руководство по системе 3D визуализации RGL.

  • R Базовые графики
  • Решетчатые графики
  • ggplot2 Графики
  • Элементы выбора цветов для отличной визуализации данных в R

Понравилась эта статья? Я был бы очень признателен, если бы вы помогли его распространению, отправив его по электронной почте другу или поделившись им в Twitter, Facebook или Linked In.

Подарите мне немного любви с помощью кнопок «Нравится» ниже… Спасибо и, пожалуйста, не забудьте поделиться и прокомментировать ниже!!

Avez vous aimé c этой статьей? Je vous serais très reconnaissant si vous aidiez à sa диффузия en l’envoyant par courriel à un ami ou en le partageant sur Twitter, Facebook или Linked In.

Montrez-moi un peu d’amour avec les like ci-dessous … Merci et n’oubliez pas, s’il vous plaît, de partager et de commenter ci-dessous!



‎Quick Graph в App Store

Описание

Это мощный, высококачественный графический калькулятор, который в полной мере использует мультитач-дисплей и мощные графические возможности iPad и iPhone как в 2D, так и в 3D.

«Приложение, которое вам нужно, чтобы пережить следующий семестр».
— Gizmodo

Простой, но интуитивно понятный интерфейс позволяет легко вводить и/или редактировать уравнения и визуализировать их в математической записи.

Он способен отображать явные и неявные (opt) уравнения, а также неравенства (opt) как в 2D, так и в 3D, во всех стандартных системах координат: декартовой, полярной, сферической и цилиндрической, все с фантастической скоростью и прекрасными результатами, которые можно скопировать, отправить по электронной почте или сохранить в библиотеке фотографий.

Он также включает функцию оценки для оценки уравнений в определенных точках и библиотеку, в которой можно хранить часто используемые уравнения.

Одновременно можно визуализировать до 6 уравнений в режимах 2D и 3D; это ограничение можно снять, купив расширенный набор функций.

Особенности:
— Enhanced 2D — Режим 3D-графики для великолепной графики высокого разрешения.
— Адаптивные 2D-алгоритмы.
— Гиперболические и обратные функции.
— Такие функции, как Мин., Макс., если.
— Настраиваемая десятичная ось / π
— Независимое двумерное масштабирование
— Поддержка декартовой, полярной, цилиндрической и сферической систем координат.
— Оценка интерактивного выражения.
— Библиотека часто используемых уравнений.
— Проведите по экрану, чтобы удалить уравнения из списка уравнений.
— Сохранить в фотобиблиотеке.
— Копировать в буфер обмена.
— Включить/выключить графики
— Перетащите, чтобы повернуть — переместить.

Расширенный набор функций дает вам доступ к:
— Поддержка внешнего экрана
— 2D-трассировка.
— 2D и 3D неявные графы.
— 2D и 3D неравенства.
— Корни и пересечения (2D)
— Таблица значений
— Дополнительные функции!

Пожалуйста, посетите веб-сайт для получения более подробной информации.

Версия 3.0

Лучше, быстрее, мощнее. Мы почистили за кулисами, чтобы сделать ваш Калькулятор еще лучше.

Прошло много времени с момента предыдущего обновления; в течение этого времени мы перестраивали Калькулятор с нуля, чтобы убедиться, что мы решили проблемы, с которыми вы сталкивались, и в полной мере использовали преимущества новых технологий, которые есть в нашем распоряжении сегодня.

— Мы упростили написание уравнений.
— Улучшена панель управления, чтобы сделать интерфейс еще более простым.
— Создана система визуализации, позволяющая повысить детализацию и точность построения графиков.

Рейтинги и обзоры

337 оценок

Замечательный!

Если вы посещаете какой-либо курс математики, это мощный инструмент, помогающий визуализировать то, с чем вы работаете. Я использовал это со времен Алгебры I, и мне всегда было интересно, в чем смысл трехмерного графика и как его использовать. Сейчас я изучаю многомерное исчисление в колледже и регулярно использую трехмерные графики. Это помогло мне визуализировать и понять касательные плоскости и трехмерные функции. Я бы хотел, чтобы он мог показывать точки пересечения на трехмерном графике (может быть, он и делает, но я еще не разобрался с этим), но он делает больше, чем многие другие калькуляторы, и делает это более элегантно. Мне нравится это приложение, и я рекомендую его всем, кто занимается математикой, инженерным делом или даже старшекласснику, который ищет быстрый, дешевый и простой графический калькулятор

Выдающийся. Спасибо за это приложение.

Кратко, просто, эффективно, профессионально. Делает тяжелую работу до мельчайших деталей. Если кто-то ежедневно использует математику и ему надоели ложные интерфейсы, это его математическое приложение. Возможность визуализировать поверхности с различными уровнями контрастности помогает с относительной ориентацией и определением их пересечений. Трехмерная перспектива под любым углом является точной. Это приложение было создано высокопрофессиональными поклонниками математики. Сообщество в долгу!

Отличная графическая программа!

Приложение работает намного быстрее, чем построение графиков на TI 84 или TI 89. Можно без труда изменить размер графика и выбрать цвет для каждого из графиков. Можно легко построить уравнения алгебры, тригонометрические, экспоненциальные и логарифмические задачи. Я часто использую это приложение для подготовки к Calc и очень рекомендую его. Версия для iPad намного больше и графики красивее!

Разработчик, KZ Labs, указал, что политика конфиденциальности приложения может включать обработку данных, как описано ниже.

Как найти модуль числа примеры: Как найти модуль и аргумент комплексного числа: онлайн, примеры

Модуль числа | Образовательная социальная сеть

Модуль числа

Цель:

изучение понятия модуля,

применение определения модуля при выполнении задач.

Задачи:

развивать умение применять теоретический материал при решении практических задач;

развивать интерес к математике через поиск примеров по данной теме;

расширить математический кругозор;

приобрести навыки исследовательской работы.

Считаю, что выбранная тема является актуальной:

Задачи, связанные с абсолютной величиной, часто встречаются на математических олимпиадах и вступительных экзаменах.

Понятие модуля широко применяется в различных разделах школьного курса математики.

Это понятие является одним из основных понятий элементарной математики. Осмысленное владение модулем позволяет воспринимать алгебру и геометрию, как единое целое. “Расстояние между точками” позволяет оценивать правильность найденных решений ряда уравнений, содержащих модуль, строить графики функций.

В ходе работы я использовала следующие методы:

Исследование литературы по теме.

Проведение поиска задач по теме.

Основная часть

Существенной характеристикой числа является понятие его абсолютной величины (модуля).

Модулем числа называют расстояние от точки, изображающей число на координатной прямой до начала отсчета.

В различных учебниках первоначальное понятие модуля вводится по-разному: как расстояние от точки, изображающей число, до начала отсчёта (Математика. Н.Я. Виленкин), как длина вектора (Математика. П.М. Эрдниев), как число “без знака” (Математика. Г.В. Дорофеев) и др.

В архитектуре – это исходная единица измерения,  устанавливаемая для данного архитектурного сооружения и служащая для выражения кратных соотношений его составных элементов.

В технике – это термин, применяемый в различных областях техники, не имеющий универсального значения и служащий для обозначения различных коэффициентов и величин, например, модуль зацепления, модуль упругости и т. п.

Понятие модуля

Модуль (modulus) в переводе с латинского языка означает “мера, размер”.

Модуль числа а обозначают | а |. Этот термин “модуль” ввёл в 1806 г. французский математик Жорж Аргон.

Геометрический смысл модуля

Модулем числа а называют расстояние (в единичных отрезках) от начала координат до точки А(а).

Модуль числа 5 равен 5, так как точка В(5) удалена от начала отсчета на 5 единичных отрезков. Пишут: |5| = 5.

Расстояние точки М(-6) от начала отсчета О равно 6 единичным отрезкам. Число 6 называют модулем числа -6.

Пишут: |-6| = 6.

Модуль числа не может быть отрицательным. Для положительного числа и нуля он равен самому числу, а для отрицательного – противоположному числу. Противоположные числа имеют равные модули.

|-а| = |а|

Модуль числа 0 равен 0, так как точка с координатой 0 совпадает с началом отсчета О, т.е. удалена от нее на 0 единичных отрезков.

|0| = 0

Так как модуль числа – это расстояние, он никогда не будет отрицательным.

Изучая понятие модуля, я рассмотрела доказательство следующей теоремы:

Абсолютная величина действительного числа a ≠ 0 равна большему из двух чисел a или -a.

        

Доказательство:

Если число a положительно, то -a отрицательно, т.е. –a

Например, число 5 положительно, тогда -5 – отрицательно и -5

В этом случае |a| = a, т.е. |a| совпадает с большим из двух чисел a и -a.

Если a отрицательно, тогда -a положительно и a

Для нахождения модуля числа можно использовать следующую блок-схему.

Отработка алгоритма. Допустим, необходимо найти модуль чисел -3 и 7.

В учебниках приводятся различные упражнения с использованием модуля числа. Вот некоторые из них:

Запишите число, противоположное данному: 4; -4; +3; -3; -6,3; 6,3.

2. Найдите модуль каждого из чисел: |- 6 |, | 9 |, | — 5 |, | 0 |, |0,8 |.

3. Найти расстояние от М(-7) и N(6) до начала отчета на координатной прямой.

При решении задач, содержащих модуль числа, основным приемом является раскрытие знака модуля в соответствии с его свойствами.

В некоторых случаях модуль раскрывается однозначно.

Например: |x2 + y2| = x2 + y2, так как выражение под знаком модуля неотрицательно при любых х и у. Или |–z2 – 1| = z2 + 1, так как выражение под модулем отрицательно при любых z.

Уравнения, содержащие знак модуля, решаются следующими способами:

алгебраический,  

графический,  

последовательное раскрытие модулей,

метод интервалов.

Рассмотрим некоторые примеры решения уравнений и неравенств, содержащих знак модуля.

Решить уравнение: |x| = 3. Мы видим, что на числовой прямой есть две точки, расстояние от которых до нуля равно трём. Это точки 3 и -3. Значит, у уравнения |x| = 3 есть два решения:

x = 3 и x = -3.

Решить уравнение: |x — 3| = 4.

Это уравнение можно прочитать так: расстояние от точки до точки равно 4. С помощью графического метода можно определить, что уравнение имеет два решения: — 1 и 7.

Решить неравенство: |x + 7|

Можно прочитать как: расстояние от точки до точки меньше четырёх. Ответ: (-11; -3).

Решить неравенство: |10 — x| ≥ 7.

Расстояние от точки 10 до точки х больше или равно семи.

Ответ: (-∞; 3]U [17, +∞)

Рассматривая модуль числа, я познакомилась с функцией y = |x|, графиком которой является ломаная линия, состоящая из двух лучей, являющихся биссектрисами I и II координатных четвертей.

Действительно,

Для x ≥ 0 имеем y = x.

Для x

ЗАКЛЮЧЕНИЕ

В результате работы я:

повторила школьный материал по данной теме,

изучила решение уравнений и неравенств, содержащих знак модуля,

научилась строить график функции вида y = |x|,

Так как изучение модуля числа продолжается в старших классах, где рассматриваются свойства модуля, а также задачи различного уровня сложности, исследование данной темы будет продолжено. В следующем году я проведу исследование задач  различного уровня сложности, а также олимпиадные и экзаменационные задачи.

Модуль числа. 6 класс — презентация онлайн

Похожие презентации:

Элементы комбинаторики ( 9-11 классы)

Применение производной в науке и в жизни

Проект по математике «Математика вокруг нас. Узоры и орнаменты на посуде»

Знакомство детей с математическими знаками и монетами

Тренажёр по математике «Собираем урожай». Счет в пределах 10

Методы обработки экспериментальных данных

Лекция 6. Корреляционный и регрессионный анализ

Решение задач обязательной части ОГЭ по геометрии

Дифференциальные уравнения

Подготовка к ЕГЭ по математике. Базовый уровень Сложные задачи

1. Тема урока : «Модуль числа»

2. Знаете ли вы, …

1.Что такое координатная прямая?
2.Что называют координатой точки на
прямой?
3.Какие числа называются
противоположными?
4.Как обозначается число, противоположное
числу а?
5.Какие числа называют целыми?
Устный счёт:
1.Даны числа: -9; 12; 3/5; -4,6; 9; 6,08;
-3/5; 0,001; 123; -12; 0.
• Назовите отрицательные,
положительные, натуральные,
дробные, целые числа.
• Назовите числа, противоположные
данным числам.
2.Каким числом будет число –а, если:
• а – отрицательное;
• а = 0;
• а – положительное число.
Запишите число
противоположное данному:
7

4
–(–
5)

–7
4
5
-3
+(–
6)
-6
–(–
2)
+
2
-9

(+9)
–(–(–
-8

5. Упражнения

1. На координатной прямой отмечены
точки М (-7), К(6), В(-6), С(-0,5),
Д(0,5) Какие из них имеют
противоположные координаты?

6. Упражнения

2.Найти расстояние от М(-7) и К(6)
до начала отсчета на координатной
прямой.

7. Упражнения

4.Найдите числа, если на
координатной прямой они находятся
на расстоянии:
а) 6 единиц от числа 0,
б) на 10 единиц от числа -4
Из истории математики
Модуль
Модуль числа
числа аа
обозначают
обозначают |а|.
|а|.
Этот
Этот термин
термин
«модуль»
«модуль»
ввел
ввел вв 1806году
1806году
французский
французский
математик
математик Жорж
Жорж Аргон.
Аргон.

9. Модулем числа а называют расстояние (в единичных отрезках) от начала координат до точки А(а)

0
А(а)
а единиц

10.

Модуль положительного числа равен самому числу. Модуль нуля равен нулю.А(7)
0
7 единиц
│7│=7
│1,5│= 1,5
│0│ = 0

11. Модуль отрицательного числа равен противоположному числу.

0
А(- 7)
7 единиц
│- 7│= 7
│- 1,5│ = 1,5

12. Противоположные числа имеют равные модули.

│5│ = 5


-5
5 единиц
│- 5│ = 5
0
5
5 единиц
Модуль не может быть
отрицательным числом!

13. Прикольно!

Представь, что модуль –
это баня, а знак «минус»
— грязь.
Оказываясь под знаком
модуля, отрицательное
число «моется» и
выходит без знака
«минус» — чистым.
В бане могут «мыться»
(т.е. стоять под знаком
модуля) как
положительные, так и
отрицательные числа.

14. Найдите модуль каждого из чисел:

12
• │12│=
7,08
• │7,08│=
6,32
• │- 6,32│=
0
• │0│=
72
• │ -72│=

15. Найдите значение выражения


│- 8│+│- 2│=
│- 5│-│ 2│= 10
│- 8│∙│ — 3│= 3
│- 27│:│-9│= 24
3

16.

Решение уравнений│х — а│ — расстояние от а до х
Решите уравнение.
│х │ = 4
х
-4
Ответ.
0
4
Х=-4 и х=4

17. Примеры решений уравнений.

│Х- 2│ = 5
2
-3
-5
Ответ.
Х=-3 и Х=7
7
5

18. Решите уравнения


│х│ = 25
│х — 12│ = 6
│х — 3│ = 0
│х│ = — 7,5
х = 25 и х = 25
х = 18 и х = 6
х = 3 и х = -3
Корней нет

19. Поняли?

А теперь…

20. Самостоятельная работа

Вариант 1
Найдите модуль числа:
— 23; 0,34; — 2/3; 2 3/4.
Запишите числа, модуль которых равен:
4; 0, 23; 3/7; 3 1/4.
Вариант 2
Найдите модуль числа:
52; — 1, 24; — 4 2/3; 3/4.
Запишите числа, модуль которых равен:
9; 0,56; 2 5/7; 1/8.

21. Проверка

Вариант 1 
1. |- 23|=23; |0,34|= 0,34; |2/3|= 2/3; |23/4|=23/4.
2. 4 =|-4|=|4|; 0,23=|-0, 23|=|0,23|;
3/7 = |- 3/7|=|3/7|; 3 1/4 = |-3 1/4|=| 3 1/4 |
Вариант 2 
1. |52 | = 52; | -1,24| = 1, 24; |- 4 2/3| = 4 2/3;
|3/4 | = 3/4
2. 9 = | -9 | = | 9 |;
0,56 = |- 0, 56 | = | 0,56 |;
2 5/7 = | -2 5/7 | = | 2 5/7 |; 1/8 = | -1/8 | = | 1/8 |
СПАСИБО ЗА
ВНИМАНИЕ!!!

English     Русский Правила

б мод м . Сначала мы воспользуемся наивным подходом, а затем посмотрим, как мы можем его усовершенствовать.

Сначала уменьшите по модулю m . Это означает, что нужно найти число a1 так, чтобы 0 <= a1 < m и a = a1 mod m . Затем несколько раз в цикле умножьте на a1 и снова уменьшите mod m . Таким образом, в псевдокоде:

 a1 = сокращенный мод m
р = 1
for(int i = 1; i <= b; i++) {
    р *= а1
    p = p приведенный по модулю m
}
 

Делая это, мы избегаем чисел больше 955 мод 221 . Во-первых, 5 это уже уменьшенный мод 221 .

  1. 1 * 5 = 5 мод 221
  2. 5 * 5 = 25 мод 221
  3. 25 * 5 = 125 мод 221
  4. 125 * 5 = 183 мод 221
  5. 183 * 5 = 31 мод 221
  6. 31 * 5 = 155 мод 221
  7. 155 * 5 = 112 мод 221
  8. 112 * 5 = 118 мод 221
  9. 118 * 5 = 148 по модулю 221
  10. 148 * 5 = 77 мод 221
  11. 77 * 5 = 164 мод 221
  12. 164 * 5 = 157 мод 221
  13. 157 * 5 = 122 мод 221
  14. 122 * 5 = 168 мод 221
  15. 168 * 5 = 177 мод 221
  16. 177 * 5 = 1 мод 221
  17. 1 * 5 = 5 мод 221
  18. 5 * 5 = 25 мод 221
  19. 25 * 5 = 125 мод 221
  20. 125 * 5 = 183 мод 221
  21. 183 * 5 = 31 мод 221
  22. 31 * 5 = 155 мод 221
  23. 155 * 5 = 112 мод 221
  24. 112 * 5 = 118 мод 221
  25. 118 * 5 = 148 мод 221
  26. 148 * 5 = 77 мод 221
  27. 77 * 5 = 164 мод 221
  28. 164 * 5 = 157 мод 221
  29. 157 * 5 = 122 мод 221
  30. 122 * 5 = 168 мод 221
  31. 168 * 5 = 177 мод 221
  32. 177 * 5 = 1 мод 221
  33. 1 * 5 = 5 мод 221
  34. 5 * 5 = 25 мод 221
  35. 25 * 5 = 125 мод 221
  36. 125 * 5 = 183 мод 221
  37. 183 * 5 = 31 мод 221
  38. 31 * 5 = 155 мод 221
  39. 155 * 5 = 112 мод 221
  40. 112 * 5 = 118 по модулю 221
  41. 118 * 5 = 148 мод 221
  42. 148 * 5 = 77 мод 221
  43. 77 * 5 = 164 мод 221
  44. 164 * 5 = 157 мод 221
  45. 157 * 5 = 122 мод 221
  46. 122 * 5 = 168 мод 221
  47. 168 * 5 = 177 мод 221
  48. 177 * 5 = 1 мод 221
  49. 1 * 5 = 5 мод 221
  50. 5 * 5 = 25 мод 221
  51. 955 = 112 по модулю 221 .

    Теперь мы можем улучшить это, используя возведение в степень путем возведения в квадрат; это известный трюк, в котором мы уменьшаем возведение в степень до требуемого только log b умножений вместо b . Обратите внимание, что с алгоритмом, который я описал выше, возведением в степень путем возведения в квадрат улучшения, вы получите двоичный метод справа налево.

     a1 = уменьшенный мод m
    р = 1
    в то время как (b > 0) {
     если (b нечетно) {
     р *= а1
     p = p приведенный по модулю m
     }
     б / = 2
     a1 = (a1 * a1) приведенный по модулю m
    }
     92 mod 221  и т. д.

    Приведенный выше алгоритм формализует эту идею.

    Как найти модуль комплексного числа

    Как найти модуль комплексного числа

    Как найти модуль комплексного числа ?

    Пусть z = a + ib — комплексное число.

    Модуль или абсолютное значение z, обозначаемое | г | определяется как

    Модуль свойств комплексных чисел


    Свойство 1 :

    Модуль суммы двух комплексных чисел всегда меньше или равен сумме их модулей.

    Приведенное выше неравенство может быть немедленно распространено по индукции на любое конечное число комплексных чисел, т. е. для любых n комплексных чисел z 1 , z 2 , z 3 , …, z n

    |z 1 + z 2 + z 3 + … + zn | ≤ | я 1 | + | я 2 | + … + | я н |

    Свойство 2 :

    Модуль разности двух комплексных чисел всегда больше или равен разности их модулей.

    Свойство 3 :

    Модуль произведения двух комплексных чисел равен произведению их модулей.

    Свойство 4 :

    Модуль отношения двух комплексных чисел равен отношению их модулей.

    Давайте рассмотрим несколько примеров, основанных на вышеуказанной концепции.

    Пример 1:

    Найдите модуль следующего комплексного числа

    − 2 + 4i

    Решение:

    Пусть z =  -2 + 4i

    5 |z| =  √(-2 + 4i)

    |z| = √ (-2) 2 + 4 2

    = √4 + 16

    = √20

    , разлагая число внутри радикала, мы получаем

    = √ (2 ⋅ 2)

      =  2√5

    Пример 2 :

    Найдите модуль следующего комплексного числа

    2 − 3i

    Решение:

    Пусть z = 2 − 3i

    |z| =  √(2 - 3i)

    |z| = √2 2 + (-3) 2

    = √4 + 9

    = √13

    Пример 3:

    Найти модуль следующего комплекса

    -3-2I 33333353333333333535353535353535353535353535353535353535353335333533353333333353333333333333333333333333533333533333333353333353335333533333533333333н

    Решение:

    Пусть z =  − 3 − 2i

    |z| =  √(− 3 − 2i)

    |z| =  √(-3) 2 + (-2) 2

    = √9 + 4

    = √13

    Пример 4:

    Найти модуль следующего комплекса №

    4 + 3I

    334.

    Пусть z =  4 + 3i

    |z| =  √(4 + 3i)

    |z| =  √4 2  + 3

      =  √16 + 9

      =  √25

    0005

    =  √5

    Давайте рассмотрим следующий пример «Как найти модуль комплексного числа».

    Пример 5:

    Найдите модуль или абсолютное значение

    [(1 + 3i) (1 - 2i)] / (3 + 4i)

    Решение:

    3 |[ ) (1 - 2i)] / (3 + 4i) | =  |(1 + 3i) (1 - 2i)| / |3 + 4i|

      =  |(1 + 3i)| |(1 - 2i)| / |3 + 4i|

      =  √(1 2  + 3 2 ) √(1 2  + (-2) 2 )/√3 2 + 4 2

    = (√ (1+ 9) √ (1+ 4))/√ (9+ 16)

    = (√10 √ (9+ 16)

    = (√10 a 5)/√25

    = √50/√25 = 5√2/5 = √2

    Связанные темы

    • Свойства комплексов
    • Добавить и подчитывать комплексы
    • Как найти модле и модле. аргумент комплексного числа

    Мы надеемся, что после изучения вышеизложенного учащиеся поняли "Как найти модуль комплексного числа".

Умножить на сопряженное выражение: Метод умножения числителя и знаменателя на сопряженное выражение — Мегаобучалка

Примеры решения типовых задач

Пример 1. Доказать .

Решение: Рассмотрим величину:

.

Пусть – произвольное число, выберем ; тогда если , то , следовательно, .

Таким образом, по определению, .

Пример 2. Вычислить .

Решение. Используя свойства предлов функций, получим:

.

Пример 3. Вычислить .

Решение.

.

Чтобы раскрыть неопределенность вида , заданную отношением двух многочленов, надо в числителе и в знаменателе выделить множитель, равный нулю при предельном значении , и сократить на него.

Пример 4. Вычислить .

Решение. Подставляя предельное значение в числитель и знаменатель, получаем, что оба выражения обращаются при этом в нуль. Имеем неопределенность вида . Стоящие в числителе и знаменателе многочлены можно разложить на множители. Следует помнить, что если , – корни квадратного трехчлена , то справедлива формула

.

Таким образом, имеем:

.

Пример 5. Вычислить

Решение. Имеет место неопределенность вида . Так как является корнем многочленов из числителя и знаменателя, то выделяется как сомножитель в числителе и знаменателе.

.

Чтобы раскрыть неопределенность вида , в которой числитель или знаменатель содержит иррациональность, следует соответствующим образом избавиться от нее (например, умножить на сопряженное выражение или ввести новую переменную).

Пример 6. Вычислить .

Решение. Подставляя предельное значение в числитель и знаменатель, получаем неопределенность вида . Знаменатель представляет собой «сумму кубов», поэтому при разложении его на множители получаем: . После умножения числителя и знаменателя на сопряженное числителю выражение , имеем:

.

Пример 7. Вычислить

Решение. Имеет место неопределенность вида . Произведем замену Тогда при имеем

Чтобы раскрыть неопределенность вида , заданную отношением двух многочленов, надо числитель и знаменатель разделить на самую высокую степень , а затем перейти к пределу.

Пример 8. Вычислить .

Решение. Имеем неопределенность вида . Для ее раскрытия можно либо разделить числитель и знаменатель на наибольшую степень переменной x и, учитывая, что величина обратная бесконечно большой величине есть бесконечно малая величина, раскрыть исходную неопределенность, либо вынести переменную в наибольшей степени в числителе и знаменателе дроби и сократить на наибольшую степень.

.

Пример 9. Вычислить .

Решение. Имеем неопределенность вида . Раскрываем ее аналогично тому, как это сделано в примере 8.

.

Пример 10. Вычислить .

Решение. Имеем неопределенность вида . Раскрываем ее аналогично тому, как это сделано в примере 8.

.

Пример 11. Вычислить

Решение. Имеем неопределенность вида . Избавимся от нее следующим образом: разделим числитель и знаменатель на степень с наивысшим основанием, т.е. на . Затем воспользуемся равенством если

Пример 12. Вычислить .

Решение. Очевидно, что при и . Поэтому имеем неопределенность вида . Далее получаем:

.

Неопределенности вида возникают, как правило, либо при исследовании разности двух дробей (в этом случае рекомендуется приводить дроби к общему знаменателю), либо при рассмотрении разности иррациональных выражений (для избавления от иррациональностей следует преобразовать исходное выражение либо к разности квадратов, либо к сумме или разности кубов).

Пример 13. Вычислить .

Решение. В данном случае имеем неопределенность . Приведем дроби к общему знаменателю:

.

Пример 14. Вычислить .

Решение. В данном случае, чтобы раскрыть неопределенность , необходимо умножить и разделить рассматриваемое выражение на «сопряженное», чтобы прийти к разности квадратов. Для таким «сопряженным» является . Таким образом, получаем:

.

Таким образом, мы попали в ситуацию, разобранную при решении примера 12. Проведем соответствующие преобразования в знаменателе:

.

При вычислении пределов, содержащих тригонометрические функции, полезно использовать «первый замечательный предел» .

Пример 15. Вычислить .

Решение. Очевидно, что при , и . Чтобы применить первый замечательный предел, необходимо получить в знаменателе выражение, совпадающие с аргументом синуса. Для этого числитель и знаменатель умножаем на число 4:

.

Пример 16. Вычислить .

Решение. Знаменатель разложим на множители как разность квадратов, а в числителе воспользуемся формулой :

.

Пример 17. Вычислить .

Решение. В данном случае, чтобы раскрыть неопределенность , можно воспользоваться формулой :

В примерах с неопределенностью выражение, стоящее под знаком предела представляет собой показательно–степенную функцию. Неопределенность устраняется при помощи выделения «второго замечательного предела» .

Пример 18. Вычислить .

Решение. Имеем неопределенность вида . Для раскрытия этой неопределенности воспользуемся вторым замечательным пределом:

.

Пример 19. Найти предел функции .

Решение. Имеем неопределенность вида , преобразуем ее к неопределенности вида . Пользуясь свойствами логарифмов: и , получим:

.

Далее

.

Пример 20. Найти предел функции .

Решение. В данном примере при выяснении вида неопределенности видим, что таковой не имеется. Имеем , тогда .

Пример 21. Найти предел функции .

Решение. Имеем неопределенность вида . Для раскрытия этой неопределенности воспользуемся вторым замечательным пределом :

.

Пример 22. Найти предел функции .

Решение. Выделим в числителе, выражение вида , а в знаменателе – . Затем воспользуемся следующим равенствами и :

.

Урок по теме «Комплексные числа и операции над ними»

Урок по теме «Комплексные числа и операции над ними» 2часа

Цель урока: расширить и закрепить знания учащихся над понятием комплексного числа и действий над ним.

Задачи урока.

Образовательные:

        1. Расширить понятие комплексного числа.

        2. Отработать и проверить знания учащихся при выполнении действий в алгебраической форме комплексного числа.

        3. Рассмотреть геометрическую интерпретацию комплексных чисел.

      Развивающие:

      1. Развивать мышление в процессе выполнения практических заданий.

      2. Развивать пространственные представления.

      Воспитывающие:

      1. Воспитывать культуру записей в тетради.

      2. Воспитывать аккуратность, усидчивость, внимательность в процессе прослушивания лекции.

      Тип урока: комбинированный.

      Ход урока

      1. Организационный момент.

      — проверка д/р

      2. Устный счет

      — Что такое «комплексное число»

      — чему равен квадрат мнимой «1»

      — какие числа называются сопряженными

      — чему равно произведение сопряженных чисел

      — для чего нужны комплексные числа (для решения квадратных ур-ний с отр. дискриминантом)

      Решить квадратное уравнение 

      Вычислим дискриминант:

      Дискриминант отрицателен, и в действительных числах уравнение решения не имеет. Но корень можно извлечь в комплексных числах!

      По известным формулам получаем два корня:

       – сопряженные комплексные корни

      Таким образом, уравнение   имеет два сопряженных комплексных корня:  , 

      3. Операции с комплексными числами

      Над комплексными числами в алгебраической форме можно выполнять следующие действия.

      Сложение комплексных чисел

      Пример 1

      Сложить два комплексных числа  , 

      Для того чтобы сложить два комплексных числа нужно сложить их действительные и мнимые части:

      Вычитание комплексных чисел

      Пример 2

      Найти разности комплексных чисел   и  , если  , 

      Действие аналогично сложению, единственная особенность состоит в том, что вычитаемое нужно взять в скобки, а затем – стандартно раскрыть эти скобки со сменой знака:

      Результат не должен смущать, у полученного числа две, а не три части. Просто действительная часть – составная:  . Для наглядности ответ можно переписать так:  .

      Умножение комплексных чисел

      Пример 3

      Найти произведение комплексных чисел   , 

      Очевидно, что произведение следует записать так:

      Что напрашивается? Напрашивается раскрыть скобки по правилу умножения многочленов. Так и нужно сделать! Все алгебраические действия вам знакомы, главное, помнить, что   и быть внимательным.

      Чтобы умножить многочлен на многочлен нужно каждый член одного многочлена умножить на каждый член другого многочлена.

      Я распишу подробно:

      Деление комплексных чисел

      Пример 4

      Даны комплексные числа  ,  . Найти частное  .

      Составим частное:

      Деление чисел осуществляется методом умножения знаменателя и числителя на сопряженное знаменателю выражение.

      Вспоминаем бородатую формулу   и смотрим на наш знаменатель:  . В знаменателе уже есть  , поэтому сопряженным выражением в данном случае является  , то есть 

      Согласно правилу, знаменатель нужно умножить на  , и, чтобы ничего не изменилось, домножить числитель на то же самое число  :

      Далее в числителе нужно раскрыть скобки (перемножить два числа по правилу, рассмотренному в предыдущем пункте). А в знаменателе воспользоваться формулой   (помним, что  и не путаемся в знаках!!!).

      Распишу подробно:

      Пример 5

      Возвести в степень комплексные числа  ,  , 

      Здесь тоже всё просто, главное, помнить знаменитое равенство.

      Если мнимая единица возводится в четную степень, то техника решения такова:

      Если мнимая единица возводится в нечетную степень, то «отщипываем» одно «и»,  получая четную степень:

      Если есть минус (или любой действительный коэффициент), то его необходимо предварительно отделить:

      Пример 6 Возвести в степень комплексные числа  , 

      Решение:

      Пример 7

      Найти корни уравнения   и разложить квадратный двучлен на множители.

      Решение:

      Разложим квадратный двучлен на множители:

      Пример 8

      Вычислить: (2+3i)(3−i)

      Решение:

      (2+3i)(3−i)=6−2i+9i−3i2=6+7i+3=9+7i.(2+3i)(3−i)=6−2i+9i−3i2=6+7i+3=9+7i

      Пример 9  Найти частное   , если   .

      Решение.

         .

      Пример 10 Вычислить а)   , б)   .

      Решение. а)   .

      б)   . Запомним: 

      4. Геометрическое изображение комплексных чисел.

      Комплексные числа изображают точками плоскости по следующему правилу: a + bi = M (a; b) (рис.1).

      Рисунок 1

      Пример. Постройте точки, изображающие комплексные числа:

      1; — i; — 1 + i; 2 – 3i (рис. 3).

      Рисунок 3

      5. Работа в парах

      Постройте на комплексной плоскости следующие

      комплексные числа: ,  ,  ,  , 
      ,  ,  , 

      6. Домашнее задание

      Дома учащимся предлагается выполнить задание

      показать на плоскости три примера комплексных чисел (на каждый вид)

      7. Подведение итогов урока

        • Что нового мы сегодня узнали

        • Трудная это тема: работа с комплексными числами

        • Оценка всех за работу у доски / работу на карточках (1 урок)

        и работу группы (2 урок).

        Вопрос Видео: Доказательство свойств присоединенной матрицы квадратной матрицы порядка 𝑛.

        Стенограмма видео

        Верно или неверно: если 𝐴 задано квадратная матрица порядка 𝑛, то 𝐴, умноженное на присоединенную матрицу 𝐴, равно к сопряженному 𝐴, умноженному на 𝐴, равен определителю 𝐴, умноженному на 𝐼, где 𝐼 — единичная матрица порядка 𝑛.

        Начнем с того, что если сингулярна, т. е. определитель 𝐴 равен нулю, то инверсия 𝐴 не существует. И мы еще вернемся к этому делу чуть позже. Если, с другой стороны, 𝑛-by-𝑛 матрица невырождена, а это означает, что существует обратная ей матрица, то показать, является ли данное утверждение истинно или ложно, мы можем использовать теорему о том, что для неособого 𝑛-by-𝑛 матрица 𝐴, обратная 𝐴 — это 𝐴 отрицательная — является сопряженной к 𝐴 разделить на определитель 𝐴. И напомним, что сопряжение матрица 𝐴 является транспонированной матрицей ее сомножителей.

        Итак, давайте предположим, что обратное значение наша матрица существует. Тогда это означает, что определитель отличен от нуля. Чтобы показать, является ли это утверждение правда или ложь, давайте возьмем наше определение, обратное 𝐴, и умножим на слева матрицей 𝐴. Теперь мы знаем, что для любого невырожденная матрица 𝑛 на 𝑛, 𝐴, умноженная на ее обратную, является тождеством матрица. Итак, теперь с левой стороны мы имеют единичную матрицу 𝑛-by-𝑛. А теперь, поскольку определитель матрица 𝐴 является скалярной, на нее можно умножить. И это дает нам определитель 𝐴, умноженное на единичную матрицу, равно 𝐴, умноженному на ее сопряженное. И это показывает, что наш первый утверждение 𝐴 сопряженное 𝐴 действительно равно определителю 𝐴, умноженному на 𝐼 для невырожденной матрицы 𝐴.

        Теперь, если мы снова начнем с нашего формулу обратной к 𝐴 и умножить это время справа на наше матрица 𝐴, мы снова имеем единичную матрицу слева, так как 𝐴 обратная 𝐴 является единичная матрица. И снова умножая на определитель 𝐴, что мы можем сделать, поскольку это скаляр, у нас есть определитель 𝐴, умноженное на единичную матрицу, равно сопряженному 𝐴, умноженному на 𝐴. И поэтому можно сказать, что наш второй выражение верно и для невырожденной матрицы. Все данное выражение поэтому верно для невырожденной матрицы 𝐴.

        Мы до сих пор не знаем, данное утверждение истинно или ложно, если наша матрица 𝐴 вырождена. Вот когда определитель равен нулю. Итак, записывая наше заявление снова, используя немного другое обозначение для определителя просто для ясности, и теперь предположим, что мы берем определитель каждого из наших выражений, мы можем использовать теорему что для 𝑛-by-𝑛 матриц 𝐴 и 𝐵 определитель произведения 𝐴𝐵 равен к определителю матрицы 𝐴, умноженному на определитель матрицы 𝐵. Тогда наше первое выражение определитель 𝐴, умноженный на определитель сопряженного 𝐴. И аналогично для нашего второго выражение, а справа оставляем наше выражение как есть.

        Теперь, если 𝐴 — сингулярная матрица, то обратного не существует и определитель равен нулю. Тогда в наших первых двух выражениях мы умножаем на ноль. И все, что умножается на ноль, равен нулю. С правой стороны мы умножение единичной матрицы на ноль, что дает нам нулевую матрицу. И определитель нуля матрица равна нулю. И так левая рука, середина член, а правый член равны нулю. Это означает, что для единичного матрица, где определитель равен нулю, то данное утверждение также истинный. Следовательно, 𝐴, умноженное на сопряженное 𝐴 равен присоединенному 𝐴, умноженному на 𝐴 равен определителю 𝐴, умноженному на 𝐼 верно для любой квадратной матрицы 𝐴 порядка 𝑛.

        сопряженный оператор умножения в коммутативной алгебре

        спросил

        Изменено 6 лет, 4 месяца назад

        Просмотрено 1к раз

        $\begingroup$

        Дэн Попеску задал мне следующий вопрос, и так как я не эксперт, то кидаю его вопрос в МО.

        Предположим, что $A$ — конечномерное векторное пространство над упорядоченным полем $k$ с $\operatorname{char}(k) = 0$, снабженное:

        • коммутативно-ассоциативным $k$-линейным умножением $\цирк\\,$;
        • положительно определенный скалярный продукт $\langle \cdot , \cdot \rangle \\,$.

        Для каждого $a \in A$ пусть $L_a \colon A \to A \colon x \mapsto a \circ x$. Это линейный оператор на $A$; рассмотрим сопряженный оператор относительно данный внутренний продукт: 9* у$.

        Вопрос Попеску следующий:

        Есть ли имя для этой операции $\star$? Изучалось ли это в литературе? Известно ли что-нибудь об алгебраических свойствах или тождествах, включающих $\star$ (возможно, также включающих другие данные $\circ$ и $\langle \cdot , \cdot \rangle$)?

        • ак.коммутативная-алгебра
        • ра.кольца-и-алгебры
        • линейная-алгебра
        • внутренний продукт
        • ссылка-запрос

        $\endgroup$

        $\begingroup$

        В случае $k=\mathbb{C}$ вы описываете конечномерную H*-алгебру . \ast$ единственна, или, другими словами, $\ast$ является инволюцией. Каждая H*-алгебра есть прямая сумма собственной алгебры и алгебры, в которой $a \circ b=0$ для всех $a$ и $b$.

        Существует изящная структурная теорема Уоррена Эмброуза (см. http://www.jstor.org/stable/1990182), показывающая, что правильные H*-алгебры всегда являются прямыми суммами полных матричных алгебр. В частности, коммутативные H*-алгебры являются прямыми суммами 1-мерных алгебр и, следовательно, точно соответствуют ортогональному базису их несущего пространства!

        Не знаю, как насчет других полей $k$, но доказательство Амброуза в основном сводится к тщательному анализу идемпотентов, который можно повторить для других полей $k$. 9*$ принадлежать $L_A$ означает, что $A$ замкнута относительно присоединенной операции, т. е. является алгеброй фон Неймана.

        $\endgroup$

        3

        $\begingroup$

        Эта операция широко изучался для нужд (взвешенных) теория автоматов.

Y ln x 2 cos x: Mathway | Популярные задачи

2
Функция — Квадрат x
ctg(x)
Функция — Котангенс от x
arcctg(x)
Функция — Арккотангенс от x
arcctgh(x)
Функция — Гиперболический арккотангенс от x
tg(x)
Функция — Тангенс от x
tgh(x)
Функция — Тангенс гиперболический от x
cbrt(x)
Функция — кубический корень из x
gamma(x)
Гамма-функция
LambertW(x)
Функция Ламберта
x! или factorial(x)
Факториал от x
DiracDelta(x)
Дельта-функция Дирака
Heaviside(x)
Функция Хевисайда

Интегральные функции:

Si(x)
Интегральный синус от x
Ci(x)
Интегральный косинус от x
Shi(x)
Интегральный гиперболический синус от x
Chi(x)
Интегральный гиперболический косинус от x

В выражениях можно применять следующие операции:

Действительные числа
вводить в виде 7. 3
— возведение в степень
x + 7
— сложение
x — 6
— вычитание
15/7
— дробь

Другие функции:

asec(x)
Функция — арксеканс от x
acsc(x)
Функция — арккосеканс от x
sec(x)
Функция — секанс от x
csc(x)
Функция — косеканс от x
floor(x)
Функция — округление x в меньшую сторону (пример floor(4.5)==4.0)
ceiling(x)
Функция — округление x в большую сторону (пример ceiling(4.5)==5.0)
sign(x)
Функция — Знак x
erf(x)
Функция ошибок (или интеграл вероятности)
laplace(x)
Функция Лапласа
asech(x)
Функция — гиперболический арксеканс от x
csch(x)
Функция — гиперболический косеканс от x
sech(x)
Функция — гиперболический секанс от x
acsch(x)
Функция — гиперболический арккосеканс от x

Постоянные:

pi
Число «Пи», которое примерно равно ~3. 14159..
e
Число e — основание натурального логарифма, примерно равно ~2,7183..
i
Комплексная единица
oo
Символ бесконечности — знак для бесконечности

Мэтуэй | Популярные задачи

92) 9(3x) по отношению к x 92+1
1 Найти производную — d/dx бревно натуральное х
2 Оценить интеграл интеграл натурального логарифма x относительно x
3 Найти производную — d/dx
21 Оценить интеграл интеграл от 0 до 1 кубического корня из 1+7x относительно x
22 Найти производную — d/dx грех(2x)
23 Найти производную — d/dx
41 Оценить интеграл интеграл от cos(2x) относительно x
42 Найти производную — d/dx 1/(корень квадратный из х)
43 Оценка интеграла 9бесконечность
45 Найти производную — d/dx х/2
46 Найти производную — d/dx -cos(x)
47 Найти производную — d/dx грех(3x)
68 Оценить интеграл интеграл от sin(x) по x
69 Найти производную — d/dx угловой синус(х)
70 Оценить предел ограничение, когда x приближается к 0 из (sin(x))/x 92 по отношению к х
85 Найти производную — d/dx лог х
86 Найти производную — d/dx арктан(х)
87 Найти производную — d/dx бревно натуральное 5х9{\pi} \log(2 — 2 \cos x) = 0$.

5 умножить на 70: Онлайн калькулятор. Умножение столбиком

Стр. 51 — ГДЗ Математика 4 класс Учебник Моро Часть 2

  1. Главная
  2. ГДЗ
  3. 4 класс
  4. Математика
  5. Моро учебник
  6. Числа, которые больше 1000. Умножение на двузначное и трёхзначное число
  7. org/ListItem»> Страница 51. Часть 2

Вернуться к содержанию учебника

Числа, которые больше 1000. Умножение на двузначное и трёхзначное число

Вопрос

198. Выполни умножение с объяснением.

351 • 18708 • 43050690 • 16801 • 401

Вопрос

199.

6000 — 560 • 65 : 700156 • 8240136 • 21

Вопрос

200. Реши задачи. Сравни задачи, сравни их решения.

1) На двух опытных участках вырастили картофель. Площадь первого участка 200 м2, а второго 300 м2. С первого участка собрали на 1500 кг картофеля меньше, чем со второго. Сколько килограммов картофеля собрали с каждого участка, если с каждого квадратного метра собирали поровну?

2) С двух опытных участков собрали 7500 кг картофеля. Площадь первого участка 200 м2, а второго 300 м2. С каждого квадратного метра собирали картофеля поровну. Сколько килограммов картофеля собрали с каждого участка?

Вопрос

201. Объясни, что показывает каждое выражение, составленное по следующей таблице:

Скорость70 км/ч65 км/ч
Время3 ч3 ч

1) 70 • 3;3) 70 + 65;5) 70 — 65;
2) 65 • 3;4) (70 + 65) • 3;6) (70 — 65) • 3.

Ответ

Вопрос

202. 1) Первый множитель 127, он на 27 больше второго множителя. Найди произведение этих чисел.

2) Делимое 5600, а делитель на 4900 меньше. Найди частное.

Ответ

Вопрос

203. Вырежи квадрат со стороной 12 см. Раздели его перегибанием на четыре равных треугольника и найди площадь каждого из них.

Ответ

Вопрос

204. Вырази:

1) в метрах: 5 км, 900 дм, 300 см;

2) в килограммах: 9 т, 6 т 5 ц, 800 ц, 4000 г;

3) в секундах: 2 мин, 1 мин 30 с, 2 мин 30 с;

4) в квадратных метрах: 300 дм2, 80000 см2, 9 км2.

Ответ

Вопрос

 

4098 + 420 • 28 : 60709 • 1952070 • 14

Вопрос

Реши. Найди лишнее уравнение.

Ответ

Вопрос

Ответ

Вернуться к содержанию учебника


Рацион питания для набора мышечной массы

Правильное питание для набора мышечной массы – основа здорового роста спортсмена, обретения красивого и рельефного тела. Вы не можете позволить себе халатный подход к составлению рациона: без понимания базовых правил и потребления нужных нутриентов тренировки не только не приведут к цели, но и могут дать обратный эффект.

Идеальный вариант – обратиться к диетологу и составить индивидуальный план питания. Это не попытка продать дополнительные услуги: усредненные данные не всегда на 100% подходят спортсмену с быстрым или замедленным метаболизмом, со специфической анатомией и хроническими заболеваниями. Диетолог принимает во внимание множество факторов, а не только ваш вес. Именно поэтому он может назначить дополнительные анализы и исследования.

Если обращение к врачу не ваш вариант – учите матчасть, консультируйтесь и разбирайтесь. Только так вы добьетесь результатов в адекватные сроки. В этой статье мы собрали все самое важное про питание для набора массы у мужчин и у женщин. Но также рекомендуем разобраться в деталях и сделать акцент на индивидуальных особенностях вашего организма.

Рекомендации для роста мышц

При наборе мышечной массы важно не допустить стремительного увеличения жировой прослойки и доминирующего разрушения белка. Для этого следуйте базовым правилам и вносите изменения не только в рацион, но и в тренировочный процесс. Об этом рассказывали в одной из наших статей.

Универсальные рекомендации:

  1. Углеводы лучше «уничтожать» во время тренировок. В период набора массы вы должны потреблять больше калорий. Важно, чтобы они не уходили в жир, поэтому коктейли с высоким содержанием углеводов лучше принимать за 1. 5-2 часа до и после интенсивной тренировки.
  2. Контролируйте прием триглицеридов. Если вы исключаете жиры животного происхождения из рациона, то это неизбежно снижает выработку тестостерона. Параллельно падает выносливость и работоспособность, повышается объем вредного холестерола, перестают усваиваться витамины и микроэлементы. Поэтому спортсмену рекомендуется принимать до 100 г триглицеридов животного происхождения, исключая при этом «вредные» и чрезмерно жирные продукты вроде снеков.
  3. Не переусердствуйте с кардионагрузками. В противном случае они могут начать «поедать» ваши мышцы. Чтобы поддерживать нормальную работу сердечно-сосудистой системы, достаточно пары получасовых пробежек в неделю. Основные тренировки при этом должны занимать не более 60 минут. Число подходов для каждого упражнения лучше снизить до 5, количество повторений – до 12.
  4. Сбалансируйте рацион. Универсальное соотношение БЖУ в период набора мышечной массы – это до 20% жиров, порядка 30% белков и до 60% углеводов. При этом жиры – полиненасыщенные, углеводы – сложные. Минимум 3 приема пищи, потребление углеводов относительно тренировок – за 1.5-2 часа до и после.
  5. Потребляйте достаточно жидкости. При наборе веса ее объем должен составлять порядка 3 литров в сутки. В противном случае вы рискуете получить обезвоживание и замедлить рост мышц.

Еще одно обязательное условие – здоровый сон и правильное чередование интенсивных нагрузок с отдыхом. Нельзя спать меньше 8 часов в сутки. Помните, что набор мышечной массы происходит не во время тренировок, а в период отдыха. Особое внимание здесь стоит обращать на растягивание мышц. Если вы мало спите, то их рост может не сопровождаться увеличением объема. Не забывайте также менять тренировочный процесс, чтобы избегать привыкания организма к нагрузкам и ограждать себя от стресса, который повышает уровень кортизола.

Значение воды и правильного питания для атлета

Правильное питание для набора массы – залог качественного восстановления и скорейшего достижения результатов. И наоборот, ошибки при формировании рациона не дают атлету прийти к цели. Если вы правильно питаетесь, то мышцы будут расти быстрее, тело всегда будет находиться в тонусе. Вы также сможете увеличивать нагрузки и организовывать разнообразный тренировочный процесс со сменой планов.

Другие преимущества правильного питания:

  • рост силовых показателей;
  • повышение выносливости и работоспособности;
  • мощный заряд энергии;
  • оптимизация образования жировой ткани;
  • нормализация уровня гликогена в мышцах.

Правильное питание также предотвращает быстрое разрушение белка, нормализует обменные процессы. Благодаря корректному рациону, вы сможете найти силы для частых тренировок и увеличить нагрузки, параллельно вырастут и силовые показатели. Не забывайте и о значении водно-солевого баланса – потребляйте порядка 3 литров, корректируя объем в зависимости от температуры воздуха, потоотделения и других сопутствующих факторов.

Суточная норма калорий

С количеством калорий все просто. Сложнее с их качеством, но об этом далее. Объем потребляемых калорий проще всего рассчитать по формуле Лайла Макдональда, также можно использовать калькуляторы из интернета (как правило, они базируются на этой же формуле).

Формула Лайла Макдональда:

Вес * К = Суточная норма калорий.

Здесь вес – это вес вашего стройного тела за вычетом % жира, а К – количество калорий из расчета на 1 кг. Последний показатель зависит от пола отлета и скорости метаболизма.

Сколько калорий нужно потреблять на 1 кг веса:

  1. Женщинам с медленным метаболизмом – 31 г.
  2. Женщинам с быстрым метаболизмом – 33 г.
  3. Мужчинам с медленным метаболизмом – 33 г.
  4. Мужчинам с быстрым метаболизмом – 35 г.

Предположим, что вы мужчина с быстрым обменом веществ. Ваш вес – 100 кг, объем жира – 20%. Соответственно, вес вашего стройного тела – 20 кг. Применяем формулу и получаем: 80 * 35 = 2800 ккал. Это и есть ваша суточная потребность в калориях. Но нам необходимо умножить ее на поправочный коэффициент запаса энергии 1. 2, так как вы хотите набрать массу. Значит, потреблять придется 3360 ккал в сутки. Тогда вы сможете рассчитывать на наращивание до 2-2.5 кг мышечной массы за месяц.

Если вы соблюдаете правила набора веса, но он не увеличивается, то вы недобираете калорий. Увеличьте их на 450-500 в сутки. Если набираете более 3 кг за месяц, то вы наращиваете и жировую массу – уменьшайте суточную калорийность рациона 350-400 г. Постоянно анализируйте ситуацию и корректируйте рацион в зависимости от показателей. Если вам не хватает опыта и знаний для объективной оценки текущей ситуации, то рекомендуем обратиться к вашему фитнес-тренеру.

Примерная суточная калорийность рациона для набора массы

Вес тела Суточная калорийность, мужчины Суточная калорийность, женщины
50 кг 2100 1860
55 кг 2310 2046
60 кг 2520 2232
65 кг 2730 2418
70 кг 2940 2604
75 кг 3150 2790
80 кг 3360 2976
85 кг 3570 3162
90 кг 3780 3348
95 кг 3990 3534

Суточная норма белка

Белок – главный материал для строительства мышц. Поэтому питание для набора массы основывается именно на нем. При регулярных силовых тренировках важно потреблять до 2 г белка на каждый килограмм веса. Но каждую ситуацию стоит рассматривать индивидуально. Нередки случаи, когда оптимальным будет потребление 2.5 г белка на каждый килограмм веса и даже его увеличение до 3 г. Однако повышать его долю в рационе стоит только в ситуациях, когда набора массы не происходит и все прочие факторы вы исключили.

ТОП-5 продуктов с самым высоким содержанием белка на 100 г:

  1. Сыр – до 29 г.
  2. Куриная грудка – до 26.8 г.
  3. Арахис – 26.3 г.
  4. Чечевица – 24.8 г.
  5. Тунец – 22.7 г.

Не забываем о БЖУ продуктов и о типе белков. Очевидно, что доля сыра в вашем рационе не должна превышать объем куриного мяса, хотя белка в нем и больше. Не забывайте о других нутриентах и их усвояемости. Не стремитесь питаться исключительно белком – не забывайте о сложных углеводах, необходимых для энергетического баланса, и о жирах животного происхождения, которые влияют на выработку тестостерона. Также старайтесь сочетать разные протеины. Оптимальный вариант – сделать ставку на белок животного происхождения, но регулярно добавлять к нему растительный.

Важно: не стоит безмерно увеличивать долю белка в рационе, это может привести к нарушению функций внутренних органов и центральной нервной системы, а также привести к серьезным проблемам с пищеварением. Старайтесь также разделить потребление белка на 4 приема пищи, чтобы дать мышцам подпитку в течение всего дня.

Суточная норма жиров

Здесь особенно важно обратить внимание на качество, а не на количество, а также на правильное употребление. Питание для набора массы для мужчин обязательно должно включать в себя триглицериды, так как они влияют на выработку тестостерона. Женщинам тоже не стоит минимизировать их. При этом важно подбирать правильные продукты.

Жиры могут быть:

  • мононенасыщенными и полиненасыщенными – то, что нам нужно;
  • насыщенными – они нам не нужны.

Включите в рацион курицу, оливки, арахис и авокадо. В них много Омега-9 жирных кислот, которые нормализуют уровень сахара, укрепляют сердечно-сосудистую систему, помогают избегать скачков артериального давления и стабилизируют обмен веществ. Не забудьте и об Омега-3,6 жирных кислотах – ими богат рыбий жир, подсолнечное и соевое масла, многие орехи и другие продукты. А вот от снеков, красного мяса, кондитерских изделий и других источников вредных насыщенных жиров лучше отказаться.

Суточная норма углеводов

Углеводы – важнейший источник энергии. Рацион питания для набора массы должен включать в себя порядка 5 г качественных углеводов на 1 кг тела в сутки. Они регулируют уровень инсулина и гормональную систему в целом, помогают восстанавливаться после тренировок, отвечают за транспортировку питательных веществ к мышечной ткани. При дефиците углеводов вы утратите работоспособность, будете чувствовать слабость и вялость.

При этом важно разделять углеводы по скорости их расщепления:

  1. Простые – их стоит употреблять за час до и после тренировок.
  2. Сложные – подходят для употребления за 2 часа до и после нагрузок.

Быстрые углеводы можно полностью исключить из рациона – это питание для набора веса, а нас интересует только мышечная масса и сохранение здоровья в целом. Поэтому никаких тортов, пирожных, печенья, макарон, картофеля, газировки, сладких фруктов и других вредных для атлета, да и для далекого от спорта человека продуктов. Сделайте ставку на рис, свежие овощи без повышенного содержания крахмала, орехи, молочные продукты и сыры.

Суточная норма витаминов и минералов

Если мы строим здоровое тело, то должны позаботиться и о достаточном потреблении витаминов и микроэлементов. Спортсменам их требуется значительно больше, чем людям, ведущим малоподвижный и сидячий образ жизни. Лучше всего потреблять витамины и микроэлементы из натуральных продуктов.

Суточная норма витаминов и микроэлементов

Витамин или минерал Суточная норма Примеры источников
А 1200 мкг рыбий жир
В1 10-20 мг соя
В2 10-20 мг яйца
В6 20 мг шпинат
В7 200 мкг яичный желток
В9 600 мкг свежая зелень
В12 5 мкг морепродукты
С 200 мг брокколи
D 20 мкг творог
Е 100 мг растительные масла
F 3 гр лосось
Кальций 1000 мг молочные продукты
Калий 4700 мг бобовые
Фосфор 700 мг рыба
Магний 420 мг бобовые
Цинк 11 мг молоко

Меню для набора мышечной массы

Рацион питания для набора мышечной массы должен состоять из следующих продуктов:

  • Мясо
  • Рыба
  • Яйца
  • Крупы (Греча, рис, овсянка, пшеница и др. )
  • Макароны из твердых сортов пшеницы
  • Овощи
  • Ягоды и фрукты
  • Мюсли
  • Орехи
  • Жиры растительного происхождения

Спортсменам рекомендуется составлять рацион в соответствии с базовыми рекомендациями и придерживаться оптимального соотношения БЖУ. Лучше всего практиковать 5-разовое питание и потреблять пищу примерно 1 раз в 3 часа. Также стоит составлять меню с учетом усвояемости и особенностей переработки продуктов. Так углеводы стоит употреблять за час до и после тренировки, а белки – в течение всего дня.

Пример меню спортсмена на день для увеличения мускулатуры

Если вы планируете питаться по универсальному плану, то наверняка будете испытывать дискомфорт. Не старайтесь копировать их точь-в-точь – подбирайте продукты с близкой пищевой ценностью, которые вы действительно любите.

Примерный рацион для набора мышечной массы на день

Завтрак:

  • 1 банан;
  • омлет из 2 яиц;
  • 1 ломтик черного хлеба;
  • черный кофе или чай.

Первый перекус:

  • чай;
  • 200 г нежирного творога;
  • грейпфрут.

Второй перекус:

  • гейнер;
  • 40 г орехов.

Обед:

  • 100 г отварного риса;
  • нежирное молоко;
  • 150 г вареной индейки;
  • 2 отварных яйца.

Ужин:

  • 100 г брокколи;
  • 200 г отварной куриной грудки;
  • 100 г риса.

Обратите внимание: первый и второй перекус должны быть за 1.5-2 часа до и после тренировки.

Пример питания спортсмена-вегетарианца на увеличение мускулатуры

При наборе мышечной массы вегетарианцам особенно важно соблюдать требований к сбалансированному питанию. Нельзя нагружать организм протеинами, но их оптимальное количество не должно быть меньше 2 г на каждый килограмм сухого веса тела.

Примерное меню на день для спортсмена-вегетарианца

Завтрак:

  • зеленый чай;
  • 100 г сыра тофу;
  • 3 злаковых хлебца или 2 ломтика черного хлеба.

Первый перекус:

  • 150 г сырников;
  • 40 г орехов.

Второй перекус:

  • вегетарианский протеиновый коктейль;
  • 40 г семечек подсолнечника.

Обед:

  • 250 мл овощного супа;
  • 100 г тушеных брокколи;
  • 100 г темпе;
  • 150 г соевого мяса.

Ужин:

  • 100 г риса;
  • 150 г отварных брокколи;
  • 50 г тофу;
  • 1/2 авокадо.

Важно: первый и второй перекус нужно делать за 1.5-2 часа до и после тренировки. При этом для вегетарианцев лучше всего подходят высокоинтенсивные занятия продолжительностью не более 30 минут – это позволяет расходовать меньше протеинов.

Что нельзя есть при наборе мышечной массы

Если вы хотите оптимизировать питание для набора мышечной массы, а не жировой, то ограничивать себя в некоторых продуктах все же стоит. В противном случае все пойдет не по плану. За счет резкого увеличения калорийности рациона вес действительно будет расти, но преимущественно из-за повышения доли подкожного жира. В дальнейшем его будет крайне проблематично сбросить. Поэтому сделайте ставку на полезные продукты сразу, чтобы добиться скорейших результатов и не навредить здоровью.

Что исключить из рациона, чтобы этого не допустить:

  • сахар, выпечку и кондитерскую продукцию;
  • алкогольные напитки, особенно пиво;
  • колбасные изделия и магазинные сосиски;
  • фастфуд;
  • крахмалистые, сладкие овощи и фрукты.

Сделайте ставку на натуральные продукты. Если хотите сосисок – приготовьте их сами, не покупайте в магазине. Или, по крайней мере, следите за надписями на упаковке. Откажитесь от вредных жиров. Оставьте привычку добавлять молоко и сахар в кофе или чай. Эти мелочи помогут нормализовать рацион и не допустят быстрого увеличения доли подкожного жира при наборе мышечной массы.

Число в 25 раз больше другого числа Если их сумма равна 70 Найдите числа…

Перейти к

  • Линейные уравнения с одной переменной. Упражнение 7А.
  • Линейные уравнения с одной переменной. Упражнение 7Б.
  • Линейные уравнения с одной переменной. Упражнение 7C.
  • Целые числа
  • Фракции
  • Десятичные
  • Рациональное число
  • Экспоненты
  • Алгебраические выражения
  • Линейные уравнения с одной переменной
  • Соотношение и пропорция
  • Унитарный метод
  • Процент
  • Доходы и расходы
  • Простой интерес
  • Линии и углы
  • Свойства параллельной линии
  • Свойства треугольников
  • Конгруэнтность
  • Конструкции
  • Отражение и вращательная симметрия
  • Трехмерные формы
  • Измерение
  • Сбор и организация данных (среднее, медиана, мода)
  • Гистограммы
  • Вероятность

Главная > Решения RS Aggarwal Класс 7 Математика > Глава 7. Линейные уравнения с одной переменной > Линейные уравнения с одной переменной. Упражнение 7Б. > Вопрос 5

Вопрос 5 Линейные уравнения с одной переменной Упражнение 7B

Число в (2/5) раз больше другого числа. Если их сумма равна 70. Найдите числа.

Ответ:

Решение:-

Пусть искомое число равно x.

Тогда

= x + (2x/5) = 70

= (5x + 2x)/5 = 70

= (7x/5) = 70

Умножение обеих сторон на (5/7)

= (7x/5) × (5/7) = 70 × (5/7)

= x = 50

Другое число равно (2x/5)

= (2 × 50)/5

= 100/5

= 20

∴необходимы числа 50 и 20

Связанные вопросы

Три раза число при увеличении на 5 дает 44. Найдите число.

Четыре, добавленные к удвоенному числу, дают (26/5). Найдите дробь.

Число, умноженное на 4, превышает само себя на 45. Найдите число.

(2/3) числа меньше исходного числа на 20. Найдите число.

Пятая часть числа при увеличении на 5 равна его четвертой части, уменьшенной на 5. Найдите число…

Найдите три последовательных положительных четных числа, сумма которых равна 90.

Фейсбук WhatsApp

Копировать ссылку

Было ли это полезно?

Exercises

Linear Equations in one variable Exercise 7A

Linear Equations in one variable Exercise 7B

Linear Equations in one variable Exercise 7C

Chapters

Integers

Fractions

Decimals

Rational Numbers

Exponents

Алгебраические выражения

Линейные уравнения в одной переменной

Соотношение и доля

Unitary Method

Процент

Прибыль и потеря

Простой процент

Линии и углы

Свойства параллельной линии

.

Отражение и вращательная симметрия

Трехмерные формы

Измерение

Сбор и организация данных (среднее, медиана, мода)

Гистограммы

Вероятность

Курсы

Быстрые ссылки

Условия и политика

Условия и политика

2022 © Quality Tutorials Pvt Ltd Все права защищены поэтому крайне важно поддерживать прочные отношения с вашими прошлыми и существующими клиентами, чтобы они оставались лояльными.

И все же многие малые предприятия тратят большую часть своих маркетинговых долларов на поиск новых клиентов, а не на поддержку уже имеющихся. Это постоянная борьба, когда вы думаете о своих бизнес-целях.

Бесплатная загрузка: Долгосрочные отношения — ключ к успеху вашего бизнеса. Узнайте, как превратить холодные лиды в долгосрочные отношения.

Загрузите бесплатное руководство «От лида к лояльности».

Не убежден? Для многих владельцев бизнеса неопровержимые факты стоят больше, чем анекдоты и предположения. Прочтите приведенную ниже статистику удержания и привлечения клиентов, чтобы увидеть разницу между ними.

Маркетинговая статистика по привлечению и удержанию клиентов

  • Привлечение нового клиента может стоить в пять раз больше, чем удержание существующего клиента.
  • Увеличение удержания клиентов на 5% может увеличить прибыль на 25-95%.
  • Вероятность успеха продажи клиенту, который у вас уже есть, составляет 60-70%, а вероятность успеха продажи новому клиенту составляет 5-20%.
  • Одно агентство по работе с клиентами обнаружило, что лояльные клиенты в 5 раз чаще совершают повторные покупки, в 5 раз чаще прощают, в 4 раза чаще рекомендуют и в 7 раз чаще пробуют новое предложение.
  • американских компаний теряют 136,8 млрд долларов в год из-за того, что переключения потребителей можно было избежать.
  • American Express обнаружила, что 33% клиентов рассмотрят возможность перехода в другую компанию после всего лишь одного случая плохого обслуживания клиентов.

Данные показывают, что удержание клиентов так же важно, как и привлечение новых, если не важнее. Так почему же так много предприятий сосредотачиваются в основном на привлечении клиентов и пренебрегают маркетингом, который поддерживает лояльность клиентов?

Маркетинговые возможности удержания клиентов

Здесь есть возможность больше сосредоточиться на существующих клиентах, а не на поиске новых. Изменение стратегии поможет вам получить больше отдачи от вашего маркетингового бюджета и тратить меньше времени на поиск каждого потенциального лида, который вы можете найти, независимо от того, подходят ли они для вашего бизнеса.

Решение, которое доказало свою эффективность, заключается в том, чтобы оставаться на связи на регулярной основе. Для некоторых предприятий с очень небольшим количеством ценных клиентов это может быть выполнено на индивидуальной основе. Но для большинства предприятий поддерживать такой личный контакт с каждым клиентом просто нереально. Вот где на помощь приходит цифровой маркетинг.

Просто оставаясь на связи с помощью контент-маркетинга, маркетинга по электронной почте и маркетинга в социальных сетях, вы можете добиться успеха несколькими способами:

  1. Оставайтесь на связи со своей текущей клиентской базой.
  2. Напоминание клиентам о предстоящих событиях и сезонных советах.
  3. Поддержание профессионального внешнего вида на всем пути к почтовому ящику и далее.
  4. Воспользуйтесь возможностью связаться с клиентами, с которыми вы не разговаривали годами.
  5. Получение большего количества рефералов.

Мы составили очень информативное (и бесплатное!) руководство о том, как долгосрочные отношения и лояльные клиенты приносят пользу вашему бизнесу. Он включает в себя полезные шаблоны электронной почты, способы улучшить общение и последующие советы, которые помогут вам выделиться и оставаться на связи.

Маркетинг удержания клиентов необходим для успеха и роста вашего бизнеса.

Матрица обратная примеры: Найти матрицу обратную матрице — примеры, решения

Примеры использования функции МОБР в Excel матрицах

Функция МОБР – это вычислительное определение матрицы. Она возвращает обратную матрицу для матрицы, хранящейся в массиве. Обратные матрицы, как и определители, обычно используются для решения систем уравнений с несколькими неизвестными. Некоторые квадратные матрицы не могут быть обращены: в таких случаях функция МОБР возвращает значение ошибки #ЧИСЛО!. Определитель такой матрицы равен = 0.

Описание использования функции МОБР в Excel

Как использовать функцию МОБР в Excel рассмотрим ниже на примерах. Но сначала ознакомимся как устроена данная функции.

Аргумент функции МОБР – это массив. Он может быть задан как диапазон ячеек, например A1:C3 как массив констант, например {1;2;3:4;5;6:7;8;9} или как имя диапазона или массива. В случае, если хотя бы одна из ячеек массива пуста или содержит текст, функция возвращает значение ошибки #ЗНАЧ!

Массив должен иметь равное количество строк и столбцов. В случае, если они не равны, то функция МОБР также возвращает значение ошибки #ЗНАЧ!

Формулы, которые возвращают массивы, должны быть введены как формулы массива.

Для вывода обратного массива необходимо после выбора диапазона для данной функции нажать комбинацию клавиш Ctrl+Shift+Enter, а не просто Enter.

Функция МОБР производит вычисления с точностью до 16 значащих цифр, что может привести к незначительным ошибкам округления. Рассмотрим применение данной функции на конкретных примерах.



Поиск обратной матрицы в Excel с помощью функции МОБР

Пример 1. Используя пакет Excel, найти обратную матрицу для матрицы, приведенной в таблице 1.

Таблица 1:

Исходные данные
12
34

Для решения данной задачи открывает пакет Excel, в произвольной ячейке вводим исходные данные, дальше выбираем функцию МОБР. В качестве массива выбираем диапазон с введенными данными и контролируем полученный результат. В Excel общий вид функции выглядит следующим образом:

Рисунок 1 – Результат расчета.

Как найти валовый показатель по матрице взаимосвязей?

Пример 2. Связь между тремя отраслями представлена матрицей прямых затрат А. Спрос (конечный продукт) задан вектором X. Найти валовой выпуск продукции отраслей Х. Описать используемые формулы, представить распечатку со значениями и с формулами.

Исходные данные приведены на рисунке 2:

Рисунок 2 – Исходные данные.

Данная задача связана с определением объема производства каждой из N отраслей, чтобы удовлетворить все потребности в продукции данной отрасли. При этом каждая отрасль выступает и как производитель некоторой продукции и как потребитель своей и произведенной другими отраслями продукции. Задача межотраслевого баланса – отыскание такого вектора валового выпуска X, который при известной матрице прямых затрат обеспечивает заданный вектор конечного продукта Y.

Матричное решение данной задачи:

X = ( E — A )-1 Y

где Е – единична матрица.

Для решения задачи в примере используем следующие 4 функции для работы с матрицами в Excel:

  1. МОБР – нахождение обратной матрицы.
  2. МУМНОЖ – умножение матриц.
  3. МОПРЕД – нахождение определителя матрицы.
  4. МЕДИН – нахождение единичной матрицы.

Результаты приведены на рисунке 3:

Рисунок 3 – Результат вычислений.

Функция МОБР возвращает ошибку #ЧИСЛО!

Пример 3. Найти обратную матрицу для матрицы, приведенной в таблице 2.

Таблица 2:

Исходные данные
124
248
4816

Результат решения приведен на рисунке 4. Видно, что определитель данной матрицы равен 0, поэтому функция МОБР выводит в результате значение #ЧИСЛО!.

Рисунок 4 – Окончательный результат.

Обратная матрица для чайников подробные примеры решений. Обратная матрица

Определение пределов последовательности и функции, свойства пределов, первый и второй замечательные пределы, примеры.

Постоянное число а называется пределом последовательности {x n }, если для любого сколь угодно малого положительного числа ε > 0 существует номер N, что все значения x n , у которых n>N, удовлетворяют неравенству

Записывают это следующим образом: или x n → a.

Неравенство (6.1) равносильно двойному неравенству

a — ε x n , начиная с некоторого номера n>N, лежат внутри интервала (a-ε , a+ε), т.е. попадают в какую угодно малую ε-окрестность точки а .

Последовательность, имеющая предел, называется сходящейся , в противном случае — расходящейся .

Понятие предел функции является обобщением понятия предел последовательности, так как предел последовательности можно рассматривать как предел функции x n = f(n) целочисленного аргумента n .

Пусть дана функция f(x) и пусть a предельная точка области определения этой функции D(f), т.е. такая точка, любая окрестность которой содержит точки множества D(f), отличные от a . Точка a может принадлежать множеству D(f), а может и не принадлежать ему.

Определение 1. Постоянное число А называется предел функции f(x) при x→ a, если для всякой последовательности {x n } значений аргумента, стремящейся к а , соответствующие им последовательности {f(x n)} имеют один и тот же предел А.

Это определение называют определением предела функции по Гейне, или “на языке последовательностей ”.

Определение 2 . Постоянное число А называется предел функции f(x) при x→a, если, задав произвольное, как угодно малое положительное число ε, можно найти такое δ >0 (зависящее от ε), что для всех x , лежащих в ε-окрестности числа а , т.е. для x , удовлетворяющих неравенству
0

Это определение называют определением предел функции по Коши, или “на языке ε — δ «

Определения 1 и 2 равносильны. Если функция f(x) при x → a имеет предел , равный А, это записывается в виде

В том случае, если последовательность {f(x n)} неограниченно возрастает (или убывает) при любом способе приближения x к своему пределу а , то будем говорить, что функция f(x) имеет бесконечный предел, и записывать это в виде:

Переменная величина (т.е. последовательность или функция), предел которой равен нулю, называется бесконечно малой величиной.

Переменная величина, предел которой равен бесконечности, называется бесконечно большой величиной .

Чтобы найти предел на практике пользуются следующими теоремами.

Теорема 1 . Если существует каждый предел

(6.4)

(6.5)

(6.6)

Замечание . Выражения вида 0/0, ∞/∞, ∞-∞ 0*∞ являются неопределенными, например, отношение двух бесконечно малых или бесконечно больших величин, и найти предел такого вида носит название “раскрытие неопределенностей”.

Теорема 2.

т.е. можно переходить к пределу в основании степени при постоянном показателе, в частности,

Теорема 3.

(6.11)

где e » 2.7 — основание натурального логарифма. Формулы (6.10) и (6.11) носят название первый замечательного предело и второй замечательный предел.

Используются на практике и следствия формулы (6.11):

(6.12)

(6.13)

(6.14)

в частности предел,

Eсли x → a и при этом x > a, то пишут x →a + 0. Если, в частности, a = 0, то вместо символа 0+0 пишут +0. Аналогично если x→a и при этом x и называются соответственно предел справа и предел слева функции f(x) в точке а . Чтобы существовал предел функции f(x) при x→ a необходимо и достаточно, чтобы . Функция f(x) называется непрерывной в точке x 0 , если предел

(6.15)

Условие (6.15) можно переписать в виде:

то есть возможен предельный переход под знаком функции, если она непрерывна в данной точке.

Если равенство (6.15) нарушено, то говорят, что при x = x o функция f(x) имеет разрыв. Рассмотрим функцию y = 1/x. Областью определения этой функции является множество R , кроме x = 0. Точка x = 0 является предельной точкой множества D(f), поскольку в любой ее окрестности, т.е. в любом открытом интервале, содержащем точку 0, есть точки из D(f), но она сама не принадлежит этому множеству. Значение f(x o)= f(0) не определено, поэтому в точке x o = 0 функция имеет разрыв.

Функция f(x) называется непрерывной справа в точке x o , если предел

и непрерывной слева в точке x o, если предел

Непрерывность функции в точке x o равносильна ее непрерывности в этой точке одновременно и справа и слева.

Для того, чтобы функция была непрерывна в точке x o , например, справа, необходимо, во-первых, чтобы существовал конечный предел , а во-вторых, чтобы этот предел был равен f(x o). Следовательно, если хотя бы одно из этих двух условий не выполняется, то функция будет иметь разрыв.

1. Если предел существует и не равен f(x o), то говорят, что функция f(x) в точке x o имеет разрыв первого рода, или скачок .

2. Если предел равен +∞ или -∞ или не существует, то говорят, что в точке x o функция имеет разрыв второго рода .

Например, функция y = ctg x при x → +0 имеет предел, равный +∞ , значит, в точке x=0 она имеет разрыв второго рода. Функция y = E(x) (целая часть от x ) в точках с целыми абсциссами имеет разрывы первого рода, или скачки.

Функция, непрерывная в каждой точке промежутка , называется непрерывной в . Непрерывная функция изображается сплошной кривой.

Ко второму замечательному пределу приводят многие задачи, связанные с непрерывным ростом какой-либо величины. К таким задачам, например, относятся: рост вклада по закону сложных процентов, рост населения страны, распад радиоактивного вещества, размножение бактерий и т.п.

Рассмотрим пример Я. И. Перельмана , дающий интерпретацию числа e в задаче о сложных процентах. Число e есть предел . В сбербанках процентные деньги присоединяются к основному капиталу ежегодно. Если присоединение совершается чаще, то капитал растет быстрее, так как в образовании процентов участвует большая сумма. Возьмем чисто теоретический, весьма упрощенный пример. Пусть в банк положено 100 ден. ед. из расчета 100 % годовых. Если процентные деньги будут присоединены к основному капиталу лишь по истечении года, то к этому сроку 100 ден. ед. превратятся в 200 ден.ед. Посмотрим теперь, во что превратятся 100 ден. ед., если процентные деньги присоединять к основному капиталу каждые полгода. По истечении полугодия 100 ден. ед. вырастут в 100 ×1,5 = 150, а еще через полгода — в 150× 1,5 = 225 (ден. ед.). Если присоединение делать каждые 1/3 года, то по истечении года 100 ден. ед. превратятся в 100 × (1 +1/3) 3 ≈ 237 (ден. ед.). Будем учащать сроки присоединения процентных денег до 0,1 года, до 0,01 года, до 0,001 года и т.д. Тогда из 100 ден. ед. спустя год получится:

100×(1 +1/10) 10 ≈ 259 (ден. ед.),

100×(1+1/100) 100 ≈ 270 (ден. ед.),

100×(1+1/1000) 1000 ≈271 (ден. ед.).

При безграничном сокращении сроков присоединения процентов наращенный капитал не растет беспредельно, а приближается к некоторому пределу, равному приблизительно 271. Более чем в 2,71 раз капитал, положенный под 100% годовых, увеличиться не может, даже если бы наросшие проценты присоединялись к капиталу каждую секунду, потому что предел

Пример 3. 1 . Пользуясь определением предела числовой последовательности, доказать, что последовательность x n =(n-1)/n имеет предел, равный 1.

Решение. Нам надо доказать, что, какое бы ε > 0 мы ни взяли, для него найдется натуральное число N, такое, что для всех n > N имеет место неравенство |x n -1|

Возьмем любое ε > 0. Так как x n -1 =(n+1)/n — 1= 1/n, то для отыскания N достаточно решить неравенство 1/n1/ε и, следовательно, за N можно принять целую часть от 1/ε N = E(1/ε). Мы тем самым доказали, что предел .

Пример 3.2. Найти предел последовательности, заданной общим членом .

Решение. Применим теорему предел суммы и найдем предел каждого слагаемого. При n → ∞ числитель и знаменатель каждого слагаемого стремится к бесконечности, и мы не можем непосредственно применить теорему предел частного. Поэтому сначала преобразуем x n , разделив числитель и знаменатель первого слагаемого на n 2 , а второго на n . Затем, применяя теорему предел частного и предел суммы, найдем:

Пример 3.3 . . Найти .

Решение.

Здесь мы воспользовались теоремой о пределе степени: предел степени равен степени от предела основания.

Пример 3.4 . Найти ().

Решение. Применять теорему предел разности нельзя, поскольку имеем неопределенность вида ∞-∞. Преобразуем формулу общего члена:

Пример 3.5 . Дана функция f(x)=2 1/x . Доказать, что предел не существует.

Решение. Воспользуемся определением 1 предела функции через последовательность. Возьмем последовательность { x n }, сходящуюся к 0, т.е. Покажем, что величина f(x n)= для разных последовательностей ведет себя по-разному. Пусть x n = 1/n. Очевидно, что , тогда предел Выберем теперь в качестве x n последовательность с общим членом x n = -1/n, также стремящуюся к нулю. Поэтому предел не существует.

Пример 3.6 . Доказать, что предел не существует.

Решение. Пусть x 1 , x 2 ,…, x n ,… — последовательность, для которой
. Как ведет себя последовательность {f(x n)} = {sin x n } при различных x n → ∞

Если x n = p n, то sin x n = sin (p n) = 0 при всех n и предел Если же
x n =2 p n+ p /2, то sin x n = sin(2 p n+ p /2) = sin p /2 = 1 для всех n и следовательно предел . Таким образом, не существует.

Пределы доставляют всем студентам, изучающим математику, немало хлопот. Чтобы решить предел, порой приходится применять массу хитростей и выбирать из множества способов решения именно тот, который подойдет для конкретного примера.

В этой статье мы не поможем вам понять пределы своих возможностей или постичь пределы контроля, но постараемся ответить на вопрос: как понять пределы в высшей математике? Понимание приходит с опытом, поэтому заодно приведем несколько подробных примеров решения пределов с пояснениями.

Понятие предела в математике

Первый вопрос: что это вообще за предел и предел чего? Можно говорить о пределах числовых последовательностей и функций. Нас интересует понятие предела функции, так как именно с ними чаще всего сталкиваются студенты. Но сначала — самое общее определение предела:

Допустим, есть некоторая переменная величина. Если эта величина в процессе изменения неограниченно приближается к определенному числу a , то a – предел этой величины.

Для определенной в некотором интервале функции f(x)=y пределом называется такое число A , к которому стремится функция при х , стремящемся к определенной точке а . Точка а принадлежит интервалу, на котором определена функция.

Звучит громоздко, но записывается очень просто:

Lim — от английского limit — предел.

Существует также геометрическое объяснение определения предела, но здесь мы не будем лезть в теорию, так как нас больше интересует практическая, нежели теоретическая сторона вопроса. Когда мы говорим, что х стремится к какому-то значению, это значит, что переменная не принимает значение числа, но бесконечно близко к нему приближается.

Приведем конкретный пример. Задача — найти предел.

Чтобы решить такой пример, подставим значение x=3 в функцию. Получим:

Кстати, если Вас интересуют базовые операции над матрицами , читайте отдельную статью на эту тему.

В примерах х может стремиться к любому значению. Это может быть любое число или бесконечность. Вот пример, когда х стремится к бесконечности:

Интуитивно понятно, что чем больше число в знаменателе, тем меньшее значение будет принимать функция. Так, при неограниченном росте х значение 1/х будет уменьшаться и приближаться к нулю.

Как видим, чтобы решить предел, нужно просто подставить в функцию значение, к которому стремиться х . Однако это самый простой случай. Часто нахождение предела не так очевидно. В пределах встречаются неопределенности типа 0/0 или бесконечность/бесконечность . Что делать в таких случаях? Прибегать к хитростям!


Неопределенности в пределах

Неопределенность вида бесконечность/бесконечность

Пусть есть предел:

Если мы попробуем в функцию подставить бесконечность, то получим бесконечность как в числителе, так и в знаменателе. Вообще стоит сказать, что в разрешении таких неопределенностей есть определенный элемент искусства: нужно заметить, как можно преобразовать функцию таким образом, чтобы неопределенность ушла. В нашем случае разделим числитель и знаменатель на х в старшей степени. Что получится?

Из уже рассмотренного выше примера мы знаем, что члены, содержащие в знаменателе х, будут стремиться к нулю. Тогда решение предела:

Для раскрытия неопределенностей типа бесконечность/бесконечность делим числитель и знаменатель на х в высшей степени.


Кстати! Для наших читателей сейчас действует скидка 10% на любой вид работы

Еще один вид неопределенностей: 0/0

Как всегда, подстановка в функцию значения х=-1 дает 0 в числителе и знаменателе. Посмотрите чуть внимательнее и Вы заметите, что в числителе у нас квадратное уравнение. Найдем корни и запишем:

Сократим и получим:

Итак, если Вы сталкиваетесь с неопределенностью типа 0/0 – раскладывайте числитель и знаменатель на множители.

Чтобы Вам было проще решать примеры, приведем таблицу с пределами некоторых функций:

Правило Лопиталя в пределах

Еще один мощный способ, позволяющий устранить неопределенности обоих типов. В чем суть метода?

Если в пределе есть неопределенность, берем производную от числителя и знаменателя до тех пор, пока неопределенность не исчезнет.

Наглядно правило Лопиталя выглядит так:

Важный момент : предел, в котором вместо числителя и знаменателя стоят производные от числителя и знаменателя, должен существовать.

А теперь – реальный пример:

Налицо типичная неопределенность 0/0 . Возьмем производные от числителя и знаменателя:

Вуаля, неопределенность устранена быстро и элегантно.

Надеемся, что Вы сможете с пользой применить эту информацию на практике и найти ответ на вопрос «как решать пределы в высшей математике». Если нужно вычислить предел последовательности или предел функции в точке, а времени на эту работу нет от слова «совсем», обратитесь в профессиональный студенческий сервис за быстрым и подробным решением. 2 стремится к нулю.

Обычно переменная величина x стремится к конечному пределу a, причем, x постоянно приближается к a, а величина a постоянна. Это записывают следующим образом: limx =a, при этом, n также может стремиться как к нулю, так и к бесконечности. Существуют бесконечные функции, для них предел стремится к бесконечности. В других случаях, когда, например, функцией замедление хода поезда, можно о пределе, стремящемся к нулю.
У пределов имеется ряд свойств. Как правило, любая функция имеет только один предел. Это главное свойство предела. Другие их свойства перечислены ниже:
* Предел суммы равен сумме пределов:
lim(x+y)=lim x+lim y
* Предел произведения равен произведению пределов:
lim(xy)=lim x*lim y
* Предел частного равен частному от пределов:
lim(x/y)=lim x/lim y
* Постоянный множитель выносят за знак предела:
lim(Cx)=C lim x
Если дана функция 1 /x, в которой x →∞, ее предел равен нулю. Если же x→0, предел такой функции равен ∞.
Для тригонометрических функций имеются исключения из этих правил. Так как функция sin x всегда стремится к единице, когда приближается к нулю, для нее справедливо тождество:
lim sin x/x=1

В ряде задач встречаются функции, при вычислении пределов которых возникает неопределенность — ситуация, при которой предел невозможно вычислить. Единственным выходом из такой ситуации становится применение правила Лопиталя. Существует два вида неопределенностей:
* неопределенность вида 0/0
* неопределенность вида ∞/∞
К примеру, дан предел следующего вида: lim f(x)/l(x), причем, f(x0)=l(x0)=0. В таком случае, возникает неопределенность вида 0/0. Для решения такой задачи обе функции подвергают дифференцированию, после чего находят предел результата. Для неопределенностей вида 0/0 предел равен:
lim f(x)/l(x)=lim f»(x)/l»(x) (при x→0)
Это же правило справедливо и для неопределенностей типа ∞/∞. Но в этом случае справедливо следующее равенство: f(x)=l(x)=∞
С помощью правила Лопиталя можно находить значения любых пределов, в которых фигурируют неопределенности. (n-1)

Определение 1: матрица называется вырожденной, если её определитель равен нулю.

Определение 2: матрица называется невырожденной, если её определитель не равен нулю.

Матрица «A» называется обратной матрицей , если выполняется условие A*A-1 = A-1 *A = E (единичной матрице).

Квадратная матрица обратима только в том случае, когда она является невырожденной.

Схема вычисления обратной матрицы:

1) Вычислить определитель матрицы «A», если A = 0, то обратной матрицы не существует.

2) Найти все алгебраические дополнения матрицы «A».

3) Составить матрицу из алгебраических дополнений (Aij )

4) Транспонировать матрицу из алгебраических дополнений (Aij )T

5) Умножить транспонированную матрицу на число, обратное определителю данной матрицы.

6) Выполнить проверку:

На первый взгляд может показаться, что это сложно, но на самом деле всё очень просто. Все решения основаны на простых арифметических действиях, главное при решении не путаться со знаками «-» и «+», и не терять их.

А теперь давайте вместе с Вами решим практическое задание, вычислив обратную матрицу.

Задание: найти обратную матрицу «A», представленную на картинке ниже:

Решаем всё в точности так, как это указано в план-схеме вычисления обратной матрицы.

1. Первое, что нужно сделать, это найти определитель матрицы «A»:

Пояснение:

Мы упростили наш определитель, воспользовавшись его основными функциями. Во первых, мы прибавили ко 2 и 3 строке элементы первой строки, умноженные на одно число.

Во-вторых, мы поменяли 2 и 3 столбец определителя, и по его свойствам поменяли знак перед ним.

В-третьих, мы вынесли общий множитель (-1) второй строки, тем самым, снова поменяв знак, и он стал положительным. Также мы упростили 3 строку также, как в самом начале примера.

У нас получилась треугольный определитель, у которого элементы ниже диагонали равны нулю, и по 7 свойству он равен произведению элементов диагонали. В итоге мы получили A = 26, следовательно обратная матрица существует.

А11 = 1*(3+1) = 4

А12 = -1*(9+2) = -11

А13 = 1*1 = 1

А21 = -1*(-6) = 6

А22 = 1*(3-0) = 3

А23 = -1*(1+4) = -5

А31 = 1*2 = 2

А32 = -1*(-1) = -1

А33 = 1+(1+6) = 7

3. Следующий шаг — составление матрицы из получившихся дополнений:

5. Умножаем эту матрицу на число, обратное определителю, то есть на 1/26:

6. Ну а теперь нам просто нужно выполнить проверку:

В ходе проверки мы получили единичную матрицу, следовательно, решение было выполнено абсолютно верно.

2 способ вычисления обратной матрицы.

1. Элементарное преобразование матриц

2. Обратная матрица через элементарный преобразователь.

Элементарное преобразование матриц включает:

1. Умножение строки на число, не равное нулю.

2. Прибавление к любой строке другой строки, умноженной на число.

3. Перемена местами строк матрицы.

4. Применяя цепочку элементарных преобразований, получаем другую матрицу.

А-1 = ?

1. (A|E) ~ (E|A-1 )

2. A-1 * A = E

Рассмотрим это на практическом примере с действительными числами.

Задание: Найти обратную матрицу.

Решение:

Выполним проверку:

Небольшое разъяснение по решению:

Сперва мы переставили 1 и 2 строку матрицы, затем умножили первую строку на (-1).

После этого умножили первую строку на (-2) и сложили со второй строкой матрицы. После чего умножили 2 строку на 1/4.

Заключительным этапом преобразований стало умножение второй строки на 2 и прибавлением с первой. В результате слева у нас получилась единичная матрица, следовательно, обратная матрица — это матрица справа.

После проверки мы убедились в правильности решения.

Как вы видите, вычисление обратной матрицы — это очень просто.

В заключении данной лекции хотелось бы также уделить немного времени свойствам такой матрицы. (2+1) = -1.

В результате вы получите матрицу дополнений, теперь транспонируйте ее. Транспонирование — это операция, симметричная относительно главной диагонали матрицы, столбцы и строки меняются местами. Таким образом, вы нашли присоединенную матрицу A*.

Обратная матрица: определение, типы, примеры

  • Автор Ритеш Кумар Гупта
  • Последнее изменение 25-01-2023

Обратная матрица: Матрица — это набор объектов, расположенных в строках и столбцах. Эти элементы известны как матричные элементы. Порядок матрицы определяется как количество строк, деленное на количество столбцов. Например, 2×2, 2×3, 3×2, 3×3, 4×4 и так далее. Только для квадратных матриц с одинаковым количеством строк и столбцов можно определить обратную. Обратная матрица — важный инструмент в математическом мире.

Используется при решении системы линейных уравнений. Обратные матрицы часто используются для шифрования или расшифровки кодов сообщений. Он также используется для изучения электрических цепей, квантовой механики и оптики. Эти матрицы имеют решающее значение для измерения выходной мощности батареи и преобразования электрической энергии в другую полезную энергию с помощью резисторов. При применении законов Кирхгофа для напряжения и тока для решения задач обратные матрицы чрезвычайно важны. Продолжайте читать, чтобы узнать больше.

Прежде чем изучать обратную матрицу, мы должны знать, что такое сопряжение матрицы.

Сопряженная квадратная матрица \(A = {\left[ {{a_{ij}}} \right]_{n \times n}}\) является транспонированной матрицей \({\left[ { {A_{ij}}} \right]_{n \times n}}\) где \({A_{ij}}\) — кофактор элемента \({a_{ij}}\) матрица \(A\) обозначается через \(Adj{\rm{}}A .\)

Пусть,

\(A = \left[ {\begin{array}{*{20}{c} } {{a_{11}}}&{{a_{12}}}&{{a_{13}}}\\ {{a_{21}}}&{{a_{22}}}&{{a_{23}}}\\ {{a_{31}}}&{{a_{32}}}&{{a_{33}}} \end{массив}} \right]\)

Затем \(Adj{\rm{}}A = \) Транспонировать \(\left[ {\begin{array}{*{20}{c}} {{А_{11}}}&{{А_{12}}}&{{А_{13}}}\\ {{А_{21}}}&{{А_{22}}}&{{А_{23}}}\\ {{А_{31}}}&{{А_{32}}}&{{А_{33}}} \end{массив}} \right] = \left[ {\begin{массив}}{*{20}{c}} {{А_{11}}}&{{А_{21}}}&{{А_{31}}}\\ {{А_{12}}}&{{А_{22}}}&{{А_{32}}}\\ {{А_{13}}}&{{А_{23}}}&{{А_{33}}} \end{array}} \right]\)

где \({A_{ij}}\) — это сомножитель \({a_{ij}}\), который вычисляется с использованием соотношения \({A_{ij }} = {( – 1)^{i + j}}{M_{ij}}\), где \({M_{ij}}\) является минором \({a_{ij}}\).

Минором элемента \({a_{ij}}\) определителя является определитель, полученный удалением его i-й строки и j-го столбца, в котором лежит элемент \({a_{ij}}\). Минор элемента \({a_{ij}}\) обозначается через \({M_{ij}}\).

В математике кофактор используется для нахождения обратной и сопряженной матрицы. Если

\(A = \left[ {\ begin {массив} {* {20} {c}} {{a_{11}}}&{{a_{12}}}&{{a_{13}}}\\ {{a_{21}}}&{{a_{22}}}&{{a_{23}}}\\ {{a_{31}}}&{{a_{32}}}&{{a_{33}}} \end{массив}} \right]\) 9T}\), где \(C\) — матрица кофакторов \(A\).

Узнайте о матрицах здесь

См. диаграмму ниже, чтобы узнать, как вычисляется сопряжение матрицы \(A\).

Как рассчитать обратную матрицу?

Если вас спросят, что является обратным числом \(5\)?, вы быстро ответите \(\frac{1}{5}\). Но задумывались ли вы когда-нибудь о том, почему оно является обратным к \(5\)? Это потому, что \(5 \times \frac{1}{5} = 1\), что является мультипликативным тождеством.

То же самое и с матрицами; если нас спросят об обратной матрице \(A\), то что нужно умножить на \(A\), чтобы получить единичную матрицу, которая в основном является обратной матрицей \(A\). 9{ – 1}} = \frac{{adjA}}{{|A|}}\) где \(|A| \ne 0\)

Формула обратной матрицы

Ниже приведены два метода нахождения обратной матрицы:

1. Элементарные операции

Предположим, что \(X, A\) и \(B\) — матрицы одного порядка такие, что \(X = AB\). Чтобы иметь последовательность элементарных операций со строками над матричным уравнением \(X = AB\), применим эти операции одновременно к \(X\) и к первой матрице \(A\) произведения \( АВ\) справа. Аналогично, чтобы иметь последовательность элементарных операций со столбцами над матричным уравнением \(X = AB\), применим эти операции одновременно к X и ко второй матрице \(B\) произведения \(AB\) на \(RHS\). 9{– 1}}.\).

  • Если у квадратной матрицы \(A\) есть обратная (несингулярная), то обратная матрица уникальна.
  • Квадратная матрица \(A\) имеет обратную матрицу тогда и только тогда, когда определитель не равен нулю, т. е. \(|A| \ne 0\). Точно так же матрица A сингулярна (не имеет обратной) тогда и только тогда, когда ее определитель равен нулю, т. { – 1}}} \right| = \frac{1}{{|A|}}\). 9{ – 1}}\)
  • Пусть \(A\) и \(B\) – квадратные матрицы порядка \(n\). Если \(AB= O\), то либо \(A = O\), либо \(B = O\), либо обе \(A\) и \(B\) являются сингулярными матрицами без обратной.
  • Обратная матрица: примеры

    Предположим, мы хотим вычислить обратную матрицу \(A = \left[ {\begin{array}{*{20}{c}} 1&0&0\\ 3&5&0\\ 2&1&8 \end{array}} \right]\)

    Чтобы найти обратную матрицу, нам сначала нужно найти сопряженную матрицу A.

    Кофактор \(1 = {A_{11}} = + \left| {\begin{array}{*{20}{c}} 5&0\\ 1 и 8 \end{массив}} \right| = + (40 – 0) = 40\)
    Сомножитель \(2 = {A_{12}} = – \left| {\begin{array}{*{20}{c}} 3&0\\ 2 и 8 \end{массив}} \right| = – (24 – 0) = – 24\)
    Сомножитель \(3 = {A_{13}} = + \left| {\begin{array}{*{20}{c}} 3&5\\ 2 и 1 \end{массив}} \right| = + (3 – 10) = – 7\)
    Сомножитель \(4 = {A_{21}} = – \left| {\begin{array}{*{20}{c}} 0&0\\ 1 и 8 \end{массив}} \right| = 0\)
    Кофактор \(5 = {A_{22}} = + \left| {\begin{array}{*{20}{c}} 1&0\\ 2 и 8 \end{array}} \right|= + (8 – 0) = 8\)
    Сомножитель \(6 = {A_{23}} = – \left| {\begin{array}{*{20}{ в}} 1&0\\ 2 и 1 \end{массив}} \right| = – (1 – 0) = – 1\)
    Сомножитель \(7 = {A_{31}} = + \left| {\begin{array}{*{20}{c}} 0&0\\ 5&0 \end{массив}} \right| = 0\)
    Кофактор \(8 = {A_{32}} = – \left| {\begin{array}{*{20}{c}} 1&0\\ 3&0 \end{массив}} \right| = – (1 – 0) = – 1\)
    Кофактор \(9= {A_{33}} = + \left| {\начать{массив}{*{20}{с}} 1&0\\ 3 и 5 \end{массив}} \right| = + (5 – 0) = 5\)
    Матрица кофакторов A равна

    \(\left[ {{A_{ij}}} \right] = \left[ {\begin{array}{*{20 {с}} {{А_{11}}}&{{А_{12}}}&{{А_{13}}}\\ {{А_{21}}}&{{А_{22}}}&{{А_{23}}}\\ {{А_{31}}}&{{А_{32}}}&{{А_{33}}} \end{массив}} \right] = \left[ {\begin{массив}}{*{20}{c}} {40}&{ – 24}&{ – 7}\\ 0&8&{ – 1}\\ 0&{ – 1}&5 \end{array}} \right]\)

    Теперь найдите транспонирование \({A_{ij}}\) 9{ – 1}} = \ frac{{adj\,A}}{{\left| A \right|}} = \frac{1}{{40}}\left[ {\begin{array}{*{20}{c}} {40}&0&0\\ { – 24}&8&{ – 1}\\ {– 7}&{– 1}&5 \end{array}} \right]\)

    Применение обратной матрицы

    Применение обратной матрицы:

    • Обратная матрица используется для решения системы линейных уравнений. Это также говорит нам о последовательном или непоследовательном поведении решения уравнений.
    • Линейные уравнения, некоторые коды с исправлением ошибок (линейные коды), линейные дифференциальные уравнения и линейные рекуррентные последовательности используют концепцию обратной матрицы.
    • Обратные матрицы часто используются для шифрования кодов сообщений. Матрицы используются программистами для кодирования или шифрования писем. Сообщение состоит из ряда двоичных чисел, которые решаются с использованием теории кодирования для связи. В результате для решения таких уравнений используется понятие матриц.
    • Инженеры и физики разрабатывают модели физических структур и выполняют точные расчеты, необходимые для работы сложной техники. Точно настроенные вычисления матричных преобразований используются в электронике, сетях, самолетах и ​​космических кораблях, а также в химической обработке.
    • В физике обратная матрица используется для изучения электрических цепей, квантовой механики и оптики. { – 1}}\) не будет существовать. 9{ – 1}}\) не будет существовать.

      Q.2. Найти обратную матрицу \(\left[ {\begin{array}{*{20}{c}} 7&1\\ 4&{ – 3} \end{array}} \right]\), используя элементарные операции
      Ответ: Учитывая, что \(A = \left[ {\begin{array}{*{20}{c}} 7&1\\ 4&{ – 3} \end{array}} \right]\)
      Мы знаем \(A = IA \Rightarrow \left[ {\begin{array}{*{20}{c}} 7&1\\ 4&{- 3} \end{array}} \right] = \left[ {\begin{ array}{*{20}{c}} 1&0\\ 0&1 \end{массив}} \right]A\)
      \( \Rightarrow \left[ {\begin{array}{*{20}{c}} 1&{\frac{1}{7}}\\ 4&{- 3} \end{array}} \right] = \left[ {\begin{array}{*{20}{c}} {\frac{1}{7}}&0\\ 0&1 \end{array}} \right] A\) (Применение \({ R_1} \ to \ frac {1} {7} {R_1} \}}
      \( \ Rightarrow \ left [ {\ begin {array} {* {20} {c}} 1 & {\ frac {1} {7 }}\\ 0&1 \end{массив}} \right] = \left[ {\begin{array}{*{20}{c}} {\frac{1}{7}}&0\\ {\frac{ 4}{7}}&{ – \frac{7}{{25}}} \end{array}} \right]A\) (Применение \({R_2} \to {R_2} – 4{R_1}\ ))
      \( \Rightarrow \left[ {\begin{array}{*{20}{c}} 1&{\frac{1}{7}}\\ 0&1 \end{array}} \right] = \left[ {\ begin {array} {* {20} {c}} {\ frac {1} {7}} & 0 \\ {\ frac {4} {{25}}} & { – \ frac {7} {{ 25}}} \end{array}} \right]A\) (Применение \({R_2} \to – \frac{7}{{25}}{R_2}\))
      \( \Rightarrow \left[ {\ begin {array} {* {20} {c}} 1 & 0 \\ 0 & 1 \ end {array}} \ right] = \ left [ {\ begin {array} {* {20} {c}} {\ frac {3}{{25}}}&{\frac{1}{{25}}}\\ {\frac{4}{{25}}}&{ – \frac{7}{{25}}} \end{array}} \right]A\) (Применение \({R_1} \to {R_1} – \frac{1}{7}{R_2}\)) 9{ – 1}} = \frac{1}{{25}}\left[ {\begin{array}{*{20}{c}} 3&1\\ 4&{ – 7} \end{массив}} \right ]\)

      Q. 3. Найдите обратную матрицу \(\left[ {\begin{array}{*{20}{c}} 0&1&2\\ 1&2&3\\ 3&1&1 \end{array}} \right]\), используя элементарные операции
      Ответ:

      Учитывая, что \(A = \left[ {\begin{array}{*{20}{c}} 0&1&2\\ 1&2&3\\ 3&1&1 \end{array}} \right]\)
      Мы знаем \(A = IA \Rightarrow \left[ {\begin{array}{*{20}{c}} 0&1&2\\ 1&2&3\\ 3&1&1 \end{array}} \right] = \left[ {\begin{array}{* {20}{c}} 1&0&0\\ 0&1&0\\ 0&0&1 \end{массив}} \right]A\)
      \( \Стрелка вправо \влево[ {\begin{array}{*{20}{c}} 0&1&2\\ { – 2}&1&2\\ 3&1&1 \end{массив}} \right] = \left[ {\begin {array}{*{20}{c}} 1&0&0\\ 0&1&{ – 1}\\ 0&0&1 \end{array}} \right]A\) (Применение \({R_2} \to {R_2} – {R_3 }\))
      \( \Rightarrow \left[ {\begin{array}{*{20}{c}} 0&1&2\\ { – 2}&1&2\\ 3&1&1 \end{array}} \right] = \left [ {\begin{array}{*{20}{c}} 1&0&0\\ 0&1&{ – 1}\\ 0&0&1 \end{array}} \right]A\) (Применение \({R_1} \to {R_1 } – {R_3}\))
      \( \Стрелка вправо \влево[ {\begin{array}{*{20}{c}} { – 3}&0&1\\ 0&3&4\\ 0&{ – 2}&{ – 2} \end{array}} \ right] = \left[ {\begin{array}{*{20}{c}} 1&0&{ – 1}\\ { – 2}&3&{ – 1}\\ 3&{ – 3}&1 \end{array} } \right]A\) (Применение \({R_2} \to 3{R_2} – 2{R_1}\))
      \( \Rightarrow \left[ {\begin{array}{*{20}{c} } { – 3}&0&1\\ 0&3&4\\ 0&{ – 2}&{ – 2} \end{массив}} \right] = \left[ {\begin{array}{*{20}{c}} 1&0& { – 1}\\ { – 2}&3&{ – 1}\\ 3&{ – 3}&1 \end{массив}} \right]A\) (Применение \({R_3} \к {R_3} – {R_2 }\))
      \( \Rightarrow \left[ {\begin{array}{*{20}{c}} { – 3}&0&1\\ 0&3&4\\ 0&1&1 \end{array}} \right] = \left[ {\begin {массив}{*{20}{c}} 1&0&{ – 1}\\ { – 2}&3&{ – 1}\\ {\ frac {{ – 3}}{2}} & {\ frac {3} {2}}&{\frac{{ – 1}}{2}} \end{array}} \right]A\) (Применение \({R_2} \to {R_2} – 4{R_3}\))
      \( \Стрелка вправо \влево[ {\begin{array}{*{20}{c}} { – 3}&0&1\\ 0&{ – 1}&0\\ 0&0&1 \end{массив}} \right] = \ слева [ {\ begin {массив} {* {20} {c}} 1 & 0 & { — 1} \\ 4 & { — 3} & 1 \\ {\ frac {5} {2}} & {\ frac {{ — 3 }}{2}}&{\frac{1}{2}} \end{array}} \right]A\) (Применение \({R_3} \to {R_3} – {R_2}\)) 9{ – 1}} = \ left[ {\ begin {массив} {* {20} {c}} {\ frac {1} {2}} & {\ frac {{ — 1}} {2}} & { \ frac {1} {2}} \\ { — 4} & 3 & { — 1} \\ {\ frac {5} {2}} & {\ frac {{ — 3}} {2}} & {\ frac {1}{2}} \end{массив}} \right]\)

      Q. 4. Решите следующую систему линейных уравнений методом обращения матриц: \(5x + 2y = 3,\,3x + 2y = 5\)
      Ответ:

      Матричная форма системы имеет вид \(AX = B,\), где \(A = \left[ {\begin{array}{*{20}{c}} 5&2\\ 3&2 \end{array}} \right],\,X = \left[ {\begin{array}{ *{20}{c}} x\\ y \end{array}} \right],\,B = \left[ {\begin{array}{*{20}{c}} 3\\ 5 \end {массив}} \справа].\) 9{ – 1}}B,\) получаем
      \(\left[ {\begin{array}{*{20}{c}} x\\ y \end{array}} \right] = \frac{1 }{4}\left[ {\begin{array}{*{20}{c}} 2&{ – 2}\\ { – 3}&5 \end{array}} \right]\left[ {\begin{ array}{*{20}{c}} 3\\ 5 \end{массив}} \right] = \frac{1}{4}\left[ {\begin{array}{*{20}{c} } 6&{ – 10}\\ { – 9}&{ + 25} \end{массив}} \right] = \frac{1}{4}\left[ {\begin{array}{*{20}{ c}} { – 4}\\ {16} \end{array}} \right] = \left[ {\begin{array}{*{20}{c}} {\frac{{ – 4}}{ 4}}\\ {\frac{{16}}{4}} \end{массив}} \right] = \left[ {\begin{array}{*{20}{c}} { – 1}\ \ 4 \end{массив}} \right]\)
      Следовательно, решение равно \(\left( {x = – 1,\,y = 4} \right). \)

      Q.5. Решите следующую систему уравнений методом обращения матриц: \(2{x_1} + 3{x_2} + 3{x_3} = 5
      {x_1} – 2{x_2} + {x_3} = – 4
      3{x_1 } – {x_2} – 2{x_3} = 3\)
      Ответ:
      Матричная форма системы имеет вид \(AX=B\), где
      \(A = \left[ {\begin{array}{* {20}{c}} 2&3&3\\ 1&{ – 2}&2\\ 3&{ – 1}&{ – 2} \end{массив}} \right],\,X = \left[ {\begin{массив }{*{20}{c}} {{x_1}}\\ {{x_2}}\\ {{x_3}} \end{массив}} \right],\,B = \left[ {\begin{ array}{*{20}{c}} 5\\ { – 4}\\ 3 \end{array}} \right].\) Находим \(\left| A \right| = \left[ {\ begin{array}{*{20}{c}} 2&3&3\\ 1&{ – 2}&2\\ 3&{ – 1}&{ – 2} \end{массив}} \right] = 2\left( {4 + 1} \вправо) – 3\влево( { – 2 – 3} \вправо) + 3\влево( { – 1 + 6} \вправо) = 10 + 15 + 15 = 40 \ne 0.\) 9{ – 1}}B,\) получаем \(\left[ {\begin{array}{*{20}{c}} {{x_1}}\\ {{x_2}}\\ {{x_3}} \end{массив}} \right] = \frac{1}{{40}}\left[ {\begin{array}{*{20}{c}} 5&3&9\\ 5&{ – 13}&1\\ 5& {11}&{ – 7} \end{массив}} \right]\left[ {\begin{array}{*{20}{c}} 5\\ { – 4}\\ 3 \end{массив} } \right] = \frac{1}{{40}}\left[ {\begin{array}{*{20}{c}} {25}&{ – 12}&{ + 27}\\ {25 }&{ + 52}&{ + 3}\\ {25}&{ – 44}&{ – 21} \end{массив}} \right] = \frac{1}{{40}}\left[ { \begin{array}{*{20}{c}} {40}\\ {80}\\ { – 40} \end{array}} \right] = \left[ {\begin{array}{*{ 20}{c}} 1\\ 2\\ { – 1} \end{массив}} \right]\)
      Следовательно, решение равно \(\left( {{x_1} = 1, {x_2} = 2, {x_3} = – 1} \right)\)

      Резюме

      Обратная матрица является важным инструментом в математике . Мы узнали об обратной матрице, ее свойствах и примерах. С его помощью можно решить массу сложных задач. Он используется для решения линейных уравнений и других математических функций, таких как исчисление, оптика и квантовая физика. Он имеет широкий спектр реальных приложений, что привело к тому, что он играет решающую роль в математике.

      Часто задаваемые вопросы об обратной матрице

      Ниже приведены наиболее часто задаваемые вопросы об обратной матрице:

      Q.1: Что является обратной матрицей \(2 \times 2\)?
      Ответ:
      Для квадратной матрицы порядка \(2\) , заданной как \(A = \left[ {\begin{array}{*{20}{c}} {{a_{11}}}&{ {a_{12}}}\\ {{a_{21}}}&{{a_{22}}} \end{array}} \right]\)
      \(adj A\) можно получить путем замены \({{a_{11}}}\) и \({{a_{22}}}\) и меняя знаки перед \({{a_{12}}}\) и \({{a_{21 }}}\), т. е. 9T},\), где C — кофакторная матрица \(A.\) 

      Q.4: Может ли матрица иметь \(2\) обратную?
      Ответ:
      Нет, матрица не может иметь \(2\) обратную. Обратная матрица единственна. Это будет доказано с помощью метода от противного.

      Q.5: Каково использование обратной матрицы?
      Ответ:
      Обратная матрица используется для решения системы линейных уравнений. Он часто используется для шифрования кодов сообщений. Матрицы используются программистами для кодирования или шифрования писем. Сообщение состоит из ряда двоичных чисел, которые решаются с использованием теории кодирования для связи, а затем используется обратная матрица для расшифровки закодированного сообщения.

      Узнайте об использовании матриц здесь

      Мы надеемся, что эта статья об обратной матрице окажется вам полезной. В случае возникновения каких-либо вопросов вы можете связаться с нами в разделе комментариев, и мы вернем ответы.

      Следите за последними обновлениями Embibe.  

      Объяснение урока: Свойства обратных матриц

      В этом объяснении мы узнаем, как использовать некоторые свойства обратной матрицы.

      Матрица 𝑛×𝑛 𝐴 называется обратимой, если существует матрица 𝑛×𝑛 𝐵 такая, что произведение 𝐴 и 𝐵 равно 𝐼, где 𝐼 — единичная матрица 𝑛×𝑛: Если 𝐴𝐵=𝐼,𝐵=𝐴.

      Если 𝐵 существует, мы говорим, что это , обратное к 𝐴, обозначаемое 𝐴.

      Обратите внимание, что, как подразумевается в этом определении, матрица должна быть квадратной, чтобы быть обратимой, но квадратность не гарантирует, что обратная матрица существует.

      Чтобы найти обратную матрицу 2 × 2 𝐴 такую, что 𝐴=𝑎𝑏𝑐𝑑, применим формулу 𝐴=1𝐴𝑑−𝑏−𝑐𝑎,det где det𝐴=𝑎𝑑−𝑏𝑐. Обратите внимание, что если определитель матрицы 𝐴 равен нулю, обратной не может быть. Если определитель не равен нулю, матрица 𝐴 будет иметь обратную. Тогда мы называем матрицу 𝐴 обратимой или невырожденной. Свойства обратных матриц, которые мы рассмотрим в этом уроке, применимы ко всем обратимым матрицам.

      Давайте воспользуемся определением обратной матрицы, чтобы вывести некоторые ключевые свойства обратных матриц.

      Пример 1.

      Определение эквивалентного выражения для матриц с использованием свойств обратных матриц

      Если 𝐴 — матрица, какое из следующего равно 𝐴?

      1. 𝐴
      2. 𝐴
      3. 𝐴
      4. 𝐴

      Ответ

      Поскольку 𝐴 существует, 𝐴 должна быть квадратной матрицей. Давайте представим, что 𝐴 — это матрица 2 × 2, такая что 𝐴=𝑎𝑏𝑐𝑑.

      На основании определения обратной матрицы 2×2, 𝐴=1𝐴𝑑−𝑏−𝑐𝑎.det

      Если возвести в квадрат обратную величину 𝐴, то 𝐴 = 1 (𝐴) 𝑑 — 𝑏 — 𝑐𝑎𝑑 — 𝑏 — 𝑐𝑎 = 1 (𝐴) 𝑑+𝑏𝑐 — 𝑏𝑑 — 𝑐𝑑 — 𝑎𝑐𝑏𝑐+𝑎. detdet

      Далее мы хотим рассчитать 𝐴 = 𝑎𝑏𝑐𝑑𝑎𝑏𝑐𝑑 = 𝑎+𝑏𝑐𝑎𝑏+𝑏𝑑𝑎𝑐+𝑐𝑑𝑏𝑐+𝑑.

      , взяв обратный квадрат матрицы 𝐴, у нас есть 𝐴 = 1 (𝐴) 𝑑+𝑏𝑐 — 𝑏𝑑 — 𝑎𝑏 — 𝑐𝑑 — 𝑎𝑐𝑏𝑐+𝑎.det

      Обратите внимание, что свойство детерминантов, которые определяют (𝐴𝐵) = (𝐴) (𝐵 ) позволяет нам вычислить определитель 𝐴 как (𝐴)det.

      Поскольку выражение для обратного квадрата матрицы 𝐴 совпадает с выражением для квадрата обратного к матрице 𝐴, мы показали, что для обратимой матрицы 2 × 2 𝐴, 𝐴=𝐴. 

      В предыдущем примере мы показали, что 𝐴=𝐴. Это можно обобщить для более высоких степеней обратной матрицы, так что для любой обратимой матрицы 𝐴, где 𝑛∈𝑁, 𝐴=(𝐴).

      В нашем следующем примере мы рассмотрим связь между транспонированием инверсии и инверсией транспонирования.

      Пример 2. Определение эквивалентного выражения для матриц с использованием свойств обратных матриц

      Если 𝐴 — матрица, какое из следующего равно 𝐴?

      1. 𝐴
      2. 𝐴
      3. 𝐴
      4. 𝐴
      5. 𝐴

      Ответ

      Напомним, что столица 𝑇 Записано в тексту SuperScript — это нотация для транспонирования матрицы. Это означает, что мы меняем местами строки со столбцами. Когда мы транспонируем матрицу, значения по диагонали не меняются.

      Поскольку 𝐴 существует, 𝐴 должна быть квадратной матрицей. Давайте представим, что 𝐴 — это матрица 2 × 2, такая что 𝐴=𝑎𝑏𝑐𝑑.

      На основании определения обратной матрицы 2×2, 𝐴=1𝑎𝑑−𝑏𝑐𝑑−𝑏−𝑐𝑎. 

      Если мы транспонируем обратную матрицу 𝐴, мы имеем 𝐴=1𝑎𝑑−𝑏𝑐𝑑−𝑐−𝑏𝑎.

      Обратите внимание, что, поскольку дробь может быть распределена по матрице, транспонирование не влияет на нее, поэтому мы можем оставить ее снаружи матрица.

      Далее мы хотим вычислить транспонирование матрицы 𝐴, что дает 𝐴=𝑎𝑐𝑏𝑑.

      Обратное преобразование матрицы 𝐴 даст 𝐴=1𝑎𝑑−𝑏𝑐𝑑−𝑐−𝑏𝑎.

      Мы только что продемонстрировали, что для обратимой матрицы 2×2 В предыдущем примере мы показали, что Это можно обобщить для всех обратимых матриц 𝐴: 𝐴=𝐴.

      В следующем примере мы рассмотрим, что происходит, когда мы находим обратную обратную матрицу.

      Пример 3. Использование свойств обратных матриц для решения задачи

      Рассмотрим матрицу 𝐴=−31−25. Найдите 𝐴.

      Ответ

      Чтобы найти обратную матрицу 𝐴, мы сначала хотим найти обратную матрицу 𝐴. Для матрицы вида 𝐴=𝑎𝑏𝑐𝑑, обратное будет равно 𝐴=1𝑎𝑑−𝑏𝑐𝑑−𝑏−𝑐𝑎. 

      Следовательно, в этом примере будет обратное 𝐴=1(−3)(5)−(1)(−2)5−12−3=−1135−12−3=⎛⎜⎜⎝−513113−213313⎞⎟⎟⎠. 

      Если бы мы нашли обратное 𝐴, мы бы взяли 1−⎛⎜⎜⎝313−113213−513⎞⎟⎟⎠, что упрощает до −13⎛⎜⎜⎝313−113213−513⎞⎟⎟⎠.

      Затем умножаем на скаляр −13: −31−25.

      Мы только что показали, что матрица, обратная обратной матрице, является исходной матрицей. Однако для решения этой задачи нет необходимости выполнять все эти вычисления, поскольку это свойство верно для всех обратимых матриц: 𝐴=𝐴.

      Как только мы покажем, что матрица 𝐴 имеет ненулевой определитель, свойства обратных матриц говорят нам, что матрица, обратная обратной, будет исходной матрицей, −31−25.

      В предыдущем примере мы продемонстрировали, что 𝐴=𝐴 для матрицы 2×2. Это можно обобщить для всех обратимых матриц: 𝐴=𝐴.

      В следующем примере мы увидим, как мы можем использовать свойства обратных матриц и единичной матрицы для упрощения решения задач.

      Пример 4. Использование свойств обратных матриц для решения задачи

      1. Имея матрицы 𝐴 и 𝐵, где 𝐴=1−230−14001 и 𝐵=1−250−14001, найдите 𝐴𝐵.
      2. Не производя дальнейших вычислений, найдите 𝐴.

      Ответ

      Часть 1

      В первой части этого вопроса нам предлагается умножить матрицу 𝐴 на матрицу 𝐵. Поскольку обе эти матрицы являются квадратными матрицами 3 × 3, их можно перемножать вместе, помня, что умножение матриц не является коммутативным. Чтобы перемножить матрицы, мы находим скалярное произведение строк в первой матрице и столбцов во второй: 𝐴𝐵=(1)(1)+(-2)(0)+(3)(0)(1)(-2)+(-2)(-1)+(3)(0)(1) (5)+(−2)(4)+(3)(1)(0)(1)+(−1)(0)+(4)(0)(0)(−2)+(−1 )(−1)+(4)(0)(0)(5)+(−1)(4)+(4)(1)(0)(1)+(0)(0)+(1) (0)(0)(−2)+(0)(−1)+(1)(0)(0)(5)+(0)(4)+(1)(1).

      Когда мы оцениваем каждое из этих выражений, мы находим 𝐴𝐵=100010001.

      Часть 2

      Во второй части вопроса нам нужно было найти обратную матрицу 𝐴 без дальнейших вычислений. Когда мы перемножили матрицы 𝐴 и 𝐵, результирующая матрица была единичной матрицей. Напомним, что любая квадратная матрица (с ненулевым определителем) имеет обратную такую, что 𝐴𝐴=𝐼. Поскольку произведение матрицы 𝐴 и матрицы 𝐵 было единичной матрицей, матрица 𝐵 должна быть обратной матрице 𝐴.

      Следовательно, 𝐴=1−250−14001.

      Есть еще одно свойство, которое мы можем использовать: связь между произведением двух матриц и их обратной.

      Для пары обратимых матриц 𝐴 и 𝐵, (𝐴𝐵)=𝐵𝐴.

      В нашем последнем примере мы рассмотрим, как применить эту связь между инверсией произведения двух матриц и произведением их соответствующих инверсий.

      Пример 5. Использование свойств обратных матриц для решения задачи

      Учитывая, что (𝐴𝐵)=165−3−3321,𝐴=−2−1−3−2, определить 𝐵.

      Ответ

      Напомним, что для пары обратимых матриц 𝐴 и 𝐵 (𝐴𝐵)=𝐵𝐴. Это означает, что мы можем переписать наше первое уравнение как 𝐵𝐴=165−3−3321. 

      Теперь нам нужна некоторая стратегия, чтобы «вычеркнуть» 𝐴 из обеих частей этого уравнения. Для этого мы используем ключевое свойство единичной матрицы. Мы знаем, что 𝐴𝐴=𝐼=𝐴𝐴. Это означает, что мы можем умножить обе части этого уравнения на матрицу 𝐴, которую нам дали, помня, что умножение матриц не является коммутативным; мы должны умножить в правильном порядке: 𝐵𝐴𝐴=165−3−3321⋅−2−1−3−2.

      Отсюда мы следуем правилам умножения матриц 2×2 и находим, что 𝐵𝐴𝐴=16(5)(−2)+(−3)(−3)(5)(−1)+(−3)(−2)(−33)(−2)+(21)(− 3)(−33)(−1)+(21)(−2)=16−113−9.

      В левой части этого уравнения мы видим, что мы умножили обратное матрицы 𝐴 на матрицу 𝐴. Мы знаем, что это равно единичной матрице. Таким образом, мы можем переписать левую часть уравнения в виде 𝐵𝐼=16−113−9.

      Вспоминая, что умножение любой матрицы на единичную матрицу дает само себя, мы можем сказать 𝐵=16−113−9.

      Последним шагом будет умножение на скаляр 16 для получения 𝐵=⎛⎜⎜⎝−161612−32⎞⎟⎟⎠.

    Косинус найти через синус: Синус, косинус и тангенс угла — урок. Геометрия, 9 класс.

    Алгоритм нахождения синуса любого угла? — Хабр Q&A

    Смотря какая точность нужна. Есть формула Бхаскара, работающая на диапазоне от 0° до 180° (0-π):
    sin(x°) = 4·x·(180−x)/(40500−x·(180−x))
    sin(x) = 16·x·(π−x)/(5·π2−4·x·(π−x))
    На большей части диапазона она даёт точность в пределах 0.2%, на краях точность падает до 2%.
    Значения для углов вне этого диапазона можно получить из тождества:
    sin(2·π+x) = sin(x)
    sin(π+x) = -sin(x)

    Ответ написан

    Иногда для скорости используют(вали) табличный метод.
    Т.е. создают таблицу в памяти с вычисленными значениями функции(с нужной точностью)
    для какого-то фиксированного шага по углу.
    Если попадаем между узлами сетки таблицы углов, то используем интерполяцию.

    Ответ написан

    Комментировать

    1) в арифметическом сопроцессоре длина дробных чисел 10 байт, а не 8(double), так что точность повыше.

    2) ряды Фурье применять тяжело, т.к. факториал (в знаменателе каждой дроби) быстро растёт.

    3) есть метод CORDIC, который по небольшой таблице может рассчитать любую точность.

    4) слышал на видеосеминаре, что скорее всего используются многочлены Чебышёва (знай наших!)

    Ответ написан

    Комментировать

    Гуглите — приближенное вычисление синуса (подставьте нужную).
    Дело в том, что нет четкой формулы для нахождения значений этих функций, есть только приближенные, которые можно вычислять с заданной точностью, чем выше точность, тем дольше будет вычисляться.

    Ответ написан

    Самым быстрым и точным, но не экономичным, считается метод интерполяции в таблице. Однако существуют более экономичные и почти столь же быстрые (а, возможно, и более быстрые) методы вычисления трансцендентных функций — это аппроксимации по методу Чебышёва и Паде-Чебышёва. Производительность соответствующих алгоритмов будет зависеть от требуемой точности. Например, для аппроксимации тригонометрических функций с одинарной точностью (7-8 значащих десятичных цифр) по методу Чебышёва, как правило, достаточно использовать многочлен с 5-ю слагаемыми. Его вычисление настолько быстрое, что может превосходить по скорости табличный метод. Для вычисления функций с двойной и более высокой точностью лучшие результаты достигаются по методу Паде-Чебышёва (аппроксимация дробно-рациональной функцией).

    Дополнительным достоинством данных методов в том, что с помощью векторных команд несложно сделать, например, параллельное вычисление синуса и косинуса одного аргумента. В методе Паде-Чебышёва с помощью векторных команд возможно параллельное вычисление числителя и знаменателя функции, что почти вдвое повышает производительность. Я проверял производительность функции вычисления натурального логарифма с двойной точностью, написанной по методу Паде-Чебышёва, она превосходила стандартную библиотечную функцию. С одинарной точностью — ещё быстрее.

    В качестве примера вот ссылка на быструю функцию вычисления тангенса в градусах с одинарной точностью по методу Чебышёва (сообщение от 2021-09-20 13:12:22):
    https://codengineering.ru/q/bystraya-realizaciya-t…

    Ответ написан

    Комментировать

    6.

    5 Соотношения синуса, косинуса и тангенса и приложения тригонометрии – Fanshawe Pre-Health Sciences Mathematics 1

    Перейти к содержимому

    Ожидается, что к концу этого раздела вы сможете

    • Находить недостающую сторону прямоугольного треугольника, используя отношения синуса, косинуса или тангенса
    • Найдите недостающий угол прямоугольного треугольника, используя отношения синуса, косинуса или тангенса
    • Решение приложений с помощью тригонометрии прямого угла

    Теперь, когда мы знаем основы алгебры и геометрии, связанные с прямоугольным треугольником, мы можем приступить к изучению тригонометрии. Многие реальные жизненные задачи можно представить и решить с помощью прямоугольной тригонометрии.

    Отношения синуса, косинуса и тангенса

    Мы знаем, что любой прямоугольный треугольник имеет три стороны и прямой угол. Сторона, противолежащая прямому углу, называется гипотенузой. Два других угла прямоугольного треугольника — острые углы (с мерой меньше [латекс]90[/латекс] градусов). Один из этих углов мы называем опорным углом и используем [латекс]θ[/латекс] (тета) для его представления.

    Гипотенуза всегда является наибольшей стороной прямоугольного треугольника. Две другие стороны называются противоположной стороной и смежной стороной. Названия этих сторон зависят от того, какой из двух острых углов используется в качестве опорного угла.

    Рисунок 6.5.1

    В прямоугольном треугольнике каждая сторона помечена строчной буквой, соответствующей прописной букве противоположной вершины.

    Назовите стороны треугольника и найдите гипотенузу, противоположную и прилежащую.

    Рисунок 6.5.2 Решение

    Мы пометили стороны строчными буквами, чтобы они совпадали с прописными буквами противоположной вершины.

    [латекс]с[/латекс] является гипотенузой
    [латекс]а[/латекс] противоположно
    [латекс]b[/латекс] примыкает к

    Рисунок 6. 5.3

    1) Обозначьте стороны треугольника и найдите гипотенузу, противолежащую и прилежащую.

    Рисунок 6.5.4 Решение

    [латекс]y[/латекс] гипотенуза
    [латекс]z[/латекс] напротив
    [латекс]х[/латекс] примыкает

    Тригонометрические соотношения

    Тригонометрические отношения — это отношения сторон прямоугольного треугольника. Для любого прямоугольного треугольника мы можем определить три основных тригонометрических соотношения: синус, косинус и тангенс.

    Давайте обратимся к рисунку 6.5.1 и определим три основных тригонометрических отношения как:

    Три основных тригонометрических отношения
        • [латекс] синус (\ тета) = \ гидроразрыва {\ текст {длина противоположной стороны}} {\ текст {длина стороны гипотенузы}} [/латекс]
        • [латекс] косинус (\ тета) = \ гидроразрыва {\ текст {длина прилегающей стороны}} {\ текст {длина стороны гипотенузы}} [/латекс]
        • [латекс]тангенс(\тета) = \frac{\text{длина противоположной стороны}}{\text{длина соседней стороны}}[/latex]

    Где [латекс]θ[/латекс] — мера опорного угла, измеренная в градусах.

    Очень часто мы используем сокращения для синуса, косинуса и тангенса.

        • [латекс] грех (\ тета) = \ гидроразрыва {опп} {hyp} [/латекс]
        • [латекс] соз (\ тета) = \ гидроразрыва {прил. {hyp} [/латекс]
        • [латекс] загар (\ тета) = \ гидроразрыв {опп} {прил} [/латекс]

    Некоторые люди помнят определение тригонометрических соотношений как SOH CAH TOA.

    Давайте воспользуемся [латекс]\Delta DEF[/латекс] из примера 6.5.2, чтобы найти три отношения.

    Для данного треугольника найти отношение синуса, косинуса и тангенса.

    Рисунок 6.5.5 Решение

    [латекс]\begin{align*}sin(\theta) &= \frac{f}{d}\\[2ex]cos(\theta)&= \frac{e} {d}\\[2ex]tan(\theta)&= \frac{f}{e}\end{align*}[/latex]

    2) Для данного треугольника найти отношение синуса косинуса и тангенса.

    Рисунок 6.5.6 Решение

    [латекс]\begin{align*}sin(\theta)&= \frac{z}{y}\\[2ex]cos(\theta)&= \frac{x} {y}\\[2ex]tan(\theta)&= \frac{z}{x}\end{align*}[/latex]

    В примере 6. 5.2 опорными углами могут быть угол [латекс]Е[/латекс] или угол [латекс]F[/латекс]. Используя определение тригонометрических отношений, мы можем написать [latex]sin(E)=\frac{e}{d}[/latex], [latex]cos(E)=\frac{f}{d}[/latex ] и [латекс]тан(Е)=\фрак{е}{е}[/латекс].

    При расчетах мы обычно округляем отношения до четырех знаков после запятой, а в конце наш окончательный ответ — до одного знака после запятой, если не указано иное.

    Для данного треугольника найти отношения синуса, косинуса и тангенса. При необходимости округлить до четырех знаков после запятой.

    Рисунок 6.5.7 Решение

    У нас есть два возможных опорных угла: [латекс]R[/латекс] и [латекс]S[/латекс].

    Используя определения, тригонометрические соотношения для угла [латекс]R[/латекс] составляют:

    • [латекс]sin(R)= \frac{4}{5} = 0,8[/латекс]
    • [латекс] cos(R)= \frac{3}{5} = 0,6[/латекс]
    • [латекс]загар(R)=\фракция{4}{3} = 1,3333…[/латекс]

    Используя определения, тригонометрические соотношения для угла [латекс]S[/латекс]:

    • [латекс]sin(S)= \frac{3}{5} = 0,6[/латекс]
    • [латекс] cos(S)= \frac{4}{5} = 0,8[/латекс]
    • [латекс]загар(S)= \фракция{3}{4} = 0,75[/латекс]

    3) Для данного треугольника найти отношения синуса, косинуса и тангенса. При необходимости округлить до четырех знаков после запятой.

    Рисунок 6.5.8 Решение
    • [латекс]sin(F)= \frac{8}{10} = 0,8[/латекс]
    • [латекс] cos(F)= \frac{6}{10} =0,6[/латекс]
    • [латекс]tan(F)= \frac{8}{6} = 1,3333…[/latex]
    • [латекс]sin(D)= \frac{6}{10} = 0,6[/латекс]
    • [латекс] cos(D)= \frac{8}{10} = 0,8[/латекс]
    • [латекс]тангенс(D)= \фракция{6}{8} = 0,75[/латекс]

    Теперь воспользуемся научным калькулятором, чтобы найти тригонометрические соотношения. Сможете ли вы найти кнопки sin, cos и tan на своем калькуляторе? Чтобы найти тригонометрические соотношения, убедитесь, что ваш калькулятор находится в режиме градусов.

    С помощью калькулятора найдите тригонометрические соотношения. При необходимости округлить до 4 знаков после запятой. 9\circ)=1[/латекс]

    Нахождение недостающих сторон прямоугольного треугольника

    В этом разделе вы будете использовать тригонометрические отношения для решения задач прямоугольного треугольника. Мы адаптируем нашу стратегию решения задач для приложений тригонометрии. Кроме того, поскольку в этих задачах будет задействован прямоугольный треугольник, полезно нарисовать его (если рисунок не предоставлен) и пометить его с помощью данной информации. Мы включим это в первый шаг стратегии решения проблем для тригонометрических приложений.

    КАК:
    Решить тригонометрические задачи
    1. Прочитайте задачу и убедитесь, что все слова и идеи понятны. Нарисуйте прямоугольный треугольник и обозначьте данные части.
    2. Определите, что мы ищем.
    3. Пометьте то, что мы ищем, выбрав переменную для ее представления.
    4. Найдите требуемое тригонометрическое соотношение.
    5. Решите отношение, используя хорошие методы алгебры.
    6. Проверьте ответ, подставив его обратно в соотношение на шаге 4 и убедившись, что он имеет смысл в контексте задачи.
    7. Ответьте на вопрос полным предложением

    В следующих нескольких примерах, зная меру одного острого угла и длину одной стороны прямоугольного треугольника, мы решим прямоугольный треугольник относительно недостающих сторон.

     Найти недостающие стороны. Округлите окончательный ответ до двух знаков после запятой

    Рисунок 6.5.9 Решение

    Шаг 1: Прочитайте задачу и убедитесь, что все слова и идеи понятны. Нарисуйте прямоугольный треугольник и обозначьте указанные части.

    Дается чертеж. Угол [латекс]Y[/латекс] — это наш опорный угол, [латекс]у[/латекс] — противоположная сторона, [латекс]z[/латекс] — смежная сторона, а [латекс]х=14[/латекс] — гипотенуза.

    Шаг 2: Определите, что вы ищете.

    а. Противоположная сторона.
    б. Соседняя сторона.

    Шаг 3: Пометьте то, что мы ищем, выбрав переменную для ее представления.

    [латекс]\begin{eqnarray*}y&=&?\\z&=&?\end{eqnarray*}[/latex] 9\circ)&=&z\\11.47&=&z\end{eqnarray*}[/latex]

    Шаг 6: Проверьте ответ в задаче и убедитесь, что он имеет смысл.

    [латекс]\begin{align*}\text{a. }\;0,57&\overset?=8,03 \div14\\ 0,57&=0,57\checkmark\\[3ex] \text{b.}\;0,82& \overset?=11,47 \div14\\ 0,82&=0,82 \checkmark \end{align*}[/latex]

    Шаг 7: Ответьте на вопрос полным предложением.

    а. Противоположная сторона — [латекс]8.03[/латекс].
    б. Соседняя сторона — [латекс]11,47[/латекс].

     5) Найдите недостающие стороны. Округлите окончательный ответ до одного знака после запятой.

    Рисунок 6.5.10 Решение

    [латекс]\begin{align*}a &= 20.2\\b &= 16.4\end{align*}[/latex]

    Найдите гипотенузу. Округлите окончательный ответ до одного знака после запятой.

    Рисунок 6.5.11 Решение

    Шаг 1: Прочитайте задачу и убедитесь, что все слова и идеи понятны. Нарисуйте прямоугольный треугольник и обозначьте указанные части.

    Дается чертеж. Угол [латекс]S[/латекс] – это наш опорный угол, [латекс]R[/латекс] – противоположная сторона, [латекс]r = 4[/латекс]  – прилежащая сторона, а [латекс]Р[/латекс] это гипотенуза.

    Шаг 2: Определите, что вы ищете.

    Гипотенуза.

    Шаг 3: Пометьте то, что мы ищем, выбрав переменную для ее представления.

    [латекс]\begin{eqnarray*}p&=&?\end{eqnarray*}[/latex] 9\circ)&=&\frac{4}{p}\end{eqnarray*}[/latex]

    Шаг 5: Решите отношение, используя хорошие методы алгебры.

    [латекс]\begin{eqnarray*}0.8480&=&\frac{4}{p}\\p&=&4.7170\;\;\text{Округление до 4 знаков после запятой}\end{eqnarray*} [/латекс]

    Шаг 6: Проверьте ответ в задаче и убедитесь, что он имеет смысл.

    [латекс]\begin{eqnarray*}0.8480&\overset?=&\frac{4}{4.7170}\\0.8480&=&0.8480\checkmark\end{eqnarray*}[/latex]

    Шаг 7: Ответьте на вопрос полным предложением.

    Гипотенуза равна [латекс]4,7[/латекс]. Окончательный ответ округлить до одного знака после запятой.

    6) Найдите гипотенузу. {-1}[/latex] находятся на вашем научном калькуляторе. . 9\circ[/латекс]

    В приведенном ниже примере у нас есть прямоугольный треугольник с двумя заданными сторонами. У нас отсутствуют острые углы. Давайте посмотрим, каковы шаги, чтобы найти недостающие углы.

    Найдите недостающий [латекс]\угол Т[/латекс] . Округлите окончательный ответ до одного знака после запятой.

    Рисунок 6.5.13 Решение

    Шаг 1: Прочитайте задачу и убедитесь, что все слова и идеи понятны. Нарисуйте прямоугольный треугольник и обозначьте указанные части.

    Дается чертеж. Угол [латекс]T[/латекс] — это наш базовый угол, [латекс]t = 7[/латекс] — противолежащая сторона, [латекс]s[/латекс] — прилежащая сторона, а [латекс]r = 11[/ латекс] — гипотенуза.

    Шаг 2: Определите, что вы ищете.

    Угол [латекс]T[/латекс].

    Шаг 3: Пометьте то, что мы ищем, выбрав переменную для ее представления.

    [латекс]\begin{eqnarray*}\angle T&=&?\end{eqnarray*}[/latex] 9\circ[/латекс]

    Найдите недостающий угол [латекс]А[/латекс]. Округлите окончательный ответ до одного знака после запятой.

    Рисунок 6.5.15 Решение

    Шаг 1: Прочитайте задачу и убедитесь, что все слова и идеи понятны. Нарисуйте прямоугольный треугольник и обозначьте указанные части.

    Дается чертеж. Угол [латекс]А[/латекс] — это наш опорный угол, [латекс]а = 9[/латекс] — противолежащая сторона, [латекс]с = 5[/латекс] — прилежащая сторона, а [латекс]b[ /латекс] — гипотенуза.

    Шаг 2: Определите, что вы ищете.

    Угол [латекс]А[/латекс].

    Шаг 3: Пометьте то, что мы ищем, выбрав переменную для ее представления.

    [латекс]\begin{eqnarray*}\angle A&=&?\end{eqnarray*}[/latex]

    Шаг 4: Найдите требуемое тригонометрическое соотношение.

    [латекс]\begin{eqnarray*}tan A&=&\frac{9}{5}\end{eqnarray*}[/latex] 9\circ\end{align*}[/latex]


    Шаг 1: Прочтите задачу и убедитесь, что все слова и идеи понятны. Нарисуйте прямоугольный треугольник и обозначьте данные части.

    Дается чертеж. Угол [латекс]А[/латекс] — это наш исходный угол, [латекс]а = 8[/латекс] — противоположная сторона, [латекс]b[/латекс] — прилежащая сторона, а [латекс]с[/латекс] ] — гипотенуза.

    Шаг 2: Определите, что вы ищете.

    9\circ)&=&\frac{8}{c}\end{eqnarray*}[/latex]

    Шаг 5: Решите отношение, используя хорошие методы алгебры.

    [латекс]\begin{eqnarray*}\text{a.}\;0.9004&=&\frac{8}{b}\\0.9004 b&=&8\\b&=&8.8849\\[3ex]\text {b.}\;0,6691&=&\frac{8}{c}\\0,6691 c&=&8\\c&=&11,9563\end{eqnarray*}[/latex]

    Шаг 6: Проверьте ответ в задаче и убедитесь, что он имеет смысл.
    9\circ\end{align*}[/latex]

    Solution

    [latex]\begin{align*}a &= 6\\b &= 15.6\\c &= 16.7\end{align*}[/ латекс]

    Решите прямоугольный треугольник. Округлить до двух знаков после запятой.

    Рисунок 6.5.19 Решение

    Шаг 1: Прочитайте задачу и убедитесь, что все слова и идеи понятны. Нарисуйте прямоугольный треугольник и обозначьте данные части.

    Дается чертеж. Пусть угол [латекс]D[/латекс] будет нашим опорным углом, [латекс]d = 4[/латекс] — противолежащая сторона, [латекс]f[/латекс] — прилежащая сторона, а [латекс]е = 9[/latex] — это гипотенуза.

    Шаг 2: Определите, что вы ищете.

    а. Угол Д.
    б. Соседний.

    Шаг 3: Пометьте то, что мы ищем, выбрав переменную для ее представления.

    [латекс]\begin{eqnarray*}\text{a.}\;\angle D&=&?\\[2ex] \text{b.}\;f&=&?\end{eqnarray*}[/latex ]

    Шаг 4: Найдите требуемое тригонометрическое соотношение.
    9\circ\end{align*}[/latex]

    Solution

    [latex]\begin{align*}d &= 29,4\\e &= 18,4\\f &= 60,6\end{align*}[/ латекс]

    Решение приложений с использованием тригонометрических соотношений

    В предыдущих примерах мы смогли найти недостающие стороны и недостающие углы прямоугольного треугольника. Теперь давайте воспользуемся тригонометрическими отношениями для решения реальных задач.

    Многие применения тригонометрических соотношений связаны с пониманием угла подъема или угла наклона. 9\circ[/latex] угол. Какова высота Harbour Centre?

    Решение

    Шаг 1: Прочитайте задачу и убедитесь, что все слова и идеи понятны. Нарисуйте прямоугольный треугольник и обозначьте данные части.

    Рисунок 6.5.23


    Угол [латекс]X[/латекс] — это наш исходный угол, [латекс]х[/латекс] — противоположная сторона, [латекс]у = 31[/латекс]м — прилежащая сторона, а [latex]z[/latex] — гипотенуза.

    Шаг 2: Определите, что вы ищете. 9\circ)&=&\frac{x}{31}\end{eqnarray*}[/latex]

    Шаг 5: Решите отношение, используя хорошие методы алгебры.

    [латекс]\begin{eqnarray*}4.7046&=&\frac{x}{31}\\x&=&145.8426\end{eqnarray*}[/latex]

    Шаг 6: Проверьте ответ в задаче и убедитесь, что он имеет смысл.

    [латекс]\begin{eqnarray*}4.7046&\overset?=&\frac{145.8426}{31}\\4.7046&=&4.7046 \checkmark\end{eqnarray*}[/latex] 9\circ[/latex] угол. Насколько высокое здание?

    Раствор

    [латекс]43,3[/латекс] метра

    Томас стоит на вершине здания высотой [latex]45[/latex] метров и смотрит на свою подругу, которая стоит на земле в [latex]22[/latex] метрах от основания здания. Что такое угол депрессии?

    Решение

    Шаг 1: Прочитайте задачу и убедитесь, что все слова и идеи понятны. Нарисуйте прямоугольный треугольник и обозначьте данные части.

    Рисунок 6.5.24

    Угол [латекс]Y[/латекс] — это наш исходный угол, [латекс]у = 45[/латекс]м — противоположная сторона, [латекс]z = 22[/латекс]м — прилежащая сторона , а [latex]x[/latex] — гипотенуза.

    Шаг 2: Определите, что вы ищете.

    Угол [латекс]Y[/латекс].

    Шаг 3: Пометьте то, что мы ищем, выбрав переменную для ее представления.

    [латекс]\begin{eqnarray*}\angle Y&=&?\end{eqnarray*}[/latex] 9\circ[/латекс]

    • Три основных тригонометрических соотношения: (Где [латекс]θ[/латекс] — мера опорного угла, измеренная в градусах).
      • [латекс] синус\;\тета = \фракция{\текст{длина противоположной стороны}}{\текст{длина стороны гипотенузы}}[/латекс]
      • [латекс]косинус\;\тета = \фракция{\текст{длина прилежащей стороны}}{\текст{длина гипотенузы}}[/латекс]
      • [латекс]тангенс\;\тета = \фракция{\текст{длина противоположной стороны}}{\текст{длина соседней стороны}}[/латекс]
    • Стратегия решения проблем для приложений тригонометрии
      1. Прочитайте задачу и убедитесь, что все слова и идеи понятны. Нарисуйте прямоугольный треугольник и обозначьте указанные части.
      2. Определите, что мы ищем.
      3. Пометьте то, что мы ищем, выбрав переменную для ее представления.
      4. Найдите требуемое тригонометрическое соотношение.
      5. Решите отношение, используя хорошие методы алгебры.
      6. Проверьте ответ, подставив его обратно в отношение, найденное на шаге 5, и убедившись, что оно имеет смысл в контексте задачи.
      7. Ответьте на вопрос полным предложением.

    Самопроверка

    а. После выполнения упражнений используйте этот контрольный список, чтобы оценить свое мастерство в выполнении целей этого раздела.

    б. В целом, после просмотра контрольного списка, как вы думаете, хорошо ли вы подготовились к следующему разделу? Почему или почему нет?

    License

    Fanshawe Pre-Health Sciences Mathematics 1 by Sav Spilotro, MSc распространяется под лицензией Creative Commons Attribution-ShareAlike 4. 0 International License, если не указано иное.

    Поделиться этой книгой

    Поделиться в Твиттере

    Использование функции косинуса для нахождения гипотенузы (Ключевой этап 3)

    Урок

    Функция косинуса связывает заданный угол с прилежащей стороной и гипотенузой прямоугольного треугольника. Длина гипотенузы находится по следующей формуле:

    В этой формуле θ — это угол прямоугольного треугольника, прилежащая — это длина стороны, следующей за углом, а гипотенуза — это длина наибольшей стороны. На изображении ниже показано, что мы имеем в виду:

    Как использовать функцию косинуса для нахождения гипотенузы прямоугольного треугольника

    Найти гипотенузу прямоугольного треугольника легко, если известны угол и прилежащие к нему.

    Какова длина гипотенузы изображенного ниже прямоугольного треугольника?

    Пошаговая инструкция:

    Начните с формулы:

    Гипотенуза = смежная / cos θ

    Не забудьте: / означает ÷

    Подставляем в формулу угол θ и длину прилежащего. В нашем примере θ = 60°, а прилежащее равно 4 см.

    Гипотенуза = 4 / cos (60°) Гипотенуза = 4 ÷ cos (60°) Гипотенуза = 4 ÷ 0,5 Гипотенуза = 8 см

    Ответ:

    Длина гипотенузы прямоугольного треугольника с углом 30° и прилежащим 4 см равна 8 см.

    Вспоминая формулу

    Часто самая сложная часть поиска неизвестного угла — это вспомнить, какую формулу использовать. Всякий раз, когда у вас есть прямоугольный треугольник, в котором вы знаете одну сторону и один угол и должны найти неизвестную сторону… ……подумайте о тригонометрии… ……………подумайте о синусе, косинусе или тангенсе… …………………………..думаю СОХ КАН ТОА .

    Глядя на приведенный выше пример, мы пытаемся найти H ypotenuse, и мы знаем A djacent.

    Две буквы, которые мы ищем, это AH , которые входят в CAH в SOH CAH TOA. Это напоминает нам уравнение:

    C os θ = A djacent / H ypotenuse

    Это изменено, чтобы получить формулу в верхней части страницы (см. Примечание ).

    H ypotenuse = A djacent / C os θ

    Слайды урока

    Ползунок ниже дает еще один пример нахождения гипотенузы прямоугольного треугольника (поскольку угол и прилежащее к нему известны). Откройте слайдер в новой вкладке

    Интерактивный виджет

    Вот интерактивный виджет, который поможет вам узнать о функции косинуса прямоугольного треугольника.

    Что такое функция косинуса?

    Функция косинуса является тригонометрической функцией. Косинусом данного угла в прямоугольном треугольнике называется отношение длины прилежащего катета к длине гипотенузы.

    Формула log: Формулы и свойства логарифмов, основные формулы логарифмов с примерами

    Функция LOG — Служба поддержки Майкрософт

    Excel для Microsoft 365 Excel для Microsoft 365 для Mac Excel для Интернета Excel 2021 Excel 2021 для Mac Excel 2019 Excel 2019 для Mac Excel 2016 Excel 2016 для Mac Excel 2013 Excel 2010 Excel 2007 Excel для Mac 2011 Excel Starter 2010 Еще…Меньше

    В этой статье описаны синтаксис формулы и использование функции LOG в Microsoft Excel.

    Описание

    Возвращает логарифм числа по заданному основанию.

    Синтаксис

    LOG(число;[основание])

    Аргументы функции LOG описаны ниже.

    • Число    Обязательный. Положительное вещественное число, для которого вычисляется логарифм.

    • org/ListItem»>

      Основание    Необязательный. Основание логарифма. Если аргумент «основание» опущен, предполагается, что он равен 10.

    Пример

    Скопируйте образец данных из следующей таблицы и вставьте их в ячейку A1 нового листа Excel. Чтобы отобразить результаты формул, выделите их и нажмите клавишу F2, а затем — клавишу ВВОД. При необходимости измените ширину столбцов, чтобы видеть все данные.

    Формула

    Описание

    Результат

    =LOG(10)

    Логарифм числа 10. Так как второй аргумент (основание) опущен, предполагается, что он равен 10. Результат (1) — степень, в которую необходимо возвести основание, чтобы получить число 10.

    1

    =LOG(8; 2)

    Логарифм числа 8 по основанию 2. Результат (3) — степень, в которую необходимо возвести основание, чтобы получить число 8.

    3

    =LOG(86; 2,7182818)

    Логарифм числа 86 по основанию e (приблизительно 2,718). Результат (4,454) — степень, в которую необходимо возвести основание, чтобы получить число 86.

    4,4543473

    Подготовка школьников к ЕГЭ (Справочник по математике — Алгебра

    Справочник по математикеАлгебраЛогарифмы
    Определение логарифма, основное логарифмическое тождество
    Свойства логарифмов
    Использование свойств логарифмов при решениии логарифмических уравнений и неравенств
    Десятичные логарифмы и натуральные логарифмы

    Определение логарифма, основное логарифмическое тождество

          Рассмотрим два произвольных действительных числа   a   и   b,   удовлетворяющих условиям

    (1)

          Определение.  Логарифмом числа   b   по основанию   a   называют такую степень, в которую надо возвести число   a,   чтобы получить число   b.

          Другими словами, логарифм числа   b   по основанию   a   – это такое число   x,   которое является решением уравнения

    a x= b .(2)

          Доказательство того, что решение уравнения (2) существует и единственно, выходит за рамки школьной программы.

          Для логарифма числа   b   по основанию   a   используется обозначение:

    loga b .

          Таким образом, для всех действительных чисел   a   и   b,   удовлетворяющих условиям (1), справедливо равенство

    которое часто называют основным логарифмическим тождеством.

          Замечание. Обратим особое внимание на то, что при решении уравнения (2) мы ищем показатель степени, а при решении уравнения

    x a = b.

    мы ищем основание степени, которое вычисляется по формуле

    и в случае, когда   a   – натуральное число, является корнем натуральной степени из числа   b.

          Пример 1. Решить уравнение

    x3 = 81 .

          Решение. Воспользовавшись понятием кубического корня и свойствами степеней, получаем

          Ответ: .

          Пример 2. Решить уравнение

    3x= 81 .

          Решение. Воспользовавшись тем, что число   81   является четвертой степенью числа   3 ,   получаем:

          Ответ:   4 .

          Задача. Доказать, что число

    log2 3

    иррационально.

          Решение. Предположим противное, т.е. предположим, что указанное число рационально. Тогда существует несократимая дробь

    ,

    числитель и знаменатель которой являются натуральными числами и такая, что справедливо равенство:

          Из определения логарифма отсюда вытекает равенство:

    следствием которого является равенство:

    2m= 3n .

          Но последнее равенство невозможно, поскольку его левая часть четное число, а правая – нечетное. Полученное противоречие доказывает требуемое в задаче утверждение.

    Свойства логарифмов

          Перечисленные ниже свойства логарифмов вытекают из основного логарифмического тождества:


    (основное свойство логарифмов),

    (основное свойство логарифмов),

    (формула перехода к новому основанию логарифмов),

    (основное свойство логарифмов),

    (основное свойство логарифмов),

    (формула перехода к новому основанию логарифмов),

    Использование свойств логарифмов при решении логарифмических уравнений и неравенств

          Для того, чтобы не ошибаться при решении логарифмических уравнений и неравенств, свойства логарифмов, перечисленные в предыдущем разделе, следует применять внимательно и аккуратно.

          Например, если при решении уравнения или неравенства требуется преобразовать выражение

    loga ( f (x)2 ) ,

    то вместо формулы

    следует применять формулу

    поскольку в противном случае можно потерять корни.

          По той же причине при преобразовании выражений

    loga ( f (x) g (x))    и

    следует использовать формулы:

    и

          Замечание. Желающим усовершенствовать свои знания и умения при решении уравнений и неравенств с логарифмами мы рекомендуем ознакомиться с нашими учебными пособиями «Решение логарифмических уравнений» и «Решение логарифмических неравенств».

    Десятичные логарифмы и натуральные логарифмы

          В математике, физике и во многих других областях естествознания и технологий важное место занимают десятичные логарифмы и натуральные логарифмы.

          Десятичные логарифмы – это логарифмы с основанием   10,   а основанием натуральных логарифмов является иррациональное и трансцендентное число   e,   которое определяется по формуле

    доказательство которой выходит за рамки школьной программы.

          Для десятичных и натуральных логарифмов используются соответственно обозначения:

    lg b       и       ln b,

    причем

    lg e = 0,43429…,

    ln 10 = 2,30259…

          Графики логарифмических функций представлены в разделе «Графики степенных, показательных и логарифмических функций» нашего справочника.

          На сайте можно также ознакомиться с нашими учебными материалами для подготовки к ЕГЭ и ОГЭ по математике.

    формул журнала — что такое формулы логарифма? Примеры

    Прежде чем изучать формулы логов, вспомним, что такое логи (логарифмы). Логарифм — это просто еще один способ записи показателей степени. Когда мы не можем решить задачу с помощью показателей, мы используем логарифмы. Существуют различные формулы логарифмов, которые выводятся с использованием законов показателей. Давайте изучим их, используя несколько решенных примеров.

    Что такое формулы журнала?

    Прежде чем приступить к изучению формул журнала, давайте вспомним несколько вещей. Существует два типа логарифмов: десятичный логарифм (который записывается как «log» и его основание равно 10, если не указано иное) и натуральный логарифм (который записывается как «ln» и его основание всегда равно «e»). Приведенные ниже формулы логарифмов показаны для десятичных логарифмов. Однако все они применимы и для натуральных логарифмов. Вот наиболее часто используемые формулы журнала .

    • журнал б 1 = 0
    • журнал б б = 1
    • log b (xy) = log b x + log b y
    • log b (x / y) = log b x — log b y
    • log b a x = x log b a
    • log b a = (log c a) / (log c b)

    Некоторые из этих правил имеют определенные имена, такие как журнал 9. 0014 b (xy) = log b x + log b y называется формулой произведения бревен. Точно так же все свойства вместе с их именами указаны в таблице ниже.

    Вывод логарифмических формул

    Вот вывод некоторых важных логарифмических формул. Мы используем законы показателей при выводе логарифмических формул.

    Формула произведения логарифмов

    Формула произведения бревен, log b (xy) = log б х + лог б у.

    Вывод:

    Предположим, что log b x = m и log b y = n. Тогда по определению логарифма

    x = b m и y = b n .

    Тогда xy = b m × b n = b m + n (по закону экспонент, a m × a n = a m + n 3 900 b m + n в логарифмическом виде, получаем

    m + n = log b xy

    Подстановка значений log b x = m и log b y = n здесь,

    log b (xy) = log b x b y

    Частная формула логарифмов

    Частная формула логарифмов: log b (x/y) = log b x — log b y.

    Вывод:

    Предположим, что log b x = m и log b y = n. Тогда по определению логарифма

    x = b m и y = b n .

    Тогда x/y = b m / b n = b m — n (по закону экспонент, a m / a n = a m — n ) 90 x/y = b m — n в логарифмическом виде, мы получаем

    m — n = log b (x/y)

    Подставляя значения log b x = m и log b y = n здесь,

    log b (x/y) = log b x — log b y

    Формула степени логарифмов

    Формула степени логарифмов говорит log b a x = x log b a.

    Вывод:

    Пусть log b a = m. Тогда по определению логарифма a = b m .

    Возведя обе стороны на x, получим

    a x = (b m ) x

    a x = b m x (по закону экспонент0040 m ) n = a mn )

    Преобразование обратно в логарифмическую форму здесь,

    лог b a x = x log b a

    Изменение базовой формулы логарифмов

    Изменение базовой формулы логарифмов говорит log b a = (log c a) / (log c4 б).

    Происхождение:

    Предположим, что log b a = x, log c a = y и log c b = z.

    Преобразование их в экспоненциальную форму

    Из (1) и (2),

    b x = c y

    (c z ) x = c y 9002 9 (из 0004 (3)) 0 зх = с у

    Так как основания одинаковые, силы тоже должны быть одинаковыми.

    zx = y (или) x = y/z.

    Подставив сюда значения x, y и z,

    log b a = (log c a) / (log c b).

    Разбивайте сложные концепции с помощью простых визуальных средств.

    Математика больше не будет сложным предметом, особенно когда вы понимаете концепции с помощью визуализаций.

    Запись на бесплатный пробный урок

    Примеры использования формул логарифмов

    Пример 1: Преобразуйте следующее из экспоненциальной формы в логарифмическую, используя логарифмические формулы. а) 5 3 = 125 б) 3 -3 = 1/27.

    Решение:

    Используя определение логарифма, = x

    Используя это,

    a) 5 3 = 125 ⇒ log 5 125 = 3

    b) 3 -3 = 1 / 27 ⇒ log

  • 3 003

    Ответ : а) log 5 125 = 3; б) лог 3 1/27 = -3.

    Пример 2: Сожмите следующее выражение в виде единичного логарифма, используя логарифмические формулы. 5 лог х + лог у — 8 лог z.

    Решение:

    Найти: Сжатую форму данного выражения в виде единичного логарифма с использованием формул логарифмирования.

    5 log x + log y — 8 log z

    = (5 log x — 8 log z) + log y (термины перегруппированы)

    = (log x 5 — log z 8 ) + log y (∵ a log x = log x a )

    = log (x 5 /z 8 ) + log y (∵ log x — log y = log (x/y) )

    = log (x 5 y/z 8 ) (∵ log x + log y = log (xy))

    Ответ : 5 log x + log y — 8 log z = log (x 5 y/z 8 ).

    Пример 3: Найдите целочисленное значение log 3 (1/9), используя логарифмические формулы.

    Решение:

    log 3 (1/9) = log 3 1 — log 3 9 (∵ log b (x / y) = log b x 4 b — log 900 )

    = 0 — log 3 3 2 (∵ log b 1 = 0)

    = — 2 log 3 3 (∵ log b 9 x a ) 014 б а)

    = -2 (1) (∵ log b b = 1)

    = -2

    Ответ: log 3 (1/9) = -2.

    Часто задаваемые вопросы о формулах логарифма

    Что такое формулы логарифмирования?

    Формулы логарифмов связаны с логарифмами и очень полезны при решении задач на логарифмы. Некоторые важные формулы журнала:

    • log b (xy) = log b x + log b y
    • log b (x / y) = log b x — log b y
    • log b a x = x log b и
    • log b a = (log c a) / (log c b)

    Как вывести формулы журнала?

    Законы экспонент используются для получения логарифмических формул. Мы также используем определение логарифма при выводе логарифмических формул. т. е. мы преобразуем логарифмическую форму в экспоненциальную форму и наоборот при выводе. Подробный вывод формул журнала можно найти в разделе «Что такое формулы журнала?» раздел этой страницы.

    Каковы применения формул журнала?

    Проблемы, которые нельзя решить с помощью свойств показателей, можно решить с помощью журналов. Формулы журнала используются либо для сжатия группы логарифмов в один логарифм, либо наоборот.

    Какая формула логарифма используется для изменения основания логарифма?

    Формула изменения базы (которая является одной из формул журнала) используется для изменения базы. Используя эту формулу, log b a = (log c а) / (лог в б). Мы видим, что основание в левой части равно «b», а основания логарифмов в правой части равны «c».

    Какая польза от формулы изменения основания (одна из логарифмических формул)?

    Вот важное использование формулы замены основания. Обычно в калькуляторах есть опции для вычисления логарифмов чисел с основанием 10 и с основанием «e». Чтобы найти логарифмы чисел с другим основанием, кроме 10 и «е», мы используем формулу замены основания. Например лог 2 3 = (лог. 3) / (лог. 2).

    Что такое натуральные логарифмические формулы?

    Формулы натуральных логарифмов такие же, как и формулы логарифмов, за исключением того, что «логарифм с некоторым основанием» заменен на «ln». Вот наиболее важные формулы натурального логарифма.

    • пер (ху) = пер х + пер у
    • пер (х/у) = пер х — пер у
    • In a x = x In a

    Формула логарифмирования — объяснение, формула, примеры решений и важные часто задаваемые вопросы

    Логарифм — это показатель степени или степень, в которую нужно возвести основание, чтобы получить заданное число. Математически логарифмы выражаются следующим образом: m — это логарифм n по основанию b, если bm = n, что также можно записать как m = logb n. Например, 43 = 64; следовательно, 3 — это логарифм 64 по основанию 4, или 3 = log464. Точно так же мы знаем, что 103 = 1000, тогда 3 = log101000. Логарифмы с основанием 10 обычно известны как обычные или бриггсовские логарифмы и просто записываются как log n. В этой статье мы обсудим, что такое логарифм, формулы логарифмов, основные формулы логарифмов, правило изменения основания, правила и формулы логарифмов, для чего используется логарифм и т. д.

     

    Правила логарифмирования 

    Существует 7 правил логарифмирования, которые полезны при расширении логарифмов, сокращении логарифмов и решении логарифмических уравнений. Семь правил логарифмов обсуждаются ниже:

    1. Правило произведения

    \[\log_{b}{(P \times Q)} = \log_{b}{P} + \log_{b}{Q} \]

     

    Логарифм произведения равен сумме логарифмов факторов.

    2. Частное правило

    \[\log_{b}{(\frac{P}{Q})} = \log_{b}{P}  —  \log_{b}{Q}\] 9{y}} = y \]

     

    Правило логарифмирования основания в степени гласит, что логарифм b с рациональным показателем равен показателю, умноженному на его логарифм.

    6. Основание степени логарифма Правило

    b log y = y

     

    Приведенное выше правило гласит, что возведение логарифма числа в основание логарифма равно числу.

    7. Правило идентификации

    \[\log_{y}{y} = 1 \]

     

    Аргумент логарифма (в скобках) аналогичен основанию. Поскольку основание равно аргументу, y может быть больше 0, но не может быть равен 0.

     

    Формулы логарифмирования

    Ниже приведены некоторые из различных формул логарифмирования, которые помогают решать уравнения логарифмирования.

     

    Основная формула логарифма

    Некоторые из различных основных формул логарифма приведены ниже:

    • \[\log_{b}{(m \times n)} = \log_{b}{m} + \ log_{b}{м}\] 9{n})} \]

     

    Сложение и вычитание

     

    Изменение базовой формулы

    .

     

    \[\log_{n}{m} = \frac{\log_{d}{m}}{\log_{d}{n}}\]

     

    Решенные примеры

    1. Решить Следующее: 2 log429

    Решение:

    Дано,

     \[2 \log_{4}{29} \]

    С помощью замены формулы основания n получаем 9{2}\] \]

    \[75x = 90\]

    \[X = \frac {90}{75}\]

    \[X = \frac {6}{5}\]

    В математике характеристики логарифма используются для решения задач логарифмирования. Многим алгебраическим характеристикам, таким как коммутативные, ассоциативные и дистрибутивные, нас учили в начальной школе. Есть пять основных особенностей логарифмических функций.

     

    Логарифмическое число связано с показателем степени и степенью, таким образом, если xn = m, то logxm = n. В результате мы также должны понимать экспоненциальный закон. Например, логарифм 10000 по основанию 10 равен 4, потому что 4 — это степень, в которую нужно возвести десять, чтобы получить 10000 : 104 = 10000, поэтому log1010000 = 4,9. 0003

     

    Мы можем представить логарифм произведения как сумму логарифмов, логарифм частного как разность логарифмов и логарифм мощности как произведение, используя эти функции.

     

    Вещественное число Логарифмы видны только в положительных действительных числах; отрицательные и комплексные числа имеют комплексные логарифмы.

     

    Применение логарифмов

    Логарифмы имеют широкий спектр применений как в математике, так и за ее пределами. Давайте рассмотрим несколько примеров того, как логарифмы используются в повседневной жизни:

    • Они используются для расчета магнитуды землетрясения.

    • Логарифмы используются для расчета уровня шума в децибелах, например звука колокола.

    • Логарифмы используются в химии для определения кислотности или уровня pH.

    • Они используются для расчета прироста денег при заданной процентной ставке.

    • Логарифмы обычно используются для расчета времени, необходимого для распада или экспоненциального развития чего-либо, например роста бактерий или радиоактивного распада.

    • Их также можно использовать в вычислениях, требующих преобразования умножения в сложение или наоборот.

     

    Вместо простого вычисления мы можем использовать таблицу логарифмов, чтобы получить логарифм целого числа. Прежде чем вычислять логарифм числа, мы должны сначала понять его характеристическую и мантиссу части.

     

    Характеристическая часть — Характеристический компонент — это целая часть числа. Любое число больше единицы имеет положительную характеристику, а если оно на единицу меньше, чем количество цифр слева от десятичной точки в данном целом числе, оно имеет отрицательную характеристику. Если число меньше единицы, характеристика отрицательна, и число на единицу больше числа нулей справа от запятой.

     

    Часть мантиссы. Часть мантиссы представляет собой десятичную часть логарифмического числа, которая всегда должна быть положительной. Если часть мантиссы имеет отрицательное значение, превратите ее в положительное значение.

     

    Как вы используете таблицу журнала?

    Процесс определения значения журнала числа с использованием таблицы журнала показан ниже. Во-первых, вы должны понять, как использовать таблицу журнала. Таблица журнала предоставляется как ресурс для определения значений.

     

    Шаг 1. Понимание идеи логарифма. Каждая таблица журнала может использоваться только на определенной основе. Логарифмическая база 10 — наиболее часто используемая форма таблицы логарифмов.

     

    Шаг 2: Определить характеристику числа и часть мантиссы. Например, чтобы получить значение log1015,27, сначала разделите характеристику и часть мантиссы.

     

    Часть характеристики = 15

    Часть мантиссы = 27

     

    Шаг 3. Используйте общую таблицу журнала. Теперь используйте строку 15, чтобы проверить столбец 2 и записать соответствующее значение. В итоге получается 1818.

     

    Шаг 4. Вычислите среднюю разницу, используя таблицу логарифмов. Проведите пальцем по столбцу средней разницы 7 и строке 15 и запишите соответствующее значение как 20.

     

    Шаг 5. Объедините значения, полученные на шагах 3 и 4. Это равно 1818 + 20 = 1838. В результате значение 1838 представляет часть мантиссы.

     

    Шаг 6. Найдите отличительный признак. Поскольку число находится в диапазоне от 10 до 100 (от 101 до 102), отличительным признаком должен быть 1.

     

    Шаг 7: Наконец, объедините часть характеристики и мантиссы, чтобы получить 1,1838.

     

    Пример

    Вот пример использования таблицы логарифмов для получения значения логарифмической функции.

     

    Определите значение log102,872.

    Решение:

    Шаг 1. Компонент характеристики равен 2, а часть мантиссы равна 872.

     

    Шаг 2. Изучите строки 28 и 7 таблицы. В итоге получается значение 4579.

     

    Шаг 3. Изучите значение средней разницы для строки 28 и среднюю разницу в столбце 2. Значение, связанное со строкой и столбцом, равно 3. получаем 4582. Это сечение мантиссы.

     

    Шаг 5: Поскольку количество цифр слева от десятичной части равно единице, характеристическая часть меньше единицы. В результате характеристическая часть равна 0

     

    Шаг 6: Наконец, соедините характеристики и части мантиссы. В результате получается 0,4582.

  • © 2015 - 2019 Муниципальное казённое общеобразовательное учреждение «Таловская средняя школа»

    Карта сайта